Sunteți pe pagina 1din 83

REPASO Nro. 1 Pag.

1 USAMEDIC 2017
REPASO ESPECIAL N 01 DE TODAS LAS AREAS PARA EL EXAMEN UNICO DE RESIDENTADO 2017
USAMEDIC 2017

AREA DE MEDICINA INTERNA

1. Indique el procedimiento ms preciso para el diagnstico etiolgico de las infecciones urinarias:


a) Hemocultivo.
b) Urocultivo.
c) Realizacin de las catalasa en la orina
d) Deteccin de nitritos en orina.
e) Deteccin de acido lactico en orina.
2. Cul es la manifestacin analtica inicial de la nefropata diabtica?
a) Microalbuminuria intermitente.
b) Proteinuria manifiesta.
c) Aumento de la creatinina srica.
d) Aumento de la filtracin glomerular.
e) Leucocituria.
3. Paciente de 54 aos presenta dolor, hinchazn, aumento de la temperatura y limitacin del movimiento de la rodilla derecha. A la
exploracin se observa una rodilla caliente, eritematosa y dolorosa a la palpacin. El lquido sinovial es de aspecto turbio y la paciente se
encuentra postrada. Cul es el diagnostico ms probable?
a) Crisis aguda de artrosis degenerativa.
b) Crisis de pseudogota.
c) Artritis sptica por S. aureus.
d) Artritis sptica por enterobacterias.
e) Episodio agudo de artritis reumatoide.
4. Mujer de 65 aos con disnea progresiva desde hace unos das. Tiene una temperatura de 39C. El mdico le ha recetado quinapril,
espironolactona, digoxina, ipratropio y salbutamol. Cul de estos incluira en su diagnstico diferencial?
a) Neumona.
b) Edema pulmonar.
c) Neumotrax espontneo.
d) Estado asmtico.
e) Hemotrax.
5. Cul es la lesin inicial en la patogenia de la fibrosis pulmonar?
a) Prdida de estructura alveolar.
b) Alveolitis.
c) Fibrosis.
d) Lesin de los vasos pulmonares.
e) Proliferacin de los neumocitos tipo I.
6. Cmo se denomina un cuadro de visin centelleante y moscas volantes junto a cefalea pulstil hemicraneal, nuseas y vmitos?
a) Migraa comn.
b) Migraa clsica.
c) Epilepsia parcial.
d) Epilepsia compleja.
e) Arteritis de Horton.
7. Al examinar la historia farmacolgica de un paciente, qu le indicara que ha sufrido una lcera?
a) Atropina.
b) Difenhidramina.
c) Famotidina.
d) Carbamazepina.
e) Diazepam.
8. La toxoplasmosis es transmitida habitualmente por:
a) Contacto sexual.
b) Fmites.
c) Animales domsticos y/o carnes contaminadas.
d) Agua.
e) Va venosa.
REPASO Nro. 1 Pag. 2 USAMEDIC 2017
9. Mujer de 35 aos padece diarrea sanguinolenta de 1 semana. Dolor abdominal intermitente de 1 mes, ahora constante y perdida de 5 kg. T
38C, FC 85 y PA 110/60; el rea cardiaca y los campos pulmonares normales; dolor abdominal difuso a palpacin. Diagnostico mas probable:
a) Colitis ulcerosa.
b) Colitis isqumica.
c) Fstula aortoentrica.
d) lcera pptica.
e) Carcinoma de colon.
10. Paciente presenta fiebre, hepatomegalia, esplenomegalia gigantesca y pancitopenia. Se debe descartar en primer lugar que padezca:
a) Leptospirosis.
b) Amebiasis.
c) Giardiasis.
d) Leishmaniasis.
e) Isosporidiasis.
11. Est trasladando a mujer de 65 aos, con insuficiencia renal, desde el albergue en que vive. Historia de anomalas previas de
laboratorio y est somnolienta y dbil. Datos de laboratorio: calcio srico de 10 mg/dl, pH de 7,28, potasio de 6,1 mEq/l. La paciente entra
en paro cardiaco despus de pasarla a la ambulancia. Despus de la adrenalina, lo primero que deber pensar en administrar es:
a) Cloruro de calcio.
b) Lactato de Ringer en embolada.
c) Sulfato de magnesio.
d) Bicarbonato sdico.
e) Dextrosa
12. Un varn de 75 aos desarroll diabetes en los ltimos 6 meses y ahora ictericia. Tuvo una perdida de peso de 5 Kg en 6 meses. Existe
una tumoracin no dolorosa en el cuadrante superior derecho que se desplaza a la respiracin. La TC muestra aumento del tamao de la
cabeza del pncreas sin defectos de llenado en el hgado. Cul es la causa mas probable de la ictericia indolora?
a) Estenosis biliar maligna.
b) Carcinoma de la cabeza del pncreas.
c) Coledocolitiasis.
d) Cirrosis heptica.
e) Pancreatitis.
13. Varn de 65 aos presenta hematemesis y melena. Historia de colelitiasis de hallazgo reciente no valorada. Historia de dolor abdominal de
3-4 semanas, ha tomado ibuprofeno 4 veces/da por 3 semanas adems de alcohol para controlar el dolor. FC 100 lpm y PA 100/60 luego de la
reanimacin. Dolor a la palpacin en cuadrante superior derecho y en epigastrio. Tacto rectal: heces con melena. Diagnostico mas probable:
a) Diverticulosis de colon.
b) Colitis ulcerosa.
c) lcera pptica.
d) Varices esofgicas.
e) lcera gstrica.
14. Una paciente de 18 aos de edad, con fiebre y malestar general de 1 semana de evolucion, acude por dolor precordial que aumenta con
la inspiracin profunda y la tos, mejora cuando se sienta inclinada hacia delante. Auscultacin cardaca: ruidos secos rozantes en
sstole y distole. Rx de trax: cardiomegalia importante sin congestin pulmonar. Diagnostico mas probable:
a) Endocarditis bacteriana.
b) Embolia pulmonar.
c) Pericarditis aguda.
d) Neumona bacteriana aguda.
e) Rotura de aneurisma congnito de aorta con derrame pericrdico.
15. A un paciente se le detecta un impulso apical palpable con ruidos cardiacos apagados. Radiografia: un leve aumento cardaco.
Cateterismo: morfologa dip-plateau (raiz cuadrada) y presin diastlica grande en el ventrculo izquierdo. Diagnostico mas probable:
a) Pericarditis constrictiva.
b) Miocardiopata restrictiva.
c) Miocardiopata dilatada.
d) Diseccin artica.
e) Miocardiopata hipertrfica.
REPASO Nro. 1 Pag. 3 USAMEDIC 2017
16. Uno de los siguientes es un farmaco que debe usarse en el tratamiento de la leucemia mieloide crnica:
a) Busulfn.
b) Uretano.
c) Clorambucilo.
d) Interfern alfa.
e) Ametopterina.
17. Un minero de 49 aos presenta tos y disnea de esfuerzo de varios meses. Rx de trax: ndulos de 3-5 mm de dimetro y un ndulo
calcificado. En las pruebas complementarias aparece disminucin de la capacidad vital y de la difusin, sin alteraciones significativas en
el FEV1 y Mantoux positiva. La actitud teraputica correcta es:
a) Corticoides.
b) Inmunosupresores.
c) Isoniazida.
d) Sales de oro.
e) Cloroquina.
18. Cul de los siguientes hallazgos en el electrocardiograma determina el diagnstico de fibrilacin auricular?
a) Complejo QRS ancho.
b) Ondas P bifsicas.
c) Supradesnivel del segmento ST.
d) Ausencia de ondas P.
e) Ondas T aplanadas.
19. En un varn con hematocrito de 60%, leucocitos 12500/mm3, plaquetas 450000/mm3, esplenomegalia de 2 cm por debajo del reborde costal,
fosfatasa alcalina granuloctica incrementada y cromosoma Philadelfia negativo, cul sndrome mieloproliferativo es mas probable?
a) Leucemia mieloide crnica.
b) Mielofibrosis.
c) Trombocitemia esencial.
d) Policitemia vera.
e) Leucemia aguda.
20. Varn de 35 aos evita salir a reuniones por temor a ser rechazado o por sntomas de ansiedad que llegan a ser muy intensos al punto
de sentir que le falta aire. Cul es el trastorno que est ms relacionado al caso descrito?
a) Ansiedad generalizada.
b) Depresin mayor.
c) Agorafobia.
d) Fobia social.
e) Pnico
21. Paciente de 50 aos, alcohlico, comienza bruscamente con escalofros y fiebre. Tos productiva y dolor pleurtico. Esputo difcil de expectorar
y denso, color marrn oscuro. Rx de trax: lesin en el lbulo superior derecho que abomba la cisura. Microorganismo productor del cuadro:
a) Klebsiella.
b) Neumococo.
c) Mycoplasma.
d) Staphylococcus.
e) Proteus.
22. Un paciente que presenta fiebre y rigidez de nuca, es sometido a un anlisis de su LCR. Se observa un lquido claro con 300 linfocitos /
microlitro y glucosa normal, cul sera su diagnstico?
a) Lquido normal.
b) Meningitis por meningococo.
c) Meningitis por haemophilus.
d) Meningitis tuberculosa.
e) Meningitis vrica.
23. Un vagabundo es hallado inconsciente en un parque. Se le hace una serie de estmulos asignndole un valor en la escala de Glasgow:
Apertura de los ojos tras intensos estmulos dolorosos. Ausencia de la respuesta verbal a las preguntas insistentes. Respuesta extensora
de la extremidad a los estmulos dolorosos. El puntaje en Glasgow es:
a) 5.
b) 4.
c) 6.
d) 3.
e) 7.
REPASO Nro. 1 Pag. 4 USAMEDIC 2017
24. Un paciente de 25 aos de hbito homosexual y con anticuerpos frente al VIH llega por odinofagia y disfagia de una semana. En la
cavidad oral hay presencia de placas blanquecinas confluentes en paladar, vula y faringe de dos das de evolucin que al arrancarlas
dejan una superficie enrojecida, dolorosa y sangrante. Diagnostico mas probable:
a) Leucoplasia oral.
b) Liquen plano.
c) Carcinoma oral.
d) Sarcoma de Kaposi.
e) Candidiasis oral.
25. Un varn de 70 aos de edad acude por coma de instauracin brusca. Durante el mes anterior, haba presentado varios episodios
transitorios de diplopa, vrtigo y ataxia. Cul es la causa mas probable del coma que padece?
a) Trombosis de la arteria basilar.
b) Hemorragia protuberancial.
c) Trombosis de la arteria cerebral media.
d) Hemorragia subaracnoidea.
e) Hematoma putaminal.
26. Un varn de 49 aos, con historia de hipertensin arterial, al jugar al baloncesto presenta cefalea, vmitos y hemiparesia, el dficit focal
aumenta con el transcurso del tiempo. Cmo se denomina esta patologa?
a) Hemorragia intracerebral.
b) Ataque isquemico transitorio.
c) Dficit neurologico isquemico reversible.
d) Hemorragia subaracnoidea.
e) Hematoma subdural.
27. Una mujer de 35 aos de edad presenta diagnostico de sindrome ansioso-depresivo. Estaba muy deprimida e ingiri 10 tabletas de 5
mg de Diazepam y 20 de 2 mg de Lorazepam. Es llevada a urgencias. Qu antdoto especfico le administrara?
a) Naloxona.
b) Flumazenil.
c) N-acetilcistena.
d) Atropina.
e) Vitamina K.
28. Cul es el agente cuya accin se considera efectiva para la amebiasis intestinal y para la amebiasis heptica?
a) Cloroquina
b) Yodohidroxiquinoleina
c) Nimorazol
d) Arsenicales
e) Metronidazol
29. Mujer de 24 aos con varios das de fiebre, de mayor intensidad por las tardes. Recibe tratamiento sintomtico sin mejoria; hace 3 das
se agrega dolor a nivel sacro iliaco, persistiendo el cuadro febril. Cul es el diagnstico ms probable?
a) Brucelosis.
b) Salmonelosis.
c) Malaria.
d) Pielonefritis aguda.
e) Leptospirosis.
30. Cul es el tratamiento de eleccin de la enfermedad de Chagas en la fase aguda?
a) Suramina.
b) Pentamidina.
c) Triparsamida.
d) Nifurtimox.
e) Eflornitina.
31. Cul de las siguientes alternativas es la causa ms frecuente de hemorragia subaracnoidea?
a) Vasculitis.
b) Aneurisma cerebral.
c) Hemofilia A.
d) Coagulacin intravascular diseminada.
e) Prpura trombocitopnica.
REPASO Nro. 1 Pag. 5 USAMEDIC 2017
32. En cul de las siguientes localizaciones anatmicas es ms frecuente la pitiriasis versicolor?
a) Codos y rodillas.
b) Dorso de manos y pies.
c) Pecho y espalda.
d) Cara y cuello.
e) Grandes pliegues.
33. Un varn de 75 aos de edad presenta una lesin papulosa brillante en la mejilla derecha de 6 meses; histolgicamente: cordones de
clulas neoplsicas epiteliales que se disponen en la periferia en empalizada. Diagnstico mas probable:
a) Sarcoma de Kaposi.
b) Melanoma maligno.
c) Carcinoma basocelular.
d) Carcinoma epidermoide.
e) Fibrosarcoma.
34. Paciente de 57 aos de edad acude a revisin anual rutinaria al centro de salud, pesa 84 kg y mide 1.75 m. Se encuentra segn sus
propias manifestaciones bien de salud, hace poco ejercicio y come normalmente. Su presin arterial es 155/90. Al explorar el fondo de ojo
previa dilatacin pupilar con tropicamida, observamos en el polo posterior de ambos ojos, rodeando el rea macular una serie de puntos
rojos y blancos junto a pequeas manchas blancas y rojas. Esto nos hace pensar que:
a) Es retinopata hipertensiva , controlaremos la presin arterial y/o recomendamos un Holter.
b) Se trata de una degeneracin macular asociada a la edad, remitimos de urgencia al oftalmlogo.
c) Podria ser una retinitis por VIH, solicitaremos la pruebas correspondientes.
d) Es una retinopata diabtica, indicaremos una curva de glucemia.
e) Pensamos en uvetis de posible origen luetico.
35. Un paciente desnutrido presenta el siguiente patrn de hormonas tiroideas: T4 y TSH normales, T3 disminuida y rT3 elevada. No existe
afectacin de otras hormonas hipofisarias. Qu enfermedad presenta?
a) Tiroiditis de Hashimoto.
b) Hipotiroidismo terciario.
c) Hipotiroidismo secundario.
d) Sndrome del eutiroideo enfermo.
e) Hipotiroidismo primario.
36. Cuando encontramos unas concentraciones sricas de T3 normal, T4 libre ligeramente disminuida y TSH elevada con respuesta
aumentada a la TRH, nos hallamos ante un:
a) Hipotiroidismo primario.
b) Hipotiroidismo hipofisario.
c) Hipotiroidismo hipotalmico.
d) Funcionamiento tiroideo dentro del lmite de la normalidad.
e) Hipertiroidismo.
37. Qu actitud seguir ante un diabtico en tratamiento con 2 inyecciones diarias de insulina que presenta hiperglucemia matutina y
episodios de mareo con sudoracin fra por las noches?
a) Disminuir la ingesta de hidratos de carbono en la cena.
b) Aumentar la dosis de insulina para controlar la hiperglucemia.
c) Reducir la dosis nocturna de insulina.
d) Aumentar dosis de insulina de la maana.
e) Tratamiento intensivo de la neuropata diabtica.
38. Un paciente varn de 27 aos, que tras ser diagnosticado hace 2 meses de un feocromocitoma, se le descubre un ndulo tiroideo fro en una
gammagrafa tiroidea. En la analtica resulta un aumento desproporcionado de calcitonina plasmtica. En qu patologa habra que pensar?
a) Adenoma tiroideo.
b) Carcinoma medular de tiroides.
c) Carcinoma papilar de tiroides.
d) Carcinoma folicular de tiroides.
e) Carcinoma paratiroideo.
39. Un varn de 18 aos de edad llega a Urgencias. Sus amigos relatan que estuvieron bebiendo alcohol y que su amigo se desmayo hace
casi 2 horas y esta difcil despertarle progresivamente. Cul de los siguientes es ms til en el manejo inmediato de este paciente?
a) Sodio serico.
b) Glucosa serica.
c) Nivel de alcohol sanguineo.
d) Calcio serico.
e) Prueba de droga serica.
REPASO Nro. 1 Pag. 6 USAMEDIC 2017
40. Cul se considera la alteracin renal ms frecuente en los pacientes que presentan una gota crnica?
a) Hialinizacin del glomrulo.
b) Pielonefritis de repeticin.
c) Nefroangioesclerosis.
d) Acidosis tubular renal secundaria.
e) Aparicin de tofos en medula o pirmides.
41. Un empleado bancario de 28 aos se somete a prueba por infertilidad que revelan oligospermia. En una exploracin posterior, se
encuentra que ha sufrido repetidos ataques de tos desde la niez y episodios recurrentes de pancreatitis. Los dedos en palillo de tambor
son evidentes. Qu prueba tiene mayores posibilidades de revelar la causa de esta enfermedad crnica del pulmn?
a) Radiografa torcica.
b) Radiografa del hmero.
c) El cloruro en el sudor se eleva por arriba de 80 meq/L.
d) El cloruro en el sudor disminuye por debajo de 50 meq/L.
e) Aspergillus en el esputo.
42. De las siguientes causas de bronquiectasias, cul es la mas frecuente?
a) Neoplasias pulmonares.
b) Agammaglobulinemia.
c) Infecciones broncopulmonares.
d) Fibrosis qustica.
e) Sndrome de disquinesia ciliar.
43. Un joven que presenta un cuadro de tos seca intensa, malestar general y fiebre de 38.5C. Se le diagnostica de sndrome gripal
inespecfico. Inicialmente se le da un tratamiento con amoxicilina. Al da siguiente ingresa a Urgencias por agudizacin de su proceso.
Sistemtico de sangre: Hb 9 g/dl. Rx de trax: patrn retculo-nodular. Qu patologa padece con ms probabilidad?
a) Neumona por neumococo.
b) Neumona por pneumocystis jiroveci.
c) Fiebre Q.
d) Neumona por Klebsiella.
e) Neumona por Mycoplasma pneumoniae.
44. Cul es el trastorno inmunolgico ms caracterstico de la enfermedad de Hodgkin?
a) Falta de respuesta a los antgenos polisacridos.
b) Incremento de linfocitos T.
c) Hipogammaglobulinemia.
d) Descenso del complemento srico.
e) Depresin de la inmunidad celular.
45. Ante un paciente que sufre de epilepsia, la etosuximida es mas util en el tratamiento de:
a) Sobredosis por fenobarbital.
b) Convulsiones de ausencia (pequeo mal).
c) Convulsiones acineticas.
d) Convulsiones tonico-clonicas.
e) Convulsiones parciales complejas.
46. Paciente acude por oliguria de 24 horas de evolucin. Analtica: urea plasmtica 120 mg/dl, creatinina 2 mg/dl, Na+ urinario 10 mEq/L y
sedimento normal. Cul es el diagnstico ms probable?
a) Necrosis tubular aguda
b) Insuficiencia renal aguda prerrenal
c) Obstruccin urinaria
d) Glomerulonefritis aguda
e) Vasculitis renal
47. Un paciente con hipertension de aparicin reciente, y mal estado general, ingresa para la valoracin de una hematuria y en el sedimento
urinario se comprueba proteinuria con cilindros hemticos y hemates dismrficos. Indique su diagnstico de sospecha:
a) Infeccin urinaria.
b) Glomerulonefritis rpidamente progresiva.
c) Tuberculosis renal.
d) Necrosis papilar.
e) Litiasis renal.
REPASO Nro. 1 Pag. 7 USAMEDIC 2017
48. Mujer de 29 aos, larga historia de asma leve. Ahora con exacerbacin y episodios recurrentes de obstruccin bronquial, fiebre, malestar
general y expectoracin con tapones mucosos de color pardusco. Sibilancias bilaterales. Sospecha de infeccin. Rx torax: infiltrados en lbulos
pulmonares superiores. Conteo de eosinfilos 2000/ml y anticuerpos precipitantes sricos para Aspergillus (+). Paso siguiente ms apropiado:
a) Terapia antihelmntica
b) Tratamiento con curso corto de glucocorticoides sistmicos
c) Tratamiento desensibilizante
d) Altas dosis de glucocorticoides inhalados
e) Tratamiento a largo plazo con glucocorticoides sistmicos
49. Varn de 65 aos presenta astenia y anorexia, presenta esplenomegalia gigante y molestias repetidas en el hipocondrio izquierdo. Los
leucocitos alcanzan la cifra de 200.000 /mm3 sin hiato. Diagnostico de presuncin:
a) Policitemia vera.
b) Trombocitosis esencial.
c) Leucemia mieloide cronica.
d) Leucemia linfocitica aguda.
e) Leucemia mieloide aguda.
50. El agente etiolgico del granuloma inguinal es:
a) Treponema pallidum.
b) Haemophilus ducreyi.
c) VHS tipo 2.
d) Chlamydia trachomatis.
e) Calymatobacterium granulomatis.
51. Una mujer de 24 aos de edad, luego de 1 ao de su inicio en las relaciones sexuales, presenta astenia marcada, fiebre alta y cefalea.
Tambin adenopatas cervicales y faringitis exudativa con placas blanquecinas recubriendo la zona amigdalar. Linfocitosis atpica.
Hepatoesplenomegalia discreta. Serologa: anticuerpos heterfilos positivos. Diagnostico mas probable:
a) Cuadro mononuclesico por CMV.
b) Leucemia linfoctica aguda.
c) Sfilis diseminada.
d) Faringitis bacteriana supurativa.
e) Infeccin por VEB.
52. Entre las neoplasias asociadas al SIDA, cul es la mas frecuente despus del sarcoma de Kaposi?
a) El linfoma de Hodgkin.
b) El cncer gstrico.
c) El cncer de pulmn.
d) El linfoma no Hodgkin.
e) La leucemia de clulas peludas.
53. Cul de las siguientes condiciones se asocia con un primer ruido con intensidad incrementada?
a) Estenosis mitral grave.
b) Bradicardia sinusal.
c) Bloqueo auriculoventricular de primer grado.
d) Sindrome de Wolff-Parkinson-White.
e) Todas las anteriores.
54. En qu enfermedad vrica se encuentran los cuerpos de Negri?
a) Hepatitis A.
b) Poliomielitis.
c) Rabia.
d) Sarampin.
e) Proteus mirabilis.
55. Qu tratamiento medicamentoso se debe recetar en una anemia hipocroma con aumento de sideroblastos y sideroblastos en anillo en
la mdula sea?
a) Hierro.
b) cido flico.
c) Vitamina B12.
d) Desferroxamina.
e) Vitamina B6.
REPASO Nro. 1 Pag. 8 USAMEDIC 2017
56. Con cul de los siguientes diagnsticos relacionara la fuga de ideas?
a) Distimia
b) Depresin reactiva.
c) Depresin endgena.
d) Episodio maniaco
e) Duelo patolgico.
57. En cul de las siguientes enfermedades se ha involucrado niveles sericos de IgE elevados?
a) Sarcoidosis.
b) Lupus eritematoso.
c) Roscea.
d) Psoriasis.
e) Dermatitis atpica.
58. Un paciente desarrolla una gran ansiedad anticipatoria cuando se enfrenta a la necesidad de tener que actuar. Presenta miedo a
situaciones en que est expuesto al examen de los dems. Cul sera su diagnostico?
a) Trastorno obsesivo compulsivo.
b) Deja vu.
c) Fobia social.
d) Agorafobia.
e) Acrofobia.
59. La hipercalcemia es grave cuando el calcio plasmtico es mayor de:
a) 10 mg/dL.
b) 10 a 12 mg/dL.
c) 12 a 14 mg/dL.
d) 14 mg/dL.
e) 16 mg/dL.
60. Cul es la primera causa de meningitis de la comunidad en adultos?
a) Meningococo.
b) Neumococo.
c) Haemophillus influenzae.
d) Grmenes gramnegativos.
e) Listeria.
61. Un varn padece ascitis, con una presin supraheptica enclavada elevada y la libre normal. Cul es la etiologa mas frecuente de este
cuadro clnico?
a) Esquistosomiasis.
b) Sndrome de Banti.
c) Cirrosis heptica.
d) Sndrome de Budd Chiari.
e) Trombosis portal.
62. La manifestacin principal de la insuficiencia pancretica es:
a) Desequilibrio de la secrecin del duodeno
b) Malabsorcin de grasa y esteatorrea
c) Hipercloremia
d) Disminucin de bicarbonato
e) Mala digestin de los almidones
63. Cualquiera que sea la causa y el tipo de choque, el resultado final es:
a) Disminucin del gasto cardiaco con una taquicardia acentuada.
b) Hipotensin, seguida de deterioro del riego sanguneo tisular e hipoxia tisular.
c) Disminucin del volumen sanguneo e hipoperfusin miocrdica.
d) Infarto al miocardio, seguido de una isquemia generalizada.
e) Vasodilatacin perifrica y estancamiento de la sangre.
64. Una joven de 18 aos llega aquejada de palpitaciones de presentacin sbita, mientras estaba viendo la televisin, que se
acompaan de ansiedad y dolor de pecho cediendo de manera sbita. En qu situacin nos encontraremos con mayor probabilidad?
a) Taquicardia sinusal.
b) Taquicardia paroxstica supraventricular.
c) Hipocondra.
d) Fibrilacin auricular.
e) Taquicardia supraventricular con bloqueo AV.
REPASO Nro. 1 Pag. 9 USAMEDIC 2017
65. Sobre el tratamiento de la enfermedad inflamatoria intestinal (EII) es correcto:
a) El componente activo de la sulfasalazina es la sulfapiridina.
b) El tratamiento urgente del paciente crtico se hace con 2g de sulfasalazina.
c) El tratamiento de mantenimiento de la enfermedad de Crohn es con esteroides.
d) El tratamiento de mantenimiento de la colitis ulcerosa es con esteroides.
e) El Metronidazol es eficaz en el Crohn con fstulas perianales.
66. Cul de los siguientes rganos se afecta frecuentemente en la roscea?
a) Pulmn
b) Corazn.
c) Ojos.
d) Odos.
e) Articulaciones.
67. Una mujer de 59 aos, cuyo nico padecimiento es HTA desde hace 10 aos, tratada y controlada con un B-bloqueante (atenolol). Hace 1
ao inicia con temblor progresivo en el miembro superior izquierdo. Las nicas anomalas patentes son el referido temblor, que se observa en
reposo y actitud, leve hipertona y lentitud de movimientos repetitivos en las 4 extremidades. Explicacin mas probable del cuadro:
a) Efecto adverso del atenolol.
b) Parkinsonismo arteriosclertico.
c) Parkinsonismo idioptico.
d) Proceso expansivo de los ganglios basales izquierdos.
e) Temblor esencial
68. La causa que con mayor frecuencia produce gastritis es:
a) Abuso del alcohol
b) Tabaquismo
c) Antiinflamatorios no esteroideos
d) Infeccion por Helicobacter pylori
e) Radiacin
69. Un varn de 62 aos de edad marcha con pasos cortos y tiene frecuentes cadas, abulia, perdida de memoria y deterioro de lenguaje,
incontinencia urinaria e hipertona. Diagnostico mas probable:
a) Enfermedad de Creutzfeldt-Jakob.
b) Enfermedad de Alzheimer.
c) Hidrocefalia normotensiva.
d) Demencia vascular.
e) Enfermedad de Pick.
70. Por lo general, el accidente cerebrovascular emblico proviene de:
a) Vegetaciones valvulares
b) Trombos murales
c) mbolos paradjicos
d) Tumores cardiacos
e) Grandes vasos
71. Una de las siguientes NO se asocia con el antgeno HLA-B 27:
a) Espondilitis anquilopoytica.
b) Fiebre reumtica.
c) Sndrome de Reiter.
d) Enfermedad intestinal inflamatoria.
e) Artritis psorisica.
72. Un varn de 33 aos de edad nunca ha sido vacunado para la hepatitis B. Las pruebas serolgicas evidencian antgeno de superficie de
la hepatitis B (HbsAg) negativo y un anticuerpo positivo para el antgeno de superficie. Para cul de las siguientes condiciones son ms
apropiados estos resultados?
a) Infeccin previa por virus de hepatitis B.
b) Hepatitis crnica activa.
c) Infeccin aguda por hepatitis B.
d) Mal pronstico.
e) Necesidad de vacunarse para hepatitis B.
REPASO Nro. 1 Pag. 10 USAMEDIC 2017
73. Respecto a un examen bacteriolgico, el mtodo de cultivo de Lowenstein-Jensen proporciona un diagnostico de certeza de:
a) Difteria.
b) Bartonelosis.
c) Malaria.
d) Tuberculosis.
e) Esquistosomiasis.
74. Varn de 70 aos presenta dolor anginoso tpico durante la realizacin de un esfuerzo. Se ausculta un soplo sistlico en la base y en el
EKG: no se observan cambios elctricos significativos. Coronariografa: vasos normales. Se sospecha de:
a) Insuficiencia mitral.
b) Comunicacin interauricular.
c) Infarto agudo de miocardio.
d) Angina de Prinzmetal.
e) Estenosis artica.
75. Una mujer de 25 aos de edad presenta fiebre, sudacin nocturna y dolores musculares desde hace un mes. Previamente estaba bien.
A la exploracin tiene adenopatas axilares y cervicales, pero no artritis o hepatoesplenomegalia. En la actualidad trabaja en una granja de
cerdos. Las pruebas para VEB, CMV y VIH son negativas. Un estudio serolgico confirma el diagnstico. Diagnostico mas probable:
a) Brucelosis.
b) Histoplasmosis.
c) Leptospirosis.
d) Mononucleosis infecciosa.
e) Tularemia.
76. La afasia total o global, de causa vascular, se produce en una de las siguientes situaciones:
a) Oclusin de la arteria cartida interna y/o arteria cerebral media derecha.
b) Oclusin de la arteria cartida interna y/o arteria cerebral media izquierda.
c) Oclusin de la arteria cerebral anterior derecha
d) Oclusin de la arteria cerebral anterior izquierda.
e) Oclusin simultanea de ambas arterias cerebrales anteriores.
77. Cul es la primera prueba diagnstica que debe realizarse en pacientes con sospecha clnica de insulinoma?
a) Prueba del ayuno prolongado.
b) Prueba del ejercicio.
c) Determinacin de la insulinemia basal.
d) Determinacin del pptido C basal.
e) Prueba de tolerancia oral a la glucosa prolongada.
78. Cul de los siguientes procesos que cursan trastornos del equilibrio cido-base presenta una orina alcalina?
a) Acidosis tubular renal clsica.
b) Acidosis tubular renal proximal.
c) Alcalosis respiratoria.
d) Alcalosis metablica.
e) Todos los anteriores.
79. El marcador morfolgico de la progresin de una hepatitis crnica es:
a) La hepatitis de la interfase.
b) La esteatosis.
c) La densidad del infiltrado inflamatorio portal.
d) La necrosis lobulillar.
e) La destruccin ductal.
80. El desplazamiento es un mecanismo de defensa tpico en uno de los siguientes trastornos:
a) Histeria.
b) Fobias.
c) Esquizofrenia.
d) Ansiedad.
e) Neurosis obsesiva.
81. Con respecto a la anemia de procesos crnicos o anemia de la inflamacin, seale la opcin correcta:
a) La sntesis de hepcidina est aumentada.
b) La anemia es caractersticamente macroctica.
c) Los niveles de ferritina estn disminuidos.
d) Los niveles de transferrina estn elevados.
e) Debe tratarse con hierro intravenoso.
REPASO Nro. 1 Pag. 11 USAMEDIC 2017
82. Cul de los siguientes cuadros clnicos NO se asocia con toxoplasmosis?
a) Abscesos cerebrales en personas con SIDA.
b) Miocarditis en trasplantados cardiacos.
c) Coriorretinitis en adultos inmunocompetentes.
d) Colitis asociada a inmunodeficiencia humoral.
e) Sndrome mononuclesico en personas sanas.
83. Ante los hallazgos que cabe esperar del anlisis del lquido cefalorraqudeo en un cuadro de meningitis bacteriana, NO se encuentra:
a) Glucosa <40 mgr/dL.
b) Abundantes hemates.
c) Protenas >45 mgr/dL.
d) Cultivo positivo en el 40% de los casos.
e) Presin de apertura >180 cm de H2O.
84. Cul es el procedimiento ms adecuado para llegar al diagnstico etiolgico de una artritis de rodilla acompaada de fiebre, dolor muscular e
inflamacin migratoria en otras articulaciones junto a lesiones vesculo-pustulosas de la piel en una mujer joven y previamente sana?
a) Cultivo del lquido sinovial.
b) Cultivo de sangre.
c) Cultivo de endocrvix.
d) Biopsia cutnea.
e) Determinacin de auto anticuerpos.
85. Cul de los siguientes microorganismos que infectan el tracto intestinal causa bacteriemia con mayor frecuencia?
a) Vibro cholerae.
b) Salmonella enteritidis.
c) Campylobacter jejuni.
d) Yersinia enterocoltica.
e) Shigella flexneri.
86. Qu tipo de paciente tiene menos riesgo de desarrollar tuberculosis activa?
a) Varn de 40 aos con infeccin por el virus de la inmunodeficiencia humana con linfocitos CD4 del 1%.
b) Mujer de 65 aos con artritis reumatoide en tratamiento con frmacos anti TNF.
c) Mujer de 50 aos con trasplante renal.
d) Nio de 12 aos con padre diagnosticado de tuberculosis bacilfera y que en el estudio de contactos presenta Mantoux positivo de 20 mm.
e) Varn de 45 aos, no vacunado de BCG, que al comenzar un nuevo trabajo, le detectan un Mantoux de 12 mm.
87. Cul, entre los siguientes, es un virus persistente que puede permanecer en latencia y reactivarse?
a) El virus de la hepatitis A.
b) El virus respiratorio sincitial.
c) El citomegalovirus.
d) El virus de la poliomielitis.
e) El rotavirus.
88. Cul es el tumor maligno en el que con mayor frecuencia degenera un nevus sebceo?
a) Carcinoma espinocelular.
b) Carcinoma basocelular.
c) Carcinoma sebceo.
d) Carcinoma apocrino.
e) Carcinoma ecrino.
89. Un hombre de 65 aos presenta un herpes zoster en la cara. Cundo debe pedirse consulta con un oftalmlogo?
a) Presenta lesiones en la punta y lado de la nariz.
b) El nervio facial y el auditivo estn afectados.
c) Los nervios supraorbitales estn afectados
d) El paciente presenta un sndrome de Ramsay-Hunt.
e) La rama maxilar del 5 nervio est afectada.
90. La leucoplasia oral vellosa esta ocasionada, en presencia del VIH, por:
a) Diplococcus gonorrohoeae.
b) Tricomonas vaginalis.
c) Herpes simple tipo II.
d) Papiloma virus.
e) Virus Epstein-Barr.
REPASO Nro. 1 Pag. 12 USAMEDIC 2017
91. El tratamiento antirretroviral en un paciente VIH+ con carga viral de 575.000/mm3 y una cifra de linfocitos de CD4 de 450/mm3:
a) Es obligado.
b) Slo estara indicado si la carga viral fuera superior a 1 milln de copias/ml.
c) En ningn caso se prescribira si los CD4+ son> 200/ml.
d) Est indicado si el paciente lo desea.
e) En estas condiciones slo estara indicado en el contexto de un estudio clnico prospectivo.
92. Cul de las siguientes situaciones NO es una indicacin de la quimioprofilaxis de la tuberculosis?
a) Menores de 35 aos con test de la tuberculina negativa y contacto con un paciente bacilifero.
b) Menores de 35 aos con test de la tuberculina positivo y contacto con un paciente bacilifero.
c) Pacientes inmunodeprimidos de cualquier edad con test de la tuberculina negativo y contacto con un paciente bacilfero.
d) Pacientes que van a recibir tratamiento con anticuerpos monoclonales anti TNF-alfa y con test de la tuberculina negativo.
e) Pacientes con infeccin por el virus de la inmunodeficiencia humana (VIH) y con test de la tuberculina positivo.
93. Cual de las siguientes nefropatas ocurre con mayor frecuencia despus del trasplante renal?
a) Esclerosis segmentaria y focal.
b) Glomerulonefritis membranosa.
c) Glomerulonefritis por anticuerpos antimembrana basal.
d) Glomerulonefritis mesangial lgA.
e) Glomerulonefritis membrano proliferativa tipo II.
94. Un paciente de 70 aos con historia de insuficiencia cardiaca consulta por disnea. La radiografa de trax demuestra un derrame
pleural. En cual de las siguientes situaciones estara indicado realizarle una toracocentesis diagnstica?
a) El derrame es bilateral.
b) El paciente tiene disnea de reposo.
c) El paciente tiene dolor torcico unilateral que en ocasiones se resuelve espontneamente.
d) El derrame ocupa ms de 1/3 del hemotrax.
e) En general es conveniente realizarla de forma rutinaria.
95. Un paciente de 65 aos diagnosticado de EPOC es trasladado al hospital por disnea progresiva y expectoracin purulenta. Durante el
traslado en la ambulancia se le administra oxigeno y fluidoterapia. A su llegada el paciente esta obnubilado y tembloroso. Se auscultan
crepitantes en la base derecha. La gasometra arterial muestra un pH 7,08 PO2 90, PCO2 106. El tratamiento inicial mas adecuado seria:
a) Retirar oxigeno y administrar bicarbonato sdico.
b) Reducir flujo de oxigeno a 1L/min y administrar bicarbonato sdico.
c) Mantener flujo de oxigeno y administrar acetazolamida.
d) Intubacin y ventilacin mecnica.
e) Administrar epinefrina IV y bicarbonato toxico.
96. Un paciente de 35 aos es trado al servicio de urgencias tras sufrir un accidente de trfico. No abre los ojos ante estmulos dolorosos.
No obedece rdenes y emite sonidos incomprensibles. Su brazo derecho aparece deformado y no responde a estmulos dolorosos. Sin
embargo, su mano izquierda responde en un movimiento intencionado hacia el estimulo. Su escala de coma de glasgow es:
a) 12.
b) 9.
c) 5.
d) 8.
e) 2.
97. Un hombre de 64 aos ha tenido dolor abdominal intermitente como resultado de una lcera duodenal durante los ltimos 6 aos. Los
sntomas recurren 6 semanas antes de la admisin. A qu grupo es ms probable que pertenezca?
a) A y secretor (antgeno de grupo sanguneo en lquido corporal).
b) B y antgeno de Lewis.
c) AB.
d) O y no secretor.
e) O y secretor.
98. Un varn de 45 aos de edad, tras una analtica de control, presenta: hematocrito del 60% con una hemoglobina de 17 g/dl, 555.000
plaquetas, una saturacin de oxgeno del 96%. La fosfatasa alcalina leucocitaria es de 150. Llega por mareo, cefalea y prurito. Exploracin:
una rubicundez facial y una esplenomegalia. El diagnostico mas probable es:
a) Trombocitosis esencial.
b) Policitemia vera.
c) Mielofibrosis idioptica.
d) Poliglobulia del fumador.
e) Eritrocitosis del fumador.
REPASO Nro. 1 Pag. 13 USAMEDIC 2017
99. En caso de tener que utilizar fenobarbital a un paciente en tratamiento por epilepsia, la principal limitacin de uso de este agente es:
a) Aparicin de anemia megaloblstica.
b) Aparicin de ataxia.
c) Aparicin de sedacin.
d) Disminucin de la libido.
e) Erupcion cutnea.
100.Varn fumador de 30 cigarrillos/da desde hace ms de 45 aos; presenta un dolor torcico con ptosis palpebral, miosis, enoftalmos y
anhidrosis de la hemicara izquierda. Causa mas probable:
a) Carcinoma epidermoide.
b) Adenocarcinoma.
c) Carcinoma de clulas grandes.
d) Carcinoma de clulas pequeas.
e) Carcinoma bronquioloalveolar.
101.La personalidad caracterizada por no aceptar sus limitaciones culpando a las dems personas de sus fracasos y fallas, y en la que hay
una sobrestimacin del YO, es:
a) Histrinica.
b) Narcisista.
c) Antisocial.
d) Limite.
e) Paranoide.
102.Una mujer padece disnea, hipotensin, palpitaciones. Tiene un ECG con unos complejos QRS ensanchado a 240 lpm. Seale el
diagnstico mas probable de entre los siguientes:
a) Fibrilacin auricular.
b) Flutter con conduccin 2:1.
c) Taquicardia de la unin.
d) Sndrome de Wolff-Parkinson-White.
e) Taquicardia helicoidal.
103.Causa mas frecuente del Cor pulmonale crnico:
a) Insuficiencia ventricular izquierda.
b) Estenosis mitral.
c) Embolia pulmonar.
d) Limitacin crnica al flujo areo.
e) Cardiopata valvular congnita.
104.En un enfermo portador de una grave insuficiencia cardiaca por una miocardiopata dilatada, presumiblemente alcohlica. Cul de
los siguientes factores considera que sera ms til para detener o incluso hacer remitir, la progresin de su enfermedad?
a) Supresin del alcohol.
b) Anticoagulantes.
c) Digoxina.
d) Diurticos.
e) Captopril.
105.En un paciente afecto del sndrome metablico, cul de los siguientes descartara?
a) Hiperinsulinismo.
b) Disminucin de HDL- Colesterol.
c) Hipertrigliceridemia.
d) Resistencia a la insulina.
e) Hipotensin arterial.
106.Paciente, luego de un catarro de vas areas altas, presenta astenia, malestar general y dolor de inicio insidioso referido a la mandbula y
occipucio, que se prolonga 2 semanas. Marcada sensibilidad y nodularidad del tiroides que afecta a ambas regiones glandulares. Laboratorio:
aumento de VSG, aumento de T3 y T4 con TSH indetectable y disminuida captacin de yodo radiactivo. Diagnostico mas probable:
a) Tiroiditis de Hashimoto.
b) Tiroiditis de Riedel.
c) Tiroiditis pigena.
d) Tiroiditis crnica ms una tiroiditis transitoria.
e) Tiroiditis subaguda granulomatosa.
REPASO Nro. 1 Pag. 14 USAMEDIC 2017
107.Varn de 18 aos llega por una crisis generalizada tnico-clnica. En los 6 meses previos sufre sacudidas musculares involuntarias en
los miembros superiores, por lo que se le caen objetos de las manos. El hermano mayor padece epilepsia con crisis generalizadas tnico-
clnicas. EEG: descargas de polipunta onda generalizada, ms frecuentes con la estimulacin luminosa intermitente. Tratamiento adecuado:
a) Carbamazepina.
b) Difenilhidantona
c) Acido valproico.
d) Fenobarbital.
e) Etosuximida.
108.Cul de los siguientes es el elemento de exploracin fsica mas til para el diagnostico de estenosis aortica?
a) Palpacin precordial.
b) Pulso carotideo.
c) Presin venosa yugular.
d) Tensin arterial.
e) Presin venosa.
109.Es el mecanismo fisiopatolgico por medio del cual el hipercortisolismo induce hipertensin arterial:
a) Aumento de volumen circulante por la retencin de Na.
b) Efecto directo del cortisol sobre receptores adrenrgicos.
c) El hipercortisolismo no causa hipertensin.
d) Por el potente efecto mineralocorticoide del cortisol.
e) Aumento del volumen circulante por la fuga de Na.
110.A qu se refiere el efecto Somogyi?
a) Hiperglucemia matutina en respuesta a hipoglucemia en la madrugada, la cual causa liberacin de hormonas contrarreguladoras.
b) Hiperglucemia matutina causada por efectividad reducida de la insulina en ese momento.
c) Hipoglucemia causada por exceso en la secrecin de cortisol.
d) Normoglucemia por buen control metablico del paciente diabtico.
e) Hipoglucemia causada por reduccin en la secrecin de estriol.
111.Cul de las siguientes caractersticas puede diferenciar de mejor manera a la enfermedad de Crohn de la colitis ulcerativa?
a) Abscesos de las criptas.
b) Granulomas no caseificantes.
c) Evidencia de involucro colnico obtenida mediante endoscopia.
d) Hemorragia rectal.
e) Ulceras orales.
112.En la biopsia de un varn con una alteracin de las pruebas hepticas aparece una esteatosis con necrosis hepatocelular en reas
centrolobulillares, fibrosis periportal, un infiltrado inflamatorio de PMN (polimorfonuclerares), inclusiones hialinas citoplasmticas (hialina
de Mallory) y mitocondrias gigantes con espacios portales normales. Diagnostico mas probable:
a) Hepatitis alcohlica aguda.
b) Hepatitis vrica aguda.
c) Hepatitis vrica crnica.
d) Cirrosis biliar primaria.
e) Hepatitis crnica autoinmune.
113.Ante una cifra de 690 mg/dl de triglicridos que tras dieta hipolipemiante no responde, qu frmaco administrara en primer lugar?
a) Pravastatina.
b) Colestipol.
c) Fenoterol.
d) Probucol.
e) Gemfibrozilo.
114.Un paciente afectado de Parkinson idioptico NO tendr uno de los siguientes sntomas:
a) Hiperreflexia.
b) Aumento de la saliva.
c) Escritura en miniatura.
d) Movimientos oculares anormales.
e) Seborrea.
REPASO Nro. 1 Pag. 15 USAMEDIC 2017
115.La artritis monoarticular septica se asocia a una infeccion debido a una de las siguientes:
a) Linfogranuloma venreo.
b) Chancroide.
c) Granuloma inguinal.
d) Neisseria.
e) Sfilis.
116.El rgano mas daado por la panarteritis nodosa es:
a) El cerebro.
b) El pulmn.
c) El bazo.
d) El rin.
e) El ojo.
117.Paciente presenta cuadro diarreico por alteracin en su motilidad intestinal. Qu tipo de diarrea NO es compatible con esta etiopatogenia?
a) Esclerosis sistema progresiva.
b) Hipertiroidismo.
c) Sndrome de Dumping.
d) Diabetes.
e) Vipoma.
118.Un paciente que presenta en el sedimento de orina microhematuria, proteinuria y cilindros hemticos, cul de los siguientes cuadros
patolgicos padece?
a) Lesin glomerular.
b) Lesin tbulo-intersticial.
c) Obstruccin de la va urinaria.
d) Infeccin renal.
e) Neoplasia renal.
119.Ante un paciente con intensa azotemia, anemia, infiltrados alveolares, hipoxemia ligera y con una capacidad de transferencia de
monxido de carbono aumentada, podemos diagnosticar con mayor probabilidad:
a) Sndrome de Goodpasture.
b) Enfisema pulmonar.
c) Fibrosis intersticial.
d) Edema pulmonar.
e) Embolia pulmonar.
120.Un adulto, en un examen rutinario de la empresa, muestra en la radiologia de trax unas adenopatas hiliares. En la exploracin fisica
se descubre un eritema nodoso y una uvetis anterior. Cul es la patologa mas probable?
a) Tuberculosis.
b) Cncer broncognico.
c) Linfoma.
d) Sarcoidosis.
e) Brucelosis.
121.Despus de ser sometido a una reseccin transuretral de la prstata, un hombre de 65 aos experimenta sangrado excesivo atribuido
a una fibrinlisis. Qu es apropiado administrar?
a) Heparina.
b) Warfarina.
c) Expansores del volumen y crioprecipitado.
d) cido aminocaproico.
e) Plasma fresco congelado y vitamina K.
122.Cul es el microorganismo causal ms frecuente de la artritis sptica del adulto?
a) Staphylococcus aureus.
b) Escherichia coli.
c) Streptococcus pyogenes.
d) Neisseria gonorrhoeae.
e) Pseudomona aeruginosa.
REPASO Nro. 1 Pag. 16 USAMEDIC 2017
123.La exposicin permanente al cobalto puede producir:
a) Cncer de colon.
b) Colelitiasis.
c) Pancreatitis.
d) Miocardiopatias.
e) Cncer de prstata.
124.El virus HTLV-1 en los adultos se asocia a:
a) Linfoma de Burkitt.
b) Enfermedad de Hodgkin.
c) Leucemia de clulas T.
d) Carcinoma nasofarngeo.
e) SIDA.
125.Ante la sospecha de un RGE, cul considera que es la prueba diagnostica de eleccin por su mayor sensibilidad?
a) Radiologa baritada.
b) Endoscopa.
c) Radiologa simple en bipedestacin.
d) pHmetra de 24 horas.
e) Manometra.
126.Cul es el tipo de hipertensin portal en el Sndrome de Budd Chiari?
a) Post heptica.
b) Intra heptica.
c) Presinusoidal.
d) Sinusoidal.
e) Extra heptica.
127.Cul de las siguientes pruebas hace el diagnostico de Haemophilus ducreyi chancroide?
a) Medio de cultivo de Thayer Martin.
b) Tincin de Gram del exudado.
c) Examen en campo oscuro.
d) Estudio en fresco.
e) Prueba serolgica.
128.Mujer de 40 aos, con diarrea y vmitos persistentes hace 2 das, luego oligoanuria y elevacin de creatinina de 3 mg/dL. La excrecin
fraccionada de sodio es menor de 1% y la relacin BUN/creatinina srica es mayor de 20. Cul es la causa de la lesin renal aguda?
a) Pre-renal.
b) Necrosis tubular.
c) Nefritis intersticial.
d) Glomrulonefritis.
e) Uropata obstructiva.
129.La infeccin cutnea conocida como Ectima es producida por:
a) Estreptococo.
b) Pseudomona.
c) Poxvirus.
d) Pityrosporum.
e) Trichophyton.
130.Un varn de 25 aos de edad diagnosticado antes de intolerancia al gluten, inicia de forma brusca con hematuria y proteinuria. Biopsia
renal: lesin proliferativa segmentaria y focal de clulas mesangiales, clulas epiteliales y endoteliales. Diagnostico mas probable:
a) Prpura de Schonlein-Henoch.
b) Glomerulonefritis aguda.
c) Sndrome de Alport.
d) Glomerulopata de cambios mnimos.
e) Nefropatia por IgA.
131.Varn de 75 aos fuma 1 cajetilla diaria desde hace 50 aos. Presenta tos, dolor de espalda, anorexia y astenia de 2 meses de evolucin.
En sangre: Na 124 meq/l, K 3,7 meq/L, osmolaridad 260 meq/l. En orina: Na 75 meq/l, osmolaridad 800 meq/l. Diagnostico mas probable:
a) Carcinoma de clulas pequeas.
b) Carcinoma de prstata.
c) Enfermedad de Hodgkin.
d) Tuberculosis.
e) Mesotelioma.
REPASO Nro. 1 Pag. 17 USAMEDIC 2017
132.Un obrero de una cantera de granito presenta una disnea de esfuerzo, tos, dolor torcico y hemoptisis. Rx de trax: adenopatas
hiliares calcificadas en cscara de huevo. Diagnostico mas probable:
a) Asbestosis.
b) Silicosis.
c) Beriliosis.
d) Caolinosis.
e) Carcinoma broncognico.
133.Un varn presenta una anemia macroctica, una medula osea megaloblstica, recuento reticulocitario disminuido e hipersegmentacion
de polimorfonucleares, con aclorhidria gstrica. Diagnostico mas probable:
a) Anemia sideroblstica.
b) Dficit de vitamina B12.
c) Anemia perniciosa.
d) Anemia crnica.
e) Anemia ferropnica.
134.Una de las siguientes situaciones es mas tpica del enfisema que de la bronquitis crnica:
a) Edad en el momento del diagnstico en torno a los 50 aos.
b) Disnea leve.
c) Infecciones pulmonares frecuentes.
d) Corazn agrandado.
e) Aspecto de soplador sonrosado.
135.En la terapia de combinacin de la tuberculosis pulmonar, el antimicrobiano mas activo en la primera fase es:
a) Estreptomicina.
b) Etambutol.
c) Isoniazida.
d) Rifampicina.
e) Pirazinamida.
136.Una de las siguientes es la causa mas frecuente de infeccin urinaria en mujeres sexualmente activas, tras Escherichia coli:
a) Proteus mirabillis.
b) Especies de Klebsiella.
c) Anaerobios.
d) Staphylococcus saprohyticus.
e) Neisseria gonorrhoeae.
137.Una de las siguientes NO es mas frecuente en el Crohn que en la colitis ulcerosa:
a) Presencia de granulomas no caseificantes en la biopsia.
b) Afectacin rectal.
c) Aparicin de fstulas perianales.
d) Enfermedad segmentaria.
e) Afectacin transmural.
138.Cul de los siguientes signos o sntomas aparece raramente en la esclerosis mltiple?
a) Sndrome piramidal.
b) Hemianopsia homonina.
c) Neuritis ptica.
d) Ataxia y temblor cerebeloso.
e) Diplopa.
139.La caracterstica mas especifica del sndrome esquizofrnico es:
a) La existencia de trastornos de la percepcin.
b) La incapacidad para distinguir entre la realidad interior y la exterior.
c) El marcado simbolismo que expresa su conducta.
d) La hipersensibilidad.
e) La incongruencia del pensamiento.
REPASO Nro. 1 Pag. 18 USAMEDIC 2017
140.Un paciente varn de 30 aos de edad con antecedentes familiares de dos hermanos jvenes fallecidos de forma sbita. En la
ecocardiografa se aprecia un engrosamiento del tabique interventricular con movimiento anterior sistlico de la vlvula mitral. Ante este
cuadro debemos pensar en:
a) Estenosis mitral.
b) Estenosis aortica.
c) Insuficiencia aortica.
d) Miocardiopatia hipertrfica obstructiva.
e) Miocardiopatia dilatada.
141.Una mujer de 50 aos presenta un cuadro de dolor torcico retroesternal y precordial izquierdo, que aumenta con la inspiracin y la
tos; tambin refiere irradiacin a la espalda y brazo izquierdo, as como disnea de grandes esfuerzos. El electrocardiograma muestra un
supradesnivelamiento del segmento ST de concavidad superior, que abarca casi todas las derivaciones: Rx trax normal. Con estos datos,
usted tiene claro el diagnostico de presuncin:
a) Angina inestable.
b) Tromboembolismo pulmonar.
c) Diseccin aortica.
d) Pericarditis aguda.
e) Infarto agudo de miocardio antero-lateral extenso
142.Paciente de 68 aos que acude a un centro de urgencias con cuadro de fibrilacin auricular de inicio reciente (duracin inferior a 48
h.). Indique la actitud correcta a seguir:
a) Cardioversin farmacolgica o elctrica.
b) Anticoagulacin durante una semana y posterior cardioversin.
c) Anticoagulacin al menos dos semanas y posterior cardioversin.
d) Instaurar el tratamiento anticoagulante por 3 semanas antes de la cardioversin y 4 semanas despus de ella.
e) No revertir.
143.Una paciente presenta un hirsutismo con niveles sricos muy altos de DHEAS (dehidroepiandrosterona), que disminuyen tras un
tratamiento con dexametasona; el diagnstico mas probable es:
a) Hirsutismo idioptico
b) Hirsutismo de origen ovrico
c) Hirsutismo de origen suprarrenal
d) Hirsutismo hipofisario
e) Hirsutismo iatrgeno
144.Qu tcnica rpida sera mas fiable para el diagnstico rpido de una meningitis?
a) Deteccin de anticuerpos antimembrana bacteriana.
b) Cultivo de LCR.
c) Examen en fresco del LCR.
d) Gram del LCR.
e) Sondas de cidos nucleicos.
145.Un paciente llega con fiebre en picos, sudoracin nocturna y estreimiento. Esplenomegalia y osteomielitis en la regin lumbosacra.
Diagnostico mas probable:
a) Linfoma de Hodgkin.
b) Linfoma no Hodgkin.
c) Brucelosis.
d) Toxoplasmosis.
e) Yersiniosis.
146.El fenmeno fundamental de la psoriasis es:
a) Aumento marcado de la melanogenesis.
b) Aumento de la queratinizacin.
c) Hipertrofia de las glndulas sudorparas.
d) Atrofia del tejido celular subcutneo.
e) Distrofia del tejido superficial.
147.Paciente con prdida de agudeza visual y miodesopsias. En el examen de fondo de ojo se halla vitritis, infiltrados retinianos y coroideos,
hemorragias y exudados. Como nico antecedente un ciclo largo de corticoides, sin hbitos txicos. Agente etiolgico mas frecuente:
a) Cndida.
b) Citomegalovirus.
c) Toxoplasma.
d) Tuberculosis.
e) Salmonelosis.
REPASO Nro. 1 Pag. 19 USAMEDIC 2017
148.Se ha demostrado que uno de los siguientes antihipertensivos tiene un efecto especfico sobre la nefropata por diabetes mellitus 1,
independientemente de su efecto sobre la tensin arterial:
a) Antagonistas del calcio.
b) Diurticos tiazdicos.
c) Diurticos de asa.
d) Betabloqueantes.
e) IECAs.
149.Un varn de 35 aos sufre un episodio recortado de alteracin de la conducta, con desconexin del medio, precedido de olor a
plstico quemado. El diagnstico mas probable es:
a) Trastorno de personalidad.
b) Cacosmia.
c) Crisis parcial compleja del lbulo temporal.
d) Crisis parcial compleja del lbulo parietal.
e) Crisis parcial secundariamente generalizada.
150.Ante una paciente de mediana edad que presenta prurito, con ictericia, esteatorrea, aumento srico de la fosfatasa alcalina y de la IgM,
y presencia de anticuerpos antimitocondriales, el diagnstico ms probable es
a) Colestasis recurrente benigna.
b) Cirrosis biliar primaria.
c) Colangitis esclerosante primaria.
d) Atresia biliar extraheptica.
e) Colestasis crnica juvenil.
151.La obtencin en un LCR con predominio de clulas mononucleadas es compatible con
a) Meningitis vrica.
b) Meningitis mictica.
c) Meningitis tuberculosa.
d) Meningitis bacteriana parcialmente tratada.
e) Todas las anteriores.
152.Un hombre de 54 aos, con cirrosis heptica por virus C e hipertensin portal conocidas, presenta un episodio de hematemesis con
repercusin hemodinmica. Se estabiliza con reposicin enrgica de la volemia, y es trasladado a la Unidad de Cuidados Intensivos,
donde se le transfunden 3 concentrados de hemates, y se instaura una perfusin endovenosa de somatostatina. Cul de las siguientes
opciones intervencionistas sera de eleccin en las siguientes 24 horas?
a) Endoscopia con escleroterapia mediante inyeccin de adrenalina.
b) Taponamiento con sonda-baln de Sengstaken-Blakemore.
c) Endoscopia con ligadura mediante bandas elsticas.
d) Embolizacin percutnea con coils de las vrices esofgicas.
e) Derivacin portosistmica intraheptica percutnea (por va transyugular).
153.El tratamiento de primera lnea de un paciente de 65 aos de leucemia mieloide crnica en primera fase crnica debe basarse en:
a) Quimioterapia intensiva hasta alcanzar la remisin completa.
b) Hidroxiurea oral para mantener valores leucocitarios normales.
c) Imatinib mesilato de forma indefinida.
d) Interfern alfa hasta mxima respuesta citogentica.
e) Trasplante alognico de progenitores hematopoyticos.
154.La Policitemia vera NO se asocia con:
a) Leucocitosis.
b) Trombocitosis.
c) Niveles elevados de B12.
d) Niveles elevados de eritropoyetina.
e) Esplenomegalia.

AREA DE PEDIATRIA

155.Se considera un signo de alarma del desarrollo psicomotor a los 18 meses de edad:
a) No come.
b) No hace una torre con 2 cubos.
c) No obedece una orden verbal sencilla.
d) No dice al menos 3 palabras aparte de mam y pap.
e) No se acerca un vaso a la boca.
REPASO Nro. 1 Pag. 20 USAMEDIC 2017
156.Un recin nacido con una frecuencia cardiaca de 80, mnima respuesta a estmulos con algn esfuerzo respiratorio, extremidades algo
flexionadas y cianticas tiene un Apgar de:
a) 6.
b) 5.
c) 4.
d) 7.
e) 3.
157.Qu parmetro en adolescentes mide la cantidad de grasa subcutnea, que se estima en un 50% de la grasa corporal?
a) Permetro del brazo.
b) ndice nutricional.
c) ndice de Quetelet.
d) Pliegues cutneos.
e) Perimetro de la mueca.
158.Un nio de 7 aos de edad presenta unas lesiones mculopapulosas no confluyentes. 1 semana antes de la erupcin, tena malestar,
cefalea y fiebre. Tales lesiones comenzaron en la frente y la cara. Ahora alcanzan el tronco y han empezado a desaparecer en la cara.
Adenopatas retroauriculares y suboccipitales. El cuadro es sugestivo de:
a) Sarampin
b) Exantema sbito
c) Eritema infeccioso
d) Sindrome del shock txico
e) Rubola
159.Cul de las siguientes vacunas tiene una aplicacin subcutnea?
a) DTP.
b) Polio Sabin.
c) Polio Salk.
d) Varicela.
e) Gripe.
160.La insuficiencia respiratoria aguda por atelectasia es caracterstica del:
a) Recin nacido.
b) Lactante.
c) Nio.
d) Adolescente.
e) Nio de 3 aos.
161.Es correcto afirmar respecto al recin nacido de bajo peso:
a) El reflejo de succin surge en el recin nacido al salir al exterior del tero independientemente de la edad.
b) La musculatura intestinal es abundante, hay tendencia al estreimiento.
c) Los capilares del RN de bajo peso son frgiles.
d) El aumento de protenas lleva a producir edemas.
e) Capacidad del estmago grande.
162.El reflejo en un neonato que consiste en una estimulacin paravertebral que provoca un movimiento del tronco se denomina:
a) Galant.
b) Bsqueda.
c) Puntos cardinales.
d) Babinski.
e) Marcha automtica.
163.Una zona de piel en el infante con cambio coloracin de hasta 1 cm de dimetro, limitada y plana, es una:
a) Mcula.
b) Ppula.
c) Vescula.
d) Ampolla.
e) Mancha.
164.En el EEG de un nio aparecieron puntas ondas a 3 Hz, qu enfermedad padece?
a) Epilepsia tipo gran mal.
b) Sndrome de West.
c) Mioclonas.
d) Epilepsia Jacksoniana.
e) Ausencias tpicas (pequeo mal).
REPASO Nro. 1 Pag. 21 USAMEDIC 2017
165.En un recin nacido de 32 semanas que presenta dificultad respiratoria. Cul es la posible causa?
a) Disminucin de surfactante producido por neumocito I.
b) Deficiencia de fibras elsticas por inmadurez.
c) Ausencia de lecitina producida por macrfagos.
d) Incremento de dipalmitoil fosfatidilcolina por neumocito I.
e) Ausencia de surfactante producido por neumocito II
166.En el recin nacido a trmino con asfixia, la infusin de glucosa que debe administrarse es..mg/kg/min.
a) 5.
b) 8.
c) 10.
d) 3.
e) 1.
167.En una nia recin nacida se advierte la existencia de dolor cuando se le mueven los brazos. Posiblemente es debido a un lesin por:
a) Parlisis de Erb-Duchenne.
b) Parlisis de Dejerine-Klumpke.
c) Hombro doloroso.
d) Luxacin acromioclavicular.
e) Fractura de clavcula.
168.Durante la reanimacin neonatal. Cules son los signos que deben evaluarse?
a) Frecuencia respiratoria y frecuencia cardiaca.
b) Actividad refleja y frecuencia cardiaca.
c) Esfuerzo respiratorio y frecuencia cardiaca.
d) Tono muscular y frecuencia respiratoria.
e) Tono muscular y actividad refleja.
169.Varn de 18 meses, esta bajo tratamiento con amoxicilina por 5 das a causa de una otitis media. Contina con fiebre y a la exploracin
fsica la membrana timpnica derecha est tensa con lquido purulento en el odo medio. Cul es el mejor antibitico a administrar?
a) Amoxicilina con cido clavulnico.
b) Dicloxacilina.
c) Cefalexina.
d) Eritromicina.
e) Penicilina.
170.Nio de 3 aos presenta diarrea crnica. Sin prueba alguna y teniendo en cuenta los datos epidemiolgicos, la etiologa mas frecuente es:
a) Fibrosis qustica.
b) Intolerancia a la lactosa.
c) Enfermedad de Crohn.
d) Diarrea inespecfica.
e) Intolerancia a la leche de vaca.
171.Para el tratamiento del cuadro convulsivo en un neonato debe usarse
a) Diazepam.
b) Alprazolam.
c) Fenobarbital.
d) Carbamazepina.
e) Hidralazina.
172.Un nio de 13 aos de edad presenta un dolor abdominal, al inicio epigstrico y mal definido y ahora se localiza en la fosa iliaca
derecha. El signo de Blumberg es positivo. Analtica: leucocitosis y aumento de la VSG. Cul de las siguientes actitudes es la correcta?
a) Alta y toma de indometacina.
b) Intervencin quirrgica urgente.
c) Acido acetilsalicilico y observacin.
d) Intervencin diferida.
e) Alta y omeprazol.
REPASO Nro. 1 Pag. 22 USAMEDIC 2017
173.Una nia de 7 aos de edad presenta problemas en la escuela y hogar. Ataques de ira y conducta rebelde. Bajo desempeo escolar en
comparacin con sus amigos y por tanto, la colocan en educacin especial. Las pruebas neuropsiquitricas confirman un coeficiente
intelectual debajo del promedio con calificacin de 66. Cul sindrome tiene la menor probabilidad de causar su retraso mental?
a) Sndrome de Down.
b) Sndrome de cromosoma X frgil.
c) Sndrome de alcoholismo fetal.
d) Sndrome de Turner.
e) Sndrome del maullido del gato.
174.La alternancia de estreimiento y diarrea en el lactante es tpico de:
a) Atresia de vas biliares.
b) Megacolon aganglinico.
c) El pncreas anular.
d) Enfermedad celiaca.
e) Malrotacin.
175.Recin nacido por cesrea por sufrimiento fetal con lquido meconial. Nace flcido, plido, con FC: 80 x; luego de efectuar las
primeras medidas, se debe realizar
a) Aspiracin.
b) Masaje cardiaco.
c) Ventilacin con oxgeno por mscara.
d) Ventilacin a presin positiva.
e) Pasar a UCIN.
176.La palpitacin de zonas de menor consistencia (craneotabes) a nivel parietal en el crneo de un neonato sugiere:
a) Sndrome de Crouzon
b) Hipotiroidismo
c) Puede aparecer en recin nacidos normales
d) Infeccin congnita por rubola
e) Raquitismo
177.En la insuficiencia cardiaca del lactante, uno de los siguientes signos o sntomas est siempre presente:
a) Esplenomegalia.
b) Ingurgitacin yugular.
c) Ascitis.
d) Hepatomegalia.
e) Reflujo hepatoyugular.
178.En pediatria, una de las siguientes anemias hemolticas cursa de manera aguda y ademas con hemoglobinuria:
a) Eliptocitosis hereditaria.
b) Dficit de piruvatoquinasa.
c) Anemia por anticuerpos calientes.
d) Esferocitosis hereditaria.
e) Dficit de glucosa 6 fosfato deshidrogenasa.
179.La causa mas frecuente de la hiperbilirrubinemia no conjugada es:
a) Ictericia neonatal fisiolgica.
b) Litiasis biliar.
c) Sndrome de Crigler-Najjar.
d) Sndrome de Gilbert.
e) Procesos neoplsicos.
180.Las perlas epiteliales de Epstein se localizan en el neonato en:
a) Raz nasal.
b) Trax.
c) Regin sacra.
d) Paladar.
e) Nuca.
REPASO Nro. 1 Pag. 23 USAMEDIC 2017
181.Un nio de 7 aos de edad es diagnosticado mediante las correspondientes pruebas de estmulo de dficit completo de GH. En la
radiologa convencional se ven calcificaciones intracraneales de localizacin supraselar. Lo mas probable es que tenga:
a) Toxoplasmosis neonatal.
b) Hipoparatiroidismo.
c) Craneofaringioma.
d) Pinealoma.
e) Infeccin por citomegalovirus.
182.Lactante de 8 meses inicia su enfermedad con fiebre de 39-40C desde hace 7 das. Irritabilidad y vmitos desde hace un da.
Convulsiones tnico clnicas generalizadas en emergencia. Al examen: Fontanela abombada, hipertona generalizada y reflejos
aumentados. Cul es el diagnstico ms probable?
a) Hipertensin endocraneana.
b) Tumor cerebral.
c) Hemorragia subaracnoidea.
d) Meningoencefalitis bacteriana.
e) Meningitis tubeculosa.
183.Cul de los siguientes cereales se puede introducir en la dieta de un nio con intolerancia al gluten?
a) Centeno.
b) Maz.
c) Avena.
d) Trigo.
e) Cebada.
184.Un lactante de 12 meses de edad, antes sana, desarrolla una enfermedad con fiebre de 3 das seguida por normotermia y un exantema
maculopapular en tronco. No hay otros signos o sntomas. Diagnostico mas probable:
a) Eritema infeccioso (quinta enfermedad).
b) Enfermedad de Kawasaki.
c) Exantema sbito.
d) Sarampin.
e) Escarlatina.
185.Indique el tratamiento de eleccin para la giardiasis intestinal en nios:
a) Pamoato de pirantel.
b) Metronidazol.
c) Mebendazol.
d) Cotrimoxazol.
e) Levamisol.
186.Una nia de 6 aos de edad presenta febrcula, cefalea y congestin nasal. Rubor facial y erupcin cutnea reticular en el tronco y
superficie extensora de los antebrazos y piernas. No presenta afeccin en las palmas y plantas. Su madre tambin ha estado con febrcula,
rigidez articular y dolor. Diagnostico mas probable:
a) Rubola.
b) Sarampin.
c) Escarlatina.
d) Exantema sbito.
e) Eritema infeccioso.
187.Cuntos das se administra el tratamiento en una meningitis bacteriana neonatal?
a) 10.
b) 18.
c) 28.
d) 7.
e) 21.
188.Se considera normal que un nio sea capaz de construir frases sencillas e inteligibles a partir de los:
a) 12 meses.
b) 18 meses.
c) 24 meses.
d) 3 aos.
e) 5 aos.
REPASO Nro. 1 Pag. 24 USAMEDIC 2017
189.Nio de 5 meses presenta agravamiento brusco de proceso catarral que arrastraba desde hace 4 das, ahora tiene T 40 C, bradipnea
inspiratoria, tiraje y disnea. Cuadro no acompaado de tos ni de afona, aunque s se nota dificultad para tragar. Diagnostico mas probable:
a) Crup viral.
b) Edema de glotis.
c) Laringitis estenosante aguda.
d) Adenoiditis bacteriana.
e) Epiglotitis aguda.
190.Todas las siguientes son caractersticas del marasmo, EXCEPTO:
a) Prdida de panculo adiposo
b) Prdida de peso mayor del 40%
c) Retraso de talla
d) Edema
e) Niveles de IgA disminuidos
191.Un neonato presenta taquipnea; la radiografia del trax muestra imgenes vasculares pulmonares prominentes, diafragma aplanado y
lquido en las cisuras. No presenta hipoxemia, hipercapnia ni acidosis. Diagnostico mas probable:
a) Enfermedad de la membrana hialina
b) Aspiracin de meconio
c) Neumomediastino
d) Taquipnea transitoria del recin nacido
e) Sndrome de Wilson-Mikity
192.Si un lactante comienza con diarrea hacia los 18 meses de vida prolongndose hasta los 30 meses, con heces pastosas en las que se
reconocen alimentos, apetito conservado y curva de crecimiento apenas afectada, lo mas probable es que padezca:
a) Una enfermedad celiaca.
b) Una fibrosis qustica.
c) Una intolerancia a protenas dietticas.
d) Una alergia a cualquiera de los alimentos recientemente introducidos.
e) Una diarrea crnica inespecfica.
193.Nio de 14 aos de edad presenta aumento del volumen mamario en el lado izquierdo. Niega dolor, secrecin o uso de frmacos. No
recibe medicamentos. Su escala de maduracin sexual (Tanner) es etapa II para genitales y vello pbico. Cul es su conducta inicial?
a) Resonancia magntica nuclear de la cabeza.
b) Deteccin de marihuana en orina.
c) Anlisis cromosmico.
d) Informar que es un cambio normal.
e) Ecografa del abdomen
194.Un varn en edad peditrica, con proteinuria renal muy selectiva y una imagen renal al microscopio ptico totalmente normal, nos
hara sospechar:
a) Esclerosis focal.
b) Sndrome de Goodpasture.
c) Glomerulonefritis de cambios mnimos.
d) Glomerulonefritis mesangial.
e) Glomerulonefritis membranosa.
195.Nia de 18 meses llega por haberle detectado una masa abdominal. Se palpa una masa dura a nivel de la fosa renal derecha. Mnimo
exoftalmos derecho con un hematoma lineal en el prpado superior derecho. Entre los exmenes complementarios destaca la existencia
de altos niveles en sangre de ferritina y de catecolaminas en orina. Qu tipo de entidad clnica sospecha?
a) Neuroblastoma.
b) Nefroblastoma.
c) Nefroma mesoblstico.
d) Hipernefrona.
e) Teratoma.
196.Seale cual de las siguientes manifestaciones NO se encuentra en el sindrome hemoltico-uremico del nio:
a) Trombocitopenia.
b) Hipertensin arterial.
c) Convulsiones
d) Insuficiencia heptica.
e) Insuficiencia renal aguda.
REPASO Nro. 1 Pag. 25 USAMEDIC 2017
197.En un nio de corta edad que presenta fiebre con dolor de garganta y cefalea, y en la exploracin se pone de manifiesto un exantema
vesculoso en la mucosa bucal y paladar, con manos, pies y rea del paal tambin afectados, seale el agente causal mas probable:
a) Virus Coxackie B16.
b) Parvovirus B19.
c) Virus ECHO 7.
d) Herpes virus tipo 6.
e) Virus Coxackie A16.
198.Nio de 2 aos de edad que comienza de manera brusca, mientras duerme, con dificultad respiratoria intensa, estridor inspiratorio y
tos ronca. Cul es el diagnostico mas probable?
a) Crisis asmtica.
b) Laringitis estridulosa.
c) Bronquiolitis.
d) Sndrome espirativo.
e) Laringitis subgltica.
199.Ante un lactante de 10 meses de edad con una obstruccin aguda y total de las vas areas superiores por aspiracin de cuerpo
extrao, la actitud mas aconsejable es:
a) Intentar la extraccin manual con el nio en decbito prono y la cabeza colocada a nivel inferior con respecto al tronco.
b) Compresin abdominal brusca, con reanimador colocado detrs del paciente y ste en posicin vertical (maniobra de Heimlich).
c) Empleo repetitivo de golpes en la espalda y compresiones torcicas con el paciente en Trendelemburg.
d) Compresin abdominal brusca con el paciente en decbito supino y cabeza por debajo del tronco.
e) Broncoscopia urgente.
200.Nia de 8 aos, pequea para la edad; en trminos generales sana. Talla corta, pues su estatura corresponde a la de una nia de 4
aos y 4 meses. Se notan ciertas anomalas en su aspecto general compatibles con el sindrome de Turner. Usted se acuerda que una
cardiopata congnita es comn en este sindrome en particular. Cardiopata congnita mas probable:
a) Estenosis artica supravalvular.
b) Defectos auriculoventriculares.
c) Coartacin de la aorta.
d) Estenosis de vlvula pulmonar.
e) Prolapso de vlvula mitral.
201.El principal objetivo de la presin positiva continua de la via aerea en el tratamiento de infantes con sndrome de dificultad respiratoria es:
a) Evitar la infeccin.
b) Evitar un neumotorax.
c) Mejorar el gasto cardiaco.
d) Aumentar la PO2 arterial.
e) Disminuir la PCO2 arterial.
202.Cul opcin debe incluir el tratamiento, despus de la rehidratacin y la correccin del desequilibrio electroltico del caso anterior?
a) Succin nasogstrica, rehidratacin y observacin.
b) Medicamentos anticolinrgicos.
c) Laxantes.
d) Ciruga de urgencia y reseccin intestinal.
e) Apendicectoma.
203.Un varn de 16 aos de edad mide 1.70 m y pesa 168 Kg. No se encuentran causas mdicas para su obesidad esencial. Cul de las
siguientes anomalas puede l tambin padecer?
a) Riesgo de muerte normal.
b) Hipotiroidismo.
c) Valores bajos de Pco2.
d) Hipertrigliceridemia.
e) Hiperadrenocorticalismo.
204.Cul de estos datos analticos espera encontrar en un lactante que presenta un raquitismo vitamino-D carencial?
a) Calcemia baja y fosforemia normal.
b) Calcemia alta y fosforemia baja.
c) Calcemia normal y fosforemia baja.
d) Calciuria elevada, fosfaturia elevada y fosfatasas alcalinas elevadas.
e) Calcemia baja y fosforemia alta.
REPASO Nro. 1 Pag. 26 USAMEDIC 2017
205.En los nios, el sndrome de Gilbert es de origen:
a) Virisico.
b) Idioptico.
c) Txico.
d) Hereditario.
e) Medicamentoso.
206.A un infante se le diagnostica con un trastorno referido en las alternativas. Cul de estos presenta el riesgo de mayor recurrencia
para los futuros hermanos de este paciente?
a) La enfermedad de Hirschsprung.
b) Fibrosis quistica.
c) Defecto septal ventricular.
d) Trisomia 21.
e) Trisomia 13.
207.En un nio sano de 3 meses, el valor de la hemoglobina es:
a) 13-17 g/dL.
b) 10-14 g/dL.
c) 15 g/dL.
d) < 9 g/dL.
e) Prcticamente igual que al nacer.
208.Nio de 1 ao de edad, que en el momento del nacimiento presentaba mltiples lesiones ampollosas en la espalda. Con el tiempo,
estas lesiones se convirtieron en ppulas de tamao variable hiperpigmentadas, de color marrn oscuro y bordes mal delimitados. Al
frotar estas lesiones, se produce eritema y edema (signo de Darier). Ahora no tiene ninguna duda de que se trata de:
a) Urticaria pigmentosa.
b) Alergia medicamentosa.
c) Telangiectasia macularis eruptiva perstans.
d) Urticaria crnica.
e) Imptigo ampolloso.
209.Nia de 6 aos presenta regular estado general y fiebre elevada. Exploracion fsica: amgdalas hipertrficas, con extensos exudados
blanquecinos, adenopatas submaxilares, laterocervicales e inguinales y hepatoesplenomegalia. Hemograma: leucocitosis con linfocitosis
(25% de linfocitos estimulados). Cul es su diagnstico?
a) Amigdalitis estreptoccica.
b) Leucemia linfoblstica aguda.
c) Amigdalitis vrica.
d) Mononucleosis infecciosa.
e) Difteria.
210.Epidemiolgicamente, con qu infeccin asociara mas frecuentemente al sndrome de Reye?
a) Virus sincitial respiratorio.
b) Estreptococo beta-hemoltico.
c) Virus de la influenza B.
d) Adenovirus.
e) Staphylococcus aureus.
211.El Beriberi es un cuadro clnico producido por:
a) Dficit de vitamina B6.
b) Exceso de vitamina A.
c) Dficit de vitamina B1.
d) Exceso de vitamina B2.
e) Dficit de cido ascrbico.
212.Un nio presenta diarrea crnica y sntomas de reaccin alrgica. Si sospecha que pueda ser una alergia a las protenas de la leche de
vaca, cul de estas es la prueba mas valiosa para el diagnostico?
a) Test cutneos.
b) Determinacin de anticuerpos aglutinantes.
c) Pruebas de eliminacin y provocacin repetidas.
d) Deteccin de inmunorreaginas IgE por radioinmunoensayo (RAST).
e) Activacin del complemento C3 in vitro.
REPASO Nro. 1 Pag. 27 USAMEDIC 2017
213.Un nio de 18 meses presenta sindrome febril de varios das, adenomegalias cervicales, inyeccin conjuntival, fisuraciones labiales,
exantema puntiforme de color rojo claro y artritis de los dedos de las manos y carpos. Diagnostico mas probable:
a) Artritis reumatoide
b) Escarlatina
c) Rubola
d) Enfermedad de Kawasaki
e) Enfermedad del suero
214.Nia de 5 meses; al presente lleva 2 das con rinorrea y 1 da de fiebre y tos cada vez mas intensa. Antes de este inicio estaba bien y
no tena sntomas similares. Exploracion fsica: estridor inspiratorio. Cul diagnostico debe considerar en el diagnostico diferencial?
a) Bronquiolitis aguda.
b) Laringotraqueobronquitis aguda.
c) Cuerpo extrao en vas respiratorias superiores.
d) Laringomalacia.
e) Absceso periamigdalino.
215.En un nio de 18 meses con gastroenteritis aguda, cul es la etiologa infecciosa mas frecuente de las siguientes?
a) Arbovirus.
b) Norwalk.
c) Rotavirus.
d) Artrovirus.
e) Adenovirus.
216.En un nio de 5 aos en anasarca de 20 das de evolucin y con analtica de proteinuria muy selectiva de 6 g/24 h, sedimento urinario
normal, hipoalbuminemia e hiperlipidemia, acompaando a una creatinina plasmtica de 0.6 mg/dL, el diagnstico mas probable ser:
a) Hialinosis focal.
b) Nefropata de cambios mnimos.
c) Glomerulonefritis aguda postestreptoccica.
d) Nefropata membranosa.
e) Nefropata de cambios mnimos con insuficiencia renal evolucionada.
217.Un nio de 6 meses con desarrollo psicomotor normal y sin antecedentes de inters, presenta unos movimientos mioclnicos flexores
en cuello, tronco y extremidades en forma de episodios agrupados. La madre lo lleva al pediatra, y ste le realiza un EEG que es normal
intercrisis. Otro EEG realizado durante una crisis result tambin normal. Cul es su diagnstico?
a) Sndrome de West.
b) Encefalopata mioclnica temprana.
c) Enfermedad de Lafora.
d) Mioclono benigno del lactante.
e) Sndrome de Lennox-Gastaut.
218.En todos los casos, usted considera necesario e importante remitir un paciente al oftalmlogo, EXCEPTO:
a) Nio de 8 aos con disminucin de la agudeza visual.
b) Nio de 3 aos con queratitis herptica.
c) Nio de 2 meses con estrabismo intermitente.
d) Nio de 5 aos con midriasis traumtica.
e) Nio de 3 meses con alteracin del reflejo rojo del ojo.
219.El tipo de anemia mas prevalente en la infancia es:
a) Beta-talasemia.
b) Anemia de Blackfan-Diamond.
c) Anemia megaloblstica.
d) Anemia ferropnica.
e) Anemia autoinmune.
220.Seale la respuesta incorrecta respecto a la maniobra de Ortolani:
a) El paciente tiene que estar en posicin de Fowler.
b) Hay que flexionarle 90 las caderas y rodillas.
c) Se le pinza el muslo entre el pulgar por la cara interna.
d) El 2 y 3 dedo apretarn el relieve del trocnter.
e) Se abduce el muslo y se estira con los dedos 2 y 3 apretando hacia el borde acetabular.
REPASO Nro. 1 Pag. 28 USAMEDIC 2017
221.La hepatomegalia en los nios es una manifestacin caracterstica de:
a) Toxocara canis.
b) Ascaris lumbricoides.
c) Necator americanus.
d) Trichuris trichiura.
e) Enterobius vermicularis.
222.Un varn de 3 aos de edad sufre ahogamiento y tos mientras come man. Tose con frecuencia, tiene estridor inspiratorio, tirajes
intercostales y retraccin supraesternal leves. El tratamiento inicial incluye:
a) Golpes en la espalda.
b) Compresin abdominal.
c) Barrido de la hipofaringe a ciegas con los dedos.
d) Permitir que el nio expulse cuerpo extrao por medio de la tos.
e) Traqueostoma de urgencia.
223.La forma de hemorragia intracraneal mas frecuente en el neonato prematuro es la:
a) Subdural
b) Infratentorial
c) Intraventricular
d) Subaracnoidea
e) Epidural
224.La fase pre-icterica de la infeccin por virus de la hepatitis B se puede asociar con:
a) Neumonia.
b) Linfadenopatia generalizada.
c) Fiebre prolongada.
d) Acrodermatitis papular.
e) Encefalopatia.
225.La mayora de casos de insuficiencia pancreatica en la niez son causados por:
a) Hipertiroidismo.
b) Atresia biliar.
c) Carcinoma.
d) Ausencia congenita del pancreas.
e) Fibrosis quistica.
226.Respecto al caput succedaneum del neonato slo una es verdadera de las siguientes:
a) El caput se extiende sobre suturas y fontanelas
b) La coleccin sangunea se sita debajo del periostio
c) Se aprecia mejor al 2- 3 da de vida
d) En general es debido a una pequea fractura
e) Las fontanelas no se cierran nunca
227.El sndrome del neonato caracterizado por cianosis, hipotensin y muerte, est producido por la toxicidad dosis-dependiente de uno
de los siguientes antibiticos:
a) Tetraciclinas.
b) Estreptomicina.
c) Cefalosporinas.
d) Cloranfenicol.
e) Aminoglucsidos.
228.Indique, de entre las siguientes, una cardiopata congnita NO ciangena:
a) Tetraloga de Fallot.
b) Atresia tricuspdea.
c) Transposicin de los grandes vasos.
d) Coartacion aortica.
e) Tronco arterioso.
229.La principal causa de la conjuntivitis en el neonato es:
a) Infeccin por chlamydias.
b) Infeccin por Staphylococcus aureus.
c) Conjuntivitis herptica.
d) Conjuntivitis qumica por nitrato de plata o por eritromicina.
e) Infeccin gonoccica.
REPASO Nro. 1 Pag. 29 USAMEDIC 2017
230.En el aceite de pescado predomina un acido graso poliinsaturado esencial (nuestro cuerpo no lo genera) muy beneficioso para la
salud peditrica:
a) cido araquidnico (AA).
b) cido esterico.
c) cido eicosapentanoico (EPA).
d) cido palmtico.
e) cido oleico.
231.Cuntos tipos de dolor abdominal recurrente se clasifican en Pediatria?
a) Dos.
b) Tres.
c) Cuatro.
d) Cinco.
e) Uno.
232.El test de Hirschberg, parte del estudio del estrabismo para descartarlo en oftalmologa peditrica, est indicado para nios:
a) Entre 6 meses y 4 aos.
b) Ms de 2 aos.
c) Ms de 4 aos.
d) A los 10 aos.
e) En la adolescencia.
233.En que situacin nos podemos encontrar una nia recin nacida con labios mayores genitales edematosos?
a) En partos prolongados.
b) Si ha habido sufrimiento fetal.
c) En nias de peso elevado.
d) Si el parto ha sido de nalgas.
e) En caso de cesrea.
234.Cul es el factor que se encuentra deficiente en los nios con hemofilia B?
a) VIII.
b) IX.
c) XI.
d) V.
e) VII.
235.Cul es el primer parmetro de laboratorio que se altera en la ferropenia?
a) Saturacin de transferrina.
b) Ferritina.
c) Concentracin de transferrina.
d) Reticulocitos.
e) VCM.
236.Las manifestaciones dermatolgicas reconocidas del sindrome del shock toxico estafilococico en nias adolescentes incluyen:
a) Petequias en las muecas y tobillos con propagacion al tronco.
b) Vesiculas en la cara y tronco.
c) Impetigo ampolloso localizado.
d) Eritema macular difuso que se descama en 1 a 2 semanas.
e) Erupcion morbiliforme en el tronco.
237.Nia de 12 aos con presencia de facies en esfinge con frente amplia, implantacin baja de orejas, epicantus, pterigium colli,
metacarpianos cortos, cubitus valgus y trax en coraza con mamilas separadas. Ante este cuadro sospecharemos:
a) Sndrome de Klinefelter.
b) Sndrome XYY.
c) Sndrome de Turner.
d) Sndrome de Noonan.
e) Sndrome de la triple X.
238.Un nio de 15 aos de edad tiene amigdalitis exudativa, adenitis cervical y esplenomegalia. Cul de los siguientes es la complicacin
ms comnmente hallada?
a) Fatiga cronica que dura ms de 6 meses.
b) Hemorragia.
c) Neumonia.
d) Encefalitis.
e) Obtruccion de las vias areas.
REPASO Nro. 1 Pag. 30 USAMEDIC 2017
239.Ante un nio con estatus convulsivo, el medicamento habitualmente utilizado para tratarlo es:
a) Carbamazepina.
b) Diazepam.
c) Fenobarbital.
d) cido valproico.
e) Hidantonas.
240.Qu agente etiolgico sospechara en una nia de 3 aos de edad con un cuadro de fiebre elevada, hiperemia faringoamigdalar y
escasas lesiones vesiculosas a nivel de los pilares amigdalares anteriores y paladar blando?
a) Estreptococo beta-hemoltico del grupo A.
b) Virus de Epstein-Barr.
c) Adenovirus.
d) Neumococo.
e) Virus Coxackie.
241.De las siguientes, cul puede ser una complicacin grave de la varicela?
a) Neumona.
b) Trombocitopenia.
c) Hepatitis.
d) Anemia.
e) Neuritis.
242.Un nio de 4 aos de edad esta jugando en el sotano; subitamente sube las escaleras gritando que le ha picado una araa, hay una
lesion roja en su cara. En las siguientes horas la lesin se vuelve ms grande, ms dolorosa y ms oscura hasta ser violacea. La
complicacion ms comn en este nio es:
a) Necrosis en el sitio de la picadura.
b) Insuficiencia renal.
c) Insuficiencia hepatica.
d) Calambres musculares y convulsiones.
e) Convulsiones.
243.Un nio de 15 aos de edad presenta un dolor unilateral en el rea frontotemporal con nuseas. Diagnostico mas probable:
a) Migraa.
b) Hemorragia subaracnoidea.
c) Hematoma subdural.
d) Cefalea histamnica.
e) Cefalea por sinusitis.
244.Los indicadores ms tempranos del sndrome de Cushing (exceso de glucocorticoide) en los nios son:
a) Ganancia de peso y cese del crecimiento
b) Cese del crecimiento y acne.
c) Acne e hipertensin.
d) Hipertensin y estras.
e) Acne y estras.
245.La edad pico de inicio del sndrome nefrotico en la niez asociada a la morfologa de cambios mnimos es:
a) Debajo de los 6 meses de edad.
b) Entre 12 y 18 meses de edad.
c) Entre 2 y 5 aos de edad.
d) Entre 5 y 10 aos de edad.
e) Entre 10 y 15 aos de edad.
246.De entre las cardiopatas congnitas se afirma que:
a) El signo radiolgico de sospecha es la cardiomegalia pasadas las 72 horas de vida.
b) Los signos clnicos en un neonato son cianosis, soplo precordial, taquipnea sin disnea y alteracin de los pulsos perifricos.
c) La atresia artica es la causa ms frecuente de insuficiencia cardiaca neonatal.
d) En la coartacin de la aorta lo ms frecuente es la disminucin o ausencia de los pulsos femorales.
e) La cardiopata congnita ciantica ms frecuente es la tetraloga de Fallot.
247.La hiperplasia suprarrenal congnita es causada mas frecuentemente por uno de los siguientes dficits enzimticos:
a) Dficit de 3 beta deshidrogenasa.
b) Dficit de 21 hidroxilasa.
c) Dficit de 20-22 desmolasa.
d) Dficit de 18 hidroxilasa.
e) Dficit de 11 hidroxilasa.
REPASO Nro. 1 Pag. 31 USAMEDIC 2017
248.Qu diagnostica si un lactante presenta atrializacin del ventrculo derecho, CIA tipo ostium secundum y sd de Wolf-Parkinson-
White?
a) Triloga de Fallot.
b) Tronco arterioso asociado a sndrome de DiGeorge.
c) Sndrome de Taussig-Bing.
d) Transposicin de los grandes vasos.
e) Enfermedad de Ebstein.
249.Cul de estas alteraciones NO encontrar en un nio afecto de sndrome de Down?
a) ngulo acetabular ms abierto de lo normal.
b) Mayor riesgo de hipertensin pulmonar.
c) Macroglosia.
d) Imagen radiolgica en doble burbuja.
e) Surco nico de flexin palmar.
250.Qu hallazgo seria ms sugerente del abuso infantil referido como "sindrome del bebe zarandeado"?
a) Equimosis sobre el area mastoidea.
b) Hemorragias retinales.
c) Equimosis y petequias sobre la parte alta de los brazos y tronco.
d) Equimosis circunferencial en las extremidades.
e) Dislocacin de la columna cervical.
251.La madre de una nia de 10 aos de edad informa de su hija que se moja constantemente, lo que requiere que use paales desde que
naci. Por otro lado, la nia es muy saludable y feliz, y adems le va bien en la escuela. Qu diagnstico sera el ms apropiado?
a) Ansiedad materna.
b) Urter ectpico con fstula hacia la vagina.
c) Incontinencia de esfuerzo.
d) Divertculos uretrales.
e) Fstula vesicovaginal.
252.Causa mas importante de diarrea grave con deshidratacin en los lactantes y nios menores de 3 aos de edad, sobre todo en los
pases en vas de desarrollo, pero tambin en los pases industrializados:
a) Salmonella
b) Rotavirus
c) Shigella
d) Escherichia coli
e) Adenovirus intestinales.
253.Una nia de 5 aos de edad presenta una fiebre elevada, un exantema morbiliforme confluente en todo el cuerpo desde hace 3 das,
adenopatas y esplenomegalia. El cuadro se inici hace 1 semana y comenz con una fiebre moderada, tos, inyeccin conjuntival y
afectacin del estado general. Diagnostico mas probable:
a) Sarampin
b) Exantema sbito
c) Varicela
d) Rubola
e) Escarlatina
254.Un nio de 13 aos de edad, de talla corta. Su tabla de crecimiento desde los 2 a 12 aos fue bien llevada. Los primeros 2 aos en P25.
Nio sano, de buen apetito y desarrollo escolar. Vive con sus padres y es hijo nico. La talla de sus padres en P50. Creci varios
centimetros luego de terminar la secundaria. Exploracin fsica normal y esta en el estadio 1 de Tanner. Causa mas probable de su talla:
a) Retraso constitucional.
b) Enanismo por privacin hormonal.
c) Talla corta familiar.
d) Deficiencia de hormona del crecimiento.
e) Hipotiroidismo.
255.Una de las siguientes es la mejor prueba para la valoracin del paciente:
a) Edad sea.
b) Estudios de imagen del crneo.
c) Estimulacin con hormona del crecimiento.
d) Pruebas de funcin tiroidea.
e) No se necesitan pruebas.
REPASO Nro. 1 Pag. 32 USAMEDIC 2017
256.En cuanto al cuadro de un nio de 4 aos de edad con neumonas frecuentes, diarrea crnica, y pobre crecimiento, seleccione uno de
los siguientes agentes:
a) Rochalimaea henselae.
b) Pseudomonas.
c) Rubivirus.
d) Herpesvirus humano 6.
e) Escherichia coli.
257.Los bronquios colapsados causados por falla del desarrollo del cartlago bronquial es indicativo de una de estas malformaciones congnitas:
a) Quistes bronquiales congnitos.
b) Enfisema neonatal congnito.
c) Fstula traqueoesofgica.
d) Enfermedad de la membrana hialina.
e) Hipoplasia pulmonar.
258.Un nio de 3 meses de edad presenta desde poco despus de su nacimiento una lesin vascular sobreelevada de superficie lisa y
crecimiento progresivo en la mejilla derecha. Diagnostico mas probable:
a) Angioma plano.
b) Hemangioma.
c) Granuloma piognico.
d) Lago venoso.
e) Angioqueratoma.
259.Ante un neonato con hepatoesplenomegalia, secrecin nasal purulenta y ampollas grandes en las palmas y plantas, pensara en una:
a) Infeccin congnita por varicela zoster
b) Infeccin por sfilis
c) Infeccin por CMV
d) Infeccin por toxoplasma
e) Infeccin herptica
260.Un neonato de 24 das de vida, desprendi el cordn umbilical a los 8 das y el ombligo no cicatriza. Presenta a su nivel una
tumoracin roja, prominente, circular y que secreta un contenido alcalino. A veces se escucha ruido de gases. Diagnostico mas probable:
a) Persistencia del conducto onfalomesenterico
b) Fstulo vesico-umbilical
c) Granuloma umbilical
d) Onfalocele
e) Ombligo amnitico
261.Neonato presenta su brazo izquierdo pegado al tronco, mano y antebrazo en pronacin, codo en extensin y mueca en flexin
palmar. El reflejo prensor de la mano izquierda es normal, pero el reflejo de Moro es asimtrico. A qu corresponde dicha descripcin?
a) Parlisis de Klumpke.
b) Parlisis de Duchenne-Erb.
c) Parlisis braquial total.
d) Parlisis del nervio frnico.
e) Parlisis facial.
262.Neonato de 3 das de vida llega por vmito progresivo y distensin abdominal. Evacu meconio a las 48 horas de vida, pero luego de un
supositorio de glicerina. No ha tolerado la va oral y ha disminuido la diuresis en las ltimas 12 h. Diagnostico probable y tratamiento esencial:
a) Descompresin con sonda nasogstrica y administracin de lquidos intravenosos.
b) Medicin de cloro en sudor.
c) Radiografas simples de abdomen.
d) Hemocultivos y urocultivos seguidos de antibiticos de amplio espectro.
e) Enema de bario.
263.La candidiasis oral (muguet) en un recien nacido a termino de una semana de edad generalmente:
a) Responde bien a la terapia tpica con micostatina.
b) Requiere terapia sistemica con anfotericina B.
c) Requiere terapia topica (micostatina) y terapia sistemica (anfotericina).
d) Requiere investigacion para descartar el sindrome de DiGeorge.
e) Esta asociada con paladar hendido.
REPASO Nro. 1 Pag. 33 USAMEDIC 2017
264.Una nia de 5 aos de edad padece una fiebre de 40C, faringitis y dolor abdominal de forma aguda. Al da siguiente presenta
amigdalitis, faringitis y una lengua blanquecina con la punta y bordes enrojecidos. En la piel un exantema finamente papular que se inicia
en el cuello y axilas y se acenta en los pliegues. Diagnostico mas probable:
a) Escarlatina.
b) Sarampin.
c) Rubola.
d) Varicela.
e) Mononucleosis infecciosa.
265.Un neonato vuelve a los 9 das por un estado de convulsion. Al examen: vesculas en el cuero cabelludo. Prueba virolgica rpida:
diagnostico de HVS 2. El beb debe ingresar para administrarle un antiviral:
a) Ribavirina
b) Aciclovir
c) Ganciclovir
d) Amantadina
e) Zidovudina
266.Cul de estas relaciones entre la afectacin ocular y la infeccin congnita es FALSA?
a) Rubola - cataratas
b) CMV - coriorretinitis
c) VHS - queratoconjuntivitis
d) Toxoplasma glaucoma
e) Sfilis queratitis intersticial
267.Un neonato tiene dificultad para la movilidad de la extremidad superior derecha. Reflejo de Moro asimtrico, tumefaccin y crepitacin
en la palpacin a nivel clavicular. Se confirma disminucion de la movilidad de esa extremidad. El tratamiento mas apropiado es:
a) Ciruga de urgencia.
b) Ciruga cuando el menor llegue a los 2 aos.
c) Reposo y vendaje.
d) Escayolamiento de la extremidad superior.
e) Derivacin al hospital para continuar la exploracin.
268.La tirotoxicosis en el primer dia de vida mayormente ocurre en un infante nacido de madre:
a) Con hipotiroidismo no tratado.
b) Con la enfermedad de Graves no tratada.
c) Con la enfermedad de Graves siendo tratada con antitiroideos.
d) Con bocio eutiroideo.
e) Que recibe yoduros como terapia para bronquitis cronica.
269.Cul de los siguientes es mas comn entre los infantes a termino que los prematuros?
a) Hemorragia intraventricular.
b) Enfermedad hemorragica del recien nacido.
c) Sepsis.
d) Hemorragia subdural.
e) Infeccion congenita.
270.La lactancia con buenos resultados se inicia con:
a) Estimulacin de estrgeno durante el embarazo.
b) Estimulacin de progesterona durante el embarazo.
c) Niveles altos de hCG.
d) Niveles altos de prolactina prximos al trmino.
e) Reduccin posparto en los niveles de esteroides sexuales en circulacin.
271.Ademas de neumonia y sarpullido, la infeccion por mycoplasma pneumoniae en la edad peditrica se asocia ms comnmente con:
a) Complicaciones hepaticas.
b) Complicaciones renales.
c) Complicaciones neurologicas.
d) Anemia aplasica.
e) Supresion inmunologica.
REPASO Nro. 1 Pag. 34 USAMEDIC 2017
272.Cul es la cardiopata congnita predominante en el sndrome de Noonan?
a) Estenosis mitral.
b) Estenosis artica.
c) Estenosis pulmonar.
d) Insuficiencia artica.
e) Tetraloga de Fallot.
273.Relativo al autismo infantil, una de las siguientes afirmaciones es FALSA:
a) Es menos frecuente en mujeres.
b) Se implican factores genticos.
c) Hay gran afectacin de pruebas verbales.
d) El tratamiento es puramente sintomtico.
e) Se manifiesta despus de los 4 aos.
274.En un nio que padece una diarrea mucohemorrgica, se comprueba la etiologa por E. coli 0157: H7. Cul de las complicaciones que
se enumeran est especialmente ligada a este germen?
a) Sndrome hemoltico-urmico.
b) Shock irreversible.
c) Hemorragia subdural.
d) Crisis febril aguda.
e) Trombosis de venas renales.
275.De los siguientes cuadros neonatales, cul se correlaciona mejor con las siguientes anormalidades neurologicas?
a) Bradicardia fetal.
b) Insuficiencia de respiracin al nacimiento.
c) Un bajo puntaje de Apgar al 1 minuto.
d) Un bajo puntaje de Apgar al 5 minuto.
e) Convulsiones en las primeras 36 horas de vida.
276.En la edad peditrica, la manifestacin mas frecuente de la salmonelosis es:
a) Fiebre tifoidea.
b) Sndrome septicmico.
c) Gastroenteritis.
d) Fiebre paratifoidea.
e) Ulceraciones en las placas de Peyer.
277.Un nio de 7 aos de edad tiene daado el tabique interventricular. Cul de las siguientes vlvulas cardacas es ms probable que
funcione defectuosamente?
a) Vlvula pulmonar.
b) Vlvula mitral.
c) Vlvula del seno coronario.
d) Vlvula tricspide.
e) Vlvula artica.
278.La anormalidad roentgenografica ms comn en un nio con asma es:
a) Bronquiectasia.
b) Hiperinsuflacion generalizada.
c) Infiltrados en el lbulo inferior.
d) Neumomediastino.
e) Atelectasia del lbulo medio derecho.
279.Una enfermedad de 2 semanas caracterizada por fiebre progresiva que llega a 40 C y se acompaa de cefalea, malestar general, tos y
dolor abdominal en un nio que acaba de volver de un pas subdesarrollado, se debe con toda probabilidad a:
a) Clera
b) Difteria
c) Shigelosis
d) Fiebre tifoidea
e) Ttanos
REPASO Nro. 1 Pag. 35 USAMEDIC 2017
280.Un lactante de 1 ao de edad presenta retraso de crecimiento, alopecia, evacuaciones voluminosas y exantema cutneo. Se establece
diagnostico presuncional de acrodermatitis enteroptica. Las manifestaciones de este trastorno son causadas por incapacidad para absorber:
a) Zinc.
b) Vitamina K.
c) Vitamina B12
d) Cobre
e) Hierro.
281.Nio de 7 meses con cuadro de 3 das de T 40 C que desaparece el mismo da que inici un exantema. El exantema es mas notable
en el cuello y tronco, de tipo macular, con lesiones dispersas de 3 a 5 mm de dimetro. Cul es el diagnostico mas probable?
a) Eritema infeccioso.
b) Sarampin.
c) Exantema sbito.
d) Rubola.
e) Escarlatina.
282.La primera opcin diagnstica ante una rinorrea supurada crnica unilateral en un nio debe ser:
a) Rinitis crnica inespecfica.
b) Ocena.
c) Cuerpo extrao.
d) Tumor de fosas o senos nasales.
e) Rinitis vasomotora.
283.La incidencia de la coartacin de aorta est notablemente aumentada en:
a) Sndrome de Turner.
b) Sndrome de Marfan.
c) Esclerosis tuberosa
d) Sndrome de Ehlers-Danlos.
e) Sndrome de Down.
284.Un lactante presenta temperatura alta desde hace 3 das y que acaba de desaparecer. Exploracin fsica normal, salvo un exantema
generalizado que ha comenzado justo al acabar la fiebre, y desaparece pocos das despus. Con mas frecuencia produce este cuadro el:
a) Virus herpes tipo 6.
b) Reovirus.
c) Parvovirus B19.
d) Virin rubelico.
e) Echovirus serotipos 17.
285.Paciente de 2 aos, sano, sin antecedentes familiares ni personales de epilepsia presenta coincidiendo con una faringitis bacteriana
convulsiones tnico-clnicas de 12 minutos de duracin. LCR y EEG postcrtico normales. Cul es el diagnstico ms probable?
a) Sindrome de Lennox-Gastaut.
b) Sindrome de West.
c) Convulsiones febriles.
d) Ausencias.
e) Gran mal.
286.Nia de 4 aos, hace 4 das presenta lesiones cutneas, primero rosadas que se transforman en vesculas de contenido transparente,
en tronco, cara y brazo. Algunas vesculas han evolucionado a costras. A la exploracin, tiene 38C, las lesiones vesculo-pustulosas
descritas y, en el velo del paladar se aprecian 5 lesiones ulcerosas de 2 mm de dimetro. Cul es el diagnstico mas probable?
a) Primoinfeccin herptica.
b) Varicela.
c) Sndrome boca-mano-pie.
d) Imptigo contagioso.
e) Erupcin variceliforme de Kaposi.
287.Nia de 13 aos de edad; llega 4 horas despus de ingerir aspirina. Despus del interrogatorio se calcula que ingiri casi 150 mg/Kg.
Exploracin fsica normal. Qu tratamiento es de utilidad?
a) Induccin del vmito.
b) Administracin de N-acetilcistena.
c) Administracin de catrticos.
d) Lquidos intravenosos para incrementar la diuresis.
e) No es necesaria ninguna intervencin mdica.
REPASO Nro. 1 Pag. 36 USAMEDIC 2017
288.Qu diagnostica si un lactante presenta atrializacin del ventrculo derecho, CIA tipo ostium secundum y sd de Wolf-Parkinson-White?
a) Triloga de Fallot.
b) Tronco arterioso asociado al sindrome de DiGeorge.
c) Sndrome de Taussig-Bing.
d) Transposicin de los grandes vasos.
e) Enfermedad de Ebstein.
289.Cul de los siguientes hallazgos le hara dudar del diagnostico de un sndrome de Reye?
a) Edema cerebral.
b) Hiperamoniemia.
c) Trastorno de conciencia.
d) Hgado agrandado.
e) Ictericia.
290.La dacrioadenitis esencial es la inflamacin en los nios de:
a) Las vas de drenaje lagrimales.
b) Las glndulas lagrimales accesorias.
c) La glndula lagrimal principal
d) El saco lagrimal
e) El conducto nasolagrimal.
291.Nio de 2 aos, sin atencin por vivir con su mam en un pueblo, presenta espasticidad con puos muy cerrados y piernas en tijera.
Hiperexcitabilidad reflxica, Babinski y persistencia anormal de reflejos neonatales. Se diagnostica parlisis cerebral. Causa mas probable:
a) Kernicterus.
b) Infeccin cerebral neonatal.
c) Anoxia cerebral neonatal.
d) Hidrocefalia.
e) Epilepsia.
292.Uno de estos elementos presentes en algunas leches es el mayor productor de reacciones alrgicas a la leche en nios:
a) Casena.
b) Beta lactoglobulina.
c) Lipasa.
d) Proteasa.
e) cidos grasos.
293.Cmo es el patrn tpico de afeccin articular en la artritis crnica juvenil poliarticular seropositiva?
a) No tiene patrn definido.
b) De predominio en grandes articulaciones de miembros inferiores.
c) Simtrico de pequeas y grandes articulaciones.
d) Asimtrico de pequeas articulaciones de las manos y pies.
e) De predominio axial con o sin sacroiletis.
294.La agudeza visual alcanza 20/20 a la edad de:
a) 1 semana.
b) 6 meses.
c) 1 ao.
d) 4 - 5 aos.
e) 7 - 9 aos.
295.Un nio presenta agenesia radial y trombopenia. Presenta un soplo pulmonar eyectivo y cianosis. Rx: corazn no cardiomeglico y
con forma de zueco. Diagnostico mas probable:
a) Tetraloga de Fallot.
b) Coartacin artica.
c) Estenosis artica.
d) Estenosis mitral.
e) Hipertensin pulmonar primaria.
296.El diagnstico de fibrosis qustica se basa en:
a) Criterios clnicos (malabsorcin e infecciones respiratorias de repeticin).
b) Criterios clnicos y prueba del sudor.
c) Potencial de membrana en el epitelio basal.
d) Criterios radiolgicos.
e) Criterios clnicos y de funcin pulmonar.
REPASO Nro. 1 Pag. 37 USAMEDIC 2017
297.La marcha de Trendelenburg es positiva si hay debilidad de:
a) Los abductores de la cadera
b) El psoas iliaco
c) Los rectos anteriores del abdomen
d) El sartorio
e) El oblicuo mayor del abdomen
298.Se asocia mas cercanamente al parto por cesarea:
a) Sindrome de dificultad respiratoria.
b) Sindrome de aspiracion meconial.
c) Taquipnea transitoria del recien nacido.
d) Neumonia congenita.
e) HIpoplasia pulmonar.
299.Un neonato de 35 semanas de gestacin con 1.650 gramos de peso, presenta convulsiones, coriorretinitis, megalocefalia, junto con
calcificaciones intracraneales difusas. Qu enfermedad sospechara?
a) Toxoplasmosis congnita.
b) Citomegalia congnita.
c) Embriopata rubelica clsica.
d) Enfermedad fetal rubelica.
e) Sfilis tarda
300.Una de las siguientes NO forma parte de la sfilis congnita tarda:
a) Queratitis intersticial
b) Dientes de Hutchinson
c) Paresia juvenil
d) Sordera y vrtigo por afectacin de VIII par
e) Pnfigo sifiltico
301.Cul de los siguientes factores NO predispone a una enterocolitis necrotizante en un neonato?
a) Prematuridad.
b) Alimentacin materna.
c) Poliglobulia.
d) Exanguinotransfusin.
e) Asfixia neonatal.
302.Un lactante de 15 das de vida presenta ambigedad genital desde el nacimiento. Comienza con un cuadro de vmitos, deshidratacin
y tendencia al colapso cardiocirculatorio. Desde un punto de vista analtico presenta una acidosis metablica, hiponatremia y natriuresis
elevada. Qu enfermedad presenta este paciente?
a) Un trastorno de la esteroidognesis suprarrenal.
b) Una disgenesia gonadal.
c) Una estenosis hipertrfica del ploro.
d) Una anomala del receptor andrognico.
e) Un hermafroditismo verdadero.
303.Un varn que desde los primeros das de vida, alimentado con leche materna, presenta una diarrea acuosa, deshidratacin y acidosis
metablica grave. Todos los intentos de alimentacin con lactancia materna que se hicieron tuvieron el mismo resultado. Biopsia yeyunal
y actividad de disacaridasas intestinales normales. Diagnostico mas probable:
a) Malabsorcin primaria de lactosa.
b) Dficit congnito de lactasa.
c) Intolerancia a las protenas de la leche de vaca.
d) Atrofia congnita de microvillis.
e) Gastroenteritis crnica.
304.La deficiencia de piruvatocinasa es la ms comn deficiencia enzimatica glucolitica. Cul de los siguientes es un signo clnico comn
en la presentacion neonatal de este trastorno?
a) Sepsis.
b) Ictericia.
c) Hepatomegalia.
d) Hipertonia.
e) Convulsiones.
REPASO Nro. 1 Pag. 38 USAMEDIC 2017
305.El pie zambo congenito comunmente se asocia ms con trastornos de uno de los siguientes sistemas:
a) El sistema nervioso central.
b) El sistema hematopoyetico.
c) El sistema gastrointestinal.
d) El sistema cardiovascular.
e) El sistema respiratorio.
306.Un neonato prematuro que al momento del nacimiento (6-12 h luego) presenta dificultad respiratoria del tipo de distres respiratorio
idioptico del recien nacido. En la fisiopatologa de esta enfermedad hay unos factores que facilitan la maduracin pulmonar fetal. Uno de
los siguientes acta como inhibidor de la maduracin pulmonar:
a) Hemorragia retroplacentaria.
b) Hidrops fetalis.
c) Herona.
d) Hipertensin.
e) Infarto placentario.
307.En el abordaje de atencin al recin nacido NO es correcto:
a) Comprobar que el cordn umbilical tiene dos arterias.
b) Comprobar que el cordn umbilical tiene una vena.
c) Pinzar el cordn alrededor de los 60 segundos.
d) Retirar el vermix caseoso.
e) Comprobar el cordn umbilical.
308.La anemia ferropenica del nio se caracteriza por los siguientes hechos, EXCEPTO:
a) Es la causa mas frecuente de anemia nutricional de la infancia.
b) La lactancia materna prolongada evita su aparicin.
c) Su desarrollo esta favorecido por las infecciones repetidas.
d) El diagnostico se basa en la determinacin de la ferritina srica.
e) La fase de ferropenia latente carece de sntomas clnicos.
309.Un nio con retraso mental y del crecimiento, presenta caractersticamente el puo cerrado con los dedos 2 y 5 superpuestos al 3 y
4, pies en mecedora con calcneo prominente y pabellones auriculares grandes. Qu sndrome sospecha?
a) Sndrome de Edwards.
b) Sndrome de Down.
c) Sndrome de Patau.
d) Sndrome del maullido del gato.
e) Sndrome de Klinefelter.
310.Un nio de 7 aos de edad presenta tras un tratamiento con sulfamidas una crisis de dolor abdominal y necrosis del dedo meique; en
la analtica se observa una disminucin de la haptoglobina y hemoglobina de 10 g/dl. Diagnostico mas probable:
a) Dficit de G6PD (glucosa-6-piruvato deshidrogenasa).
b) Dficit de piruvato quinasa.
c) Hb S.
d) Beta talasemia.
e) Eliptocitosis.
311.Nio de 3 aos que comienza bruscamente con fiebre de 38 C, intranquilidad, tos ronca de tipo metlica, disfona, estridor inspiratorio
y tiraje de grado medio, es sospechoso de padecer:
a) Neumona vrica.
b) Neumona bacteriana.
c) Laringitis subgltica.
d) Bronquitis aguda viral.
e) Absceso retrofarngeo.
312.Nia de 14 aos presenta amenorrea primaria. Exploracin fsica: talla baja, cuello corto y lnea de implantacin posterior del cabello
baja. Cul es la anomala cromosmica ms probable?
a) Sndrome del cromosoma X frgil.
b) Trisoma 18.
c) Trisoma 21.
d) 45 XO.
e) XXY.
REPASO Nro. 1 Pag. 39 USAMEDIC 2017
AREA DE CIRUGIA

313.La zona II del cuello se encuentra:


a) Por encima de las clavculas hasta el cartlago tiroides
b) Desde el cartlago cricoides hasta la base del crneo
c) Desde el cartlago tiroides hasta el borde inferior de la mandbula
d) Desde el cartlago cricoides hasta el borde inferior del maxilar inferior
e) Por debajo de las clavculas hasta el cartlago cricoides
314.Para abordar las lesiones de la Zona III del cuello se necesita:
a) Una incisin en collar (Kocher)
b) Una incisin supraclavicular
c) Luxacin de la mandbula
d) Una incisin oblicua
e) Una incisin transversa
315.La cmara cardiaca ms frecuentemente comprometida por lesin de arma de fuego es:
a) La auricula derecha
b) El ventriculo derecho
c) El ventriculo izquierdo
d) La auricula izquierda
e) Cualquiera puede ser lesionada con la misma frecuencia
316.Un nio de 9 aos, tras una cada del columpio, presenta en su antebrazo intenso dolor espontneo, impotencia funcional absoluta y
gran deformidad del vrtice volar en el tercio medio. Diagnostico mas probable:
a) Fractura diafisaria de cbito y radio
b) Epifisiolisis de mueca
c) Fractura de Smith
d) Fractura-luxacin de Monteggia
e) Fractura de Colles
317.Un varn presenta, tras un sondaje urinario, un dolor lumbar y escalofrio de inicio brusco, al tacto rectal se halla una prstata
inflamada, caliente y dolorosa. Diagnostico mas probable:
a) Prostatitis aguda.
b) Prostatitis crnica.
c) Prostatitis no bacteriana.
d) Prostatodinia.
e) Uretritis.
318.Un paciente presenta un aumento del volumen escrotal, pesadez y cierto dolorimiento en el testculo izquierdo que aumenta con la
bipedestacin. Cul es su diagnostico probable?
a) Hidrocele.
b) Varicocele.
c) Seminoma.
d) Epididimitis.
e) Torsin de hidtide.
319.El tratamiento para un retardo de consolidacin es:
a) Reduccin cruenta de los bordes de la fractura.
b) Colocar injertos
c) Realizar un correcto enclavamiento intramedular.
d) Prolongar el tratamiento de la fractura.
e) Realizar osteosntesis precoz.
320.Un alcohlico crnico, con antecedente de haber participado en una ria callejera, acude a revisin; refiere dolor generalizado en la
mano derecha, que tiene tumefaccin y deformidad en el dedo meique. El diagnstico es fractura de:
a) Colles
b) Escafoides
c) Boxeador
d) Falange
e) Primer dedo
REPASO Nro. 1 Pag. 40 USAMEDIC 2017
321.La existencia de una parlisis bicipital en la pierna derecha tras una luxacin posterior de cadera, nos orienta hacia la lesin de:
a) Nervio crural.
b) Nervio citico.
c) Nervio obturador.
d) Nervio femoro-cutneo.
e) Races S1 y S2.
322.Un deportista joven llega tras sufrir un traumatismo sobre su rodilla, a raz del cual viene notando episodios de bloqueo articular y
derrame. Aporta RMN donde se aprecia una rotura en asa de cubo del menisco interno. El tratamiento mas adecuado es:
a) Observacin y antiinflamatorios.
b) Reposo absoluto y rodillera de estabilizacin articular.
c) Tratamiento quirrgico mediante ciruga artroscpica.
d) Tratamiento quirrgico mediante ciruga abierta.
e) Inmovilizacin con yeso.
323.La contractura isqumica de Volkmann est relacionada con:
a) Fractura de Colles
b) Separacin traumtica del hombro
c) Fractura de la difisis del fmur
d) Fractura supracondlea del hmero
e) Contractura de Dupuytren
324.A un recien nacido pretermino, que cuando le toca la primera toma la rechaza, est ictrico con distensin abdominal y en la ecografa
descubrimos aire en la zona portal, pensamos en:
a) Atresia esofgica.
b) Estenosis hipertrfica de ploro.
c) Atresia duodenal.
d) Enterocolitis necrotizante.
e) Invaginacin intestinal.
325.Cul de las siguientes lesiones, en un paciente politraumatizado, requiere ser intervenida quirurgicamente, de ser posible, en el
perodo primario de su evolucin (antes de las primeras 24-48 horas)?
a) Reconstrucciones articulares acetabulares.
b) Fracturas del antebrazo.
c) Fracturas femorales.
d) Aplicacin de injertos seos.
e) Ninguna anterior debe hacerse tan precozmente.
326.Un neonato menor de 1 ao tiene un cuadro de obstruccin intestinal con emisin de sangre por el ano. Seale el diagnstico
correcto:
a) Poliposis colnica.
b) Diverticulitis.
c) Divertculo de Meckel.
d) Invaginacin intestinal.
e) Atresia intestinal.
327.Una de las siguientes fracturas NO es tpica en la cada vertical sobre los talones:
a) Difisis femoral
b) Calcneo.
c) Vertebras lumbares.
d) Vertebras dorsales.
e) Cuello femoral.
328.Una mujer de 50 aos de edad presenta una fibrilacion auricular crnica en tratamiento con digoxina y antiagregantes; se presenta
bruscamente dolor, palidez, anestesia e impotencia funcional en el pie, pierna y mitad inferior de muslo derecho. Ausencia de pulsos
pedio, tibial posterior y poplteo derecho. Diagnostico mas probable:
a) Trombosis venosa profunda.
b) Trombosis venosa superficial.
c) Arteritis de Takayasu.
d) Embolia arterial aguda.
e) Tromboangeitis obliterante.
REPASO Nro. 1 Pag. 41 USAMEDIC 2017
329.Dnde se localiza la acumulacin de sangre en el otohematoma?
a) Cartlago y pericondrio
b) Pericondrio y piel.
c) Piel y tejido graso.
d) Tejido graso y pericondrio.
e) Tmpano y pared interna de caja.
330.Un paciente de 58 aos de edad que se trata con insulina desde hace 12 aos, acude a urgencias por haber notado una repentina
disminucin de visin en ojo derecho. La agudeza visual es de contar dedos a 50 cms. No tiene dolor y el segmento anterior del ojo es
normal. Cual es la causa mas probable de esta perdida de visin?
a) Catarata.
b) Uveitis anterior.
c) Presbicia.
d) Hemorragia vtrea.
e) Glaucoma neovascular.
331.Un enfermo que sufre ceguera repentina, sin dolor ni signos oculares externos. Qu puede ser?
a) Obstruccin de la arteria central de la retina.
b) Conjuntivitis bilateral intensa.
c) Glaucoma.
d) Iritis.
e) Retinoblastoma.
332.Cul anestesia es la ms usada en pacientes que no requieren de la regional?
a) Anestesia general inhalatoria
b) Anestesia general endovenosa
c) Anestesia general balanceada
d) Anestesia total intravenosa
e) Neuroleptoanestesia
333.Indique el sntoma cardinal de la otitis externa:
a) Secrecin excesiva de cerumen.
b) Dolor a la presin en el trago.
c) Vrtigo.
d) Hipoacusia.
e) Signo de la fistula positivo.
334.En el traumatismo de colon el factor de riesgo ms importante es:
a) El tiempo de evolucin de la herida
b) La edad del paciente (mayor de 40 aos)
c) Heridas asociadas
d) Presencia de shock
e) Hemoglobina baja
335.La herida de la vesicular biliar amerita:
a) Sutura
b) Sutura ms drenaje
c) Colecistectoma
d) Colecistectoma dependiendo del grado de la lesin
e) Solo drenaje
336.Paciente que recibi dos heridas por proyectil de arma de fuego: una a nivel del miembro superior izquierdo (brazo) con lesin
humeral sangrante; y otra, con localizacin toracoabdominal que parece presentar lesin de bazo. Su estado es de shock profundo. El
manejo de este paciente debe ser, adems del ABC:
a) Laparatomia, esplecnotomia y reparo vascular
b) Reparo vascular, laparotoma y esplenectoma
c) Laparotoma, esplenectoma y ligadura vascular
d) Laparotoma, reparo vascular humeral y esplenectoma
e) Reparo vascular, ligadura vascular y laparotomia
REPASO Nro. 1 Pag. 42 USAMEDIC 2017
337.Paciente femenino de 28 aos que recibi politraumatismo al estrellarse en automvil; hubo un pasajero muerto dentro del mismo
vehculo. Ella se encuentra hemodinmicamente estable y con leves laceraciones superficiales. La conducta a seguir es:
a) Darle de alta
b) Formularle analgsicos y darle de alta
c) Hacer la evaluacin primaria y tomar rayos X de columna cervical lateral, torax y pelvis
d) Observacin
e) Atencin domiciliaria
338.Un paciente anticoagulado con warfarina necesita ciruga de urgencia. La mejor forma de prepararlo es:
a) Usar sulfato de protamina para revertirlo
b) Dosis grandes de vitamina K
c) Plasma fresco congelado
d) Cualquiera de las anteriores es til
e) Altas dosis de Clomifeno
339.La causa ms frecuente (etiopatogenia) del absceso heptico pigeno es:
a) Infeccin ascendente biliar
b) Diseminacin hematgena por la porta
c) Septicemia por la arteria heptica
d) Extensin directa de infeccin intraperitoneal
e) Infeccin descendente biliar
340.El elemento ms importante en el diagnstico de la diverticulitis es:
a) El colon por enema
b) La rectosigmoidoscopia
c) El cuadro clnico
d) La TAC
e) La ecografia
341.Las principales causas de HVDB que pueden producir con ms frecuencia un sangrado masivo son:
a) Los plipos y el cncer
b) Las angiodisplasias y los divertculos de colon
c) La enfermedad inflamatoria del intestino y las fisuras anales
d) Las hemorroides y los divertculos de intestino delgado
e) La colitis ulcerativa y el cancer
342.Cul es la causa ms frecuente de pancreatitis aguda en nuestro medio?
a) Idioptica.
b) Medicamentosa.
c) Traumtica.
d) Biliar.
e) Post colangiopancreatografa.
343.A qu se denomina hernia incarcerada?
a) Aquella que contiene asas intestinales en su interior.
b) La que ha sido intervenida previamente.
c) La que est perforada.
d) La que tiene problemas de irrigacin.
e) La que es imposible reducir.
344.Mujer de 65 aos con historia de estreimiento crnico es transferida de un asilo debido a un dolor abdominal y una distensin
abdominal marcada. Su abdomen se hallaba distendido y sensible en el cuadrante inferior izquierdo. Diagnstico ms probable:
a) Apendicitis.
b) Carcinoma de colon.
c) Vlvulo del colon sigmoide.
d) Vlvulo del ciego.
e) Obstruccin del intestino delgado.
345.En la quimioterapia del cncer colorrectal el agente aislado mas eficaz es:
a) Metotrexato.
b) Vincristina.
c) Ciclofosfamida.
d) 5-Fluorouracilo.
e) Cisplatino.
REPASO Nro. 1 Pag. 43 USAMEDIC 2017
346.Llega a Urgencias un paciente con quemaduras en diversas partes de su cuerpo. Despus de estabilizarle sus signos vitales
constantes, se plantea su hospitalizacin. Solo uno de los siguientes criterios NO es utilizado para hospitalizar a este tipo de pacientes:
a) Nios con extensin quemada mayor de 10%.
b) Adultos con extensin quemada mayor de 20%.
c) Quemadura de 3 grado con extensin quemada menor a 0.5%.
d) Lesiones de vas areas por inhalacin.
e) Lesiones por electrocucin.
347.Un varn de 18 aos llega con dolor anal punzante intenso, durante la evacuacin y despus de ella, con evolucin de 1 mes. La mitad
de las veces, se acompaa de sangre fresca escasa en el papel higinico. En la exploracin del ano, en la comisura posterior, se observa
una fisura anal. El origen mas probable de la fisura es:
a) Parasitario
b) Enfermedad de Crohn
c) Infeccioso
d) Neoplsico
e) Traumtico
348.Varn de 30 aos de edad presenta un cuadro de 6 horas con dolor epigstrico de inicio sbito. Febrcula, dolor a la palpacin de todo
el abdomen con rigidez involuntaria de la pared abdominal. La Rx de abdomen muestra neumoperitoneo. El diagnostico mas probable es:
a) Apendicitis aguda.
b) lcera pptica perforada.
c) Diverticulitis de sigmoides.
d) Pancreatitis aguda.
e) Vlvulo de ciego.
349. Cul es el procedimiento diagnstico inicial en sospecha clnica de colelitiasis?
a) Colangiografa endovenosa.
b) Tomografa abdominal.
c) Ecografa abdominal.
d) Colangiografa retrgada endoscpica.
e) Radiografa de abdomen simple.
350. Cul es el manejo ms conveniente de la herida operatoria en apendicitis aguda con peritonitis generalizada?
a) Cerrada.
b) Cerrada con dren laminar.
c) Abierta con cierre diferido.
d) Semi abierta cerrada con dren tubular.
e) Cerrada con infiltracin de antibiticos.
351.Un abogado de 38 aos desarrolla dolor abdominal despus de ingerir una comida grasosa. El examen revela dolor a la palpacin en el
hipocondrio derecho y signo de Murphy positivo. Qu prueba tiene ms posibilidades de revelar una colecistitis aguda?
a) Exploracin HIDA.
b) Colecistograma oral.
c) Colangiograma intravenoso.
d) TC del abdomen.
e) CPRE.
352.Un paciente diagnosticado de colitis ulcerosa universal hace 18 aos presenta unas heces acintadas, en el enema se objetiva una
estenosis. Diagnostico mas probable:
a) Absceso perirrectal.
b) Fstulas.
c) Cncer de colon.
d) Pseudopoliposis.
e) Disfuncin.
353.Un varn de 35 aos de edad presenta un sindrome de poliposis gastrointestinal. Al hacer un estudio histolgico, se aprecian
hamartomas. Tiene afectacin del estmago, intestino delgado e intestino grueso. Se transmite con herencia autosmica dominante.
Asociado a todo lo anterior, aparecen papilomas y cncer de tiroides. Cul es su diagnstico mas probable?
a) Sndrome de Turcot.
b) Sndrome de Cowden.
c) Sndrome de Cronkhite-Canada.
d) Sndrome de Peutz-Jeghers.
e) Sndrome de Gardner.
REPASO Nro. 1 Pag. 44 USAMEDIC 2017
354.Un varn de 23 aos de edad presenta una pigmentacin mucocutnea. En la endoscopa se observan plipos intestinales en
estmago y duodeno. Su padre y abuelo tambin tienen la pigmentacin. Qu diagnstico se plantea?
a) Sindrome de Gardner.
b) Sindrome de Turcot.
c) Sindrome de Peutz-Jeghers.
d) Poliposis colnica juvenil.
e) Poliposis colnica familiar.
355.Acerca de las manifestaciones clnicas del glaucoma de ngulo abierto, cul es la mas frecuente?
a) Asintomticos.
b) Sntomas atpicos.
c) Halos luminosos.
d) Cefaleas.
e) Visin borrosa.
356.Cul de las siguientes conjuntivitis es la menos probable de que ocurra de forma bilateral?
a) Conjuntivitis alrgica.
b) Conjuntivitis viral.
c) Conjuntivitis bacteriana.
d) Conjuntivitis vernal.
e) Conjuntivitis micotica.
357.El tipo de malformacin anorrectal ms comn es:
a) El ano imperforado.
b) La agenesia anal.
c) La agenesia anorrectal.
d) La atresia rectal.
e) El megacolon congnito.
358.Las fracturas-luxaciones del antebrazo reciben el nombre de:
a) Tallo verde.
b) Incurvacin plstica diafisaria.
c) Monteggia y Galeazzi.
d) Pronacin dolorosa.
e) Epifisiolisis.
359.Hombre de 20 aos sufre una cada sobre el brazo derecho extendido en abduccin; presenta dolor intenso, sobre todo a la rotacin
interna de su brazo afectado; una hora ms tarde siente adormecimiento en el hombro ipsolateral. El diagnstico mas probable es:
a) Luxacin glenohumeral anterior
b) Luxacin glenohumeral inferior
c) Desgarro del manguito rotador
d) Lesin de Hill-Sachs
e) Luxacin glenohumeral posterior
360.De los estudios que se pueden ordenar al paciente, el ms importante es:
a) Radiografas en proyeccin de Waters
b) Radiografas anteroposteriores y laterales de la escpula
c) Radiografa de axila
d) Artrografa o artroscopia
e) Resonancia magntica
361.En la fractura-luxacin de Monteggia se puede lesionar una de las siguientes:
a) El nervio radial.
b) El nervio mediano.
c) El nervio cubital.
d) La arteria humeral.
e) La arteria radial.
362.Cul es el sintoma de presentacin mas frecuente del embolismo pulmonar?
a) Hemoptisis
b) Dolor pleurtico
c) Disnea de aparicin brusca
d) Tos coqueluchoide
e) Agitacin
REPASO Nro. 1 Pag. 45 USAMEDIC 2017
363.En un paciente de 63 aos con retencin urinaria, en quien no se puede cateterizar la vejiga, qu posibilidad diagnstica es
probable?
a) Estenosis uretral
b) Espasmo del esfnter externo
c) Hipertrofia prosttica
d) Contractura del cuello de la vejiga
e) Cncer de prstata
364.Nia de 7 aos tiene padecimiento de 2 das con dolor en el odo derecho. No hay fiebre, congestin u otros sntomas. Tiene dolor con
el desplazamiento del pabelln auricular. Afebril y hay drenaje a travs del CAE derecho. Cul es el diagnstico mas probable?
a) Otitis media aguda con perforacin.
b) Dermatitis de contacto.
c) Cuerpo extrao en el conducto auditivo externo.
d) Mastoiditis.
e) Otitis externa.
365.Un jugador de ftbol americano extiende su brazo derecho para hacer una tacleada, pero experimenta dolor intenso en el contacto,
con incapacidad subsiguiente para mover el brazo derecho. El examen revela edema y dolor a la palpacin en el hombro con prdida del
contorno deltoideo normal. Cul es el diagnstico ms probable?
a) Lesin del plexo braquial.
b) Dislocacin anterior del hombro.
c) Fractura de la porcin posterior proximal del hmero.
d) Rotura del msculo deltoides.
e) Dislocacin posterior del hombro.
366.Un nio tras una cada sobre su mano, presenta una epifisiolisis en el extremo distal del radio, cuya lnea de fractura presenta una
prolongacin diafisaria. Seale que tipo es:
a) Grado I.
b) Grado II.
c) Grado III.
d) Grado IV.
e) Grado V.
367.Mujer de 45 aos presenta desde hace meses dolor y parestesias en los 3 primeros dedos de la mano izquierda sobre todo por la
noche. Discreta atrofia de la eminencia tenar, los signos de Phalen, Tinel y Durkan son positivos. Qu diagnostico sospechara?
a) Sndrome del canal de Guyon.
b) Sndrome del tnel carpiano.
c) Tenosinovitis de De Quervain.
d) Compresin del nervio interseo posterior.
e) Afectacin del nervio musculocutneo.
368.La embolia grasa es una complicacin frecuente de:
a) Luxaciones de cadera.
b) Quiste seo solitario
c) Fractura de escapula.
d) Esguince grave de rodilla.
e) Fractura diafisaria del fmur.
369.El tipo celular mas comn en el cncer testicular es el seminoma. Qu marcador srico puede utilizarse para vigilar el seguimiento
del tratamiento de este tipo de cncer?
a) Antgeno carcinoembrionario (CEA).
b) Gonadotropina corinica humana (hCG).
c) Tasa de eritrosedimentacin.
d) Deshidrogenasa de cido lctico (LDH).
e) Antgeno prosttico especfico (PSA).
370.La causa mas frecuente de vmito no bilioso en el neonato es:
a) Estenosis hipertrfica del ploro.
b) Vlvulo gstrico.
c) Duplicacin gstrica.
d) Atresia esofgica.
e) Atresia gstrica.
REPASO Nro. 1 Pag. 46 USAMEDIC 2017
371.Un politraumatizado presenta una lesin en D1 inestable, rotura esplnica, un neumotrax asfixiante y la fractura de la tibia derecha.
Qu lesin de las siguientes es la mas urgente?
a) La lesin vertebral.
b) La fractura tibial.
c) Todas por igual.
d) El neumotrax.
e) La rotura esplnica.
372.Uno de los siguientes grados de quemadura es menos doloroso:
a) De primer grado
b) De segundo grado superficial
c) De segundo grado profunda
d) De tercer grado
e) De primer grado y segundo grado
373.Un hombre de 72 aos de edad operado de apendicectoma hace 15 aos, presenta clico tipo retortijn con intervalos de alivio y
reaparece con vmito de contenido gstrico. El diagnstico ms probable es:
a) Cncer del sigmoide
b) Embolia mesentrica
c) Hernia interna
d) Obstruccin por adherencias
e) leo paraltico
374.A un varn de 45 aos se le evala una masa en la cara posterior del cuello. La masa ha estado presente por varios aos, pero
aument de tamao en los ltimos 2 aos. Es una masa subcutnea de consistencia blanda, no dolorosa y desplazable. Diagnostico mas
probable:
a) Carcinoma tiroideo.
b) Lipoma.
c) Adenoma pleomrfico.
d) Carcinoma larngeo.
e) Adenoma paratiroideo.
375.Si en la palpacin de la glndula tiroides se detecta una tumoracin nodular, irregular y de consistencia dura, es probable que se trate de:
a) Absceso tiroideo.
b) Ganglio linftico hipertrfico.
c) Carcinoma.
d) Tiroiditis.
e) Higroma qustico.
376.En qu basara con mayor confianza el diagnstico de una pancreatitis crnica?
a) Hiperamilasemia.
b) Mayor densidad de la zona pancretica en la Rx abdominal.
c) Pncreas engrosado en la ecografa.
d) Anomalas del conducto de Wirsung en la CPRE.
e) Esteatorrea en el test de D-xilosa normal.
377.La apendicitis aguda es la urgencia Qx abdominal mas frecuente. Hay complicaciones en el 10 % de casos. La mas frecuente es:
a) Perforacin.
b) Infeccin de la herida operatoria.
c) Linfadenitis mesentrica reactiva.
d) Absceso.
e) Plastrn.
378.Dnde se localiza la acumulacin de sangre en el otohematoma?
a) Pericondrio y piel.
b) Piel y tejido graso.
c) Cartlago y pericondrio
d) Tejido graso y pericondrio.
e) Tmpano y pared interna de caja.
REPASO Nro. 1 Pag. 47 USAMEDIC 2017
379.La complicacin ms frecuente de la anestesia epidural es:
a) Falla en la insercin del catter
b) Dolor
c) Hipotensin
d) Mala difusin del anestsico en el rea prevertebral
e) Espasmos en las extremidades
380.En la operacin abierta se halla un apndice normal. Cul es el procedimiento ms comn que un cirujano debe realizar si encuentra
un apndice normal?
a) Evaluar la pelvis por enfermedad inflamatoria plvica por abscesos ovricos, malignidad o embarazo ectpico.
b) Remover el apndice.
c) Evaluar el leon terminal y el ciego por signo de enteritis regional o bacteriana.
d) Evaluar el abdomen superior por colecistitis o lcera duodenal perforada.
e) Evaluar por divertculo de Meckel.
381.Un hombre de 33 aos se lesiona en un accidente automovilstico. En la cirugia se observa una laceracin heptica y sangrado
intenso. Cul de las siguientes opciones sera relativamente bien tolerada por el paciente?
a) Sangrado heptico persistente.
b) Comunicacin portacava.
c) Ligadura de la vena porta.
d) Ligadura arterial heptica.
e) Hemorragia digestiva por rotura de vrices esofgicas.
382.Una chica de 19 aos de edad ingiri vitaminas que contenan hierro. Cules son los primeros sntomas que se observaran en la
evolucin del envenenamiento agudo con hierro?
a) Ditesis hemorrgica.
b) Hemorragia gastrointestinal.
c) Insuficiencia heptica.
d) Prolongacin del intervalo QT.
e) Insuficiencia respiratoria.
383.Mujer de 45 aos, digitadora a 12 horas diarias. Refiere hemorroides que sangran frecuentemente con la deposicin y que prolapsan fuera
del canal anal, siendo dificultosa su reduccin. Refiere prurito anal y dolorimiento ligero en la zona perianal. Qu tratamiento indicara?
a) Dieta rica en fibra mas laxantes.
b) Esclerosis de las hemorroides.
c) Hemorroidectoma quirrgica.
d) Ligadura con bandas elsticas.
e) Cualquiera de las anteriores.
384.En cul tipo de hernia es frecuente asociar a la reparacin de la misma una reseccin intestinal?
a) Incarceradas
b) Crurales
c) Umbilicales
d) Estranguladas
e) Femorales
385.Complicacin mas frecuente del divertculo de Meckel:
a) La inflamacin sin peritonitis
b) La perforacin con peritonitis
c) La hemorragia
d) La obstruccin intestinal
e) La malignizacin
386.Varn de 72 aos de edad, presenta rectorragia, moderado dolor abdominal de predominio en fosa ilaca izquierda y febrcula de 2 das
de evolucin. Signos locales de peritonismo y defensa muscular. Cual es la causa mas frecuente de este cuadro?
a) Diverticulitis clica aguda.
b) Angiodisplasia de colon.
c) Cncer de colon.
d) Colitis ulcerosa.
e) Sndrome de Ogilvie.
REPASO Nro. 1 Pag. 48 USAMEDIC 2017
387.De los siguientes tumores benignos del intestino delgado, uno de ellos presenta mayor potencial para malignizarse:
a) Neurofibroma
b) Plipo hamartomatoso
c) Plipo adenomatoso en la poliposis familiar mltiple
d) Poliposis juvenil
e) Fibroma
388.El objetivo principal de la ecografa en pacientes con traumatismo abdominal cerrado o no penetrante, es detectar
a) Hematoma subcapsular heptico.
b) Neumoperitoneo.
c) Laceracin esplnica.
d) Perforacin del duodeno.
e) Lquido libre intraperitoneal.
389.Uno de los accidentes operatorios mas frecuente de la hernia inguinal es:
a) Herida de la arteria epigstrica.
b) Herida de los vasos femorales.
c) Seccin del conducto deferente.
d) Herida del intestino.
e) Herida de la vejiga.
390.Mujer de 35 aos de edad; fumadora, nulpara, hbito longilneo, en tratamiento con ACO, que sbitamente presenta dolor en el
costado izquierdo, con disnea y tos. Su ECG es normal. Gasometra arterial 85% de la saturacin de oxgeno, PO2 de 55, con PCO2 de 21.
Temperatura 38.5 C. Rx de trax normal. Su primer diagnostico de sospecha:
a) Neumona.
b) Infarto agudo de miocardio.
c) Neumotrax.
d) Tromboembolismo pulmonar.
e) Rotura costal espontnea.
391.Un nio de 9 aos de edad trado a la Urgencia tras cada de un columpio. Presenta su antebrazo con intenso dolor espontaneo,
impotencia funcional absoluta y gran deformidad de vrtice volar en tercio medio. Cul sera el diagnostico mas probable?
a) Fractura diafisaria de cubito y radio.
b) Epifisiolisis de mueca
c) Fractura de Smith.
d) Fractura-luxacin de Monteggia.
e) Fractura de Colles.
392.Cul de las siguientes enfermedades se presenta con elevacin del periostio, secuestro e involucro seo?
a) Enfermedad de Paget.
b) Osteosarcoma.
c) Osteomielitis.
d) Osteocondroma.
e) Ninguna de las opciones anteriores es correcta.
393.Obeso de 45 aos, cuyos nicos antecedentes son una hipercolesterolemia sin control y una ingesta de aprox. 100 gr de alcohol
diarios, consulta por un dolor muy intenso en el muslo de inicio brusco hace 2 semanas, sin antecedente traumtico. Camina a duras
penas con marcada cojera y ayudndose de 2 muletas. Diagnostico mas probable:
a) Osteoporosis transitoria.
b) Coxartrosis.
c) Fractura por estrs del cuello femoral.
d) Espondilitis anquilosante con afectacin monoarticular.
e) Necrosis isqumica de la cabeza femoral.
394.A un nio de ao y medio de edad lo llevan a consulta con el cirujano pediatra. Sus padres han notado un tumor de aspecto
translcido en el escroto derecho; ste aparece cuando se despierta y va decreciendo durante el da, adquiriendo su tamao mximo antes
de irse a dormir. El diagnstico mas probable es:
a) Hernia inguinal
b) Hidrocele no comunicante
c) Hidrocele comunicante
d) Hernia femoral
e) Tumor de testculo
REPASO Nro. 1 Pag. 49 USAMEDIC 2017
395.La cirugia mas realizada para el tratamiento de la hiperplasia prosttica benigna es:
a) Reseccin transuretral.
b) Adenomectoma retropbica.
c) Prostatectoma suprapbica.
d) Prostatectoma perineal.
e) Incisin del cuello vesical.
396.Nio de 2 aos es trado por su padre porque llevndolo de la mano, y tras tropezar para evitar que se cayera, lo ha levantado tirando
de la mano hacia arriba. El nio presenta impotencia funcional de ese brazo, y dolor a la movilizacin pasiva. Diagnostico mas probable:
a) Luxacin proximal de cbito.
b) Epifisilisis distal del hmero.
c) Subluxacion de la cabeza del radio
d) Arrancamiento del plexo braquial.
e) Fractura de hmero.
397.La complicacin mas frecuente de las fracturas de la difisis tibial es:
a) Sndrome compartimental.
b) Amputacin.
c) Pseudoartrosis.
d) Acortamiento del miembro.
e) Hematoma postraumtico.
398.Un anciano llega por una cada tras la cual se ve imposibilitado para deambular, no remite el dolor en el rea inguinal. La pierna
lesionada aparece con una rotacin externa y con ligero acortamiento. El diagnostico mas probable es:
a) Fractura del cuello femoral.
b) Luxacin de la cadera.
c) Fractura subtrocantrea.
d) Fractura de la difisis femoral.
e) Epifisilisis.
399.En el tratamiento de las fracturas del escafoides carpiano es cierta una de las siguientes afirmaciones:
a) Se inmovilizan incluyendo siempre el pulgar.
b) Se inmovilizan incluyendo siempre el codo
c) Se inmovilizan incluyendo siempre el pulgar y el codo.
d) No se incluye nunca el pulgar.
e) No se incluye nunca el codo.
400.Paciente con severo dolor, palidez y parestesias distales a una lesin de la extremidad. Hay preocupacin relativa al yeso asociado
con esta lesin. La contractura isqumica de Volkman se asocia con:
a) Fractura femoral intertrocanterica.
b) Fractura supracondilea del hmero.
c) Dislocacin posterior de la rodilla.
d) Separacin traumtica del hombro.
e) Fractura de Colles.
401.La presentacin usual de un pancreas anular en la niez es:
a) Hipoglucemia.
b) Acidocis hiperglucemica.
c) Ictericia.
d) Vomito.
e) Esteatorrea.
402.Ante la visualizacin por otoscopia de masas algodonosas en el CAE, en cul de las siguientes entidades se debe pensar?
a) Tuberculosis.
b) Micosis.
c) Cuerpo extrao.
d) Otitis serosa.
e) Eczema.
REPASO Nro. 1 Pag. 50 USAMEDIC 2017
403.Llega un paciente de color de 52 aos refiriendo una tumoracin laterocervical que, segn nos comenta, aumenta de tamao cuando
toca el saxofn. Posiblemente estamos ante un:
a) Quiste dermoide.
b) Linfangioma qustico.
c) Quemodectoma carotideo.
d) Quiste branquial.
e) Laringocele externo.
404.Ante un RN pretermino, que cuando le toca la primera toma la rechaza, se halla ictrico con distensin abdominal y en la ecografa
descubrimos aire en la zona portal, pensamos en:
a) Atresia esofgica.
b) Estenosis hipertrfica de ploro.
c) Atresia duodenal.
d) Enterocolitis necrotizante.
e) Invaginacin intestinal.
405.Mujer de 22 aos de edad presenta fiebre, perdida de peso, sudores nocturnos y agrandamiento indoloro de varios ndulos linfticos
supraclaviculares. Una biopsia de uno de estos ndulos muestra una clula gigante binucleada o bilobulada con prominentes nucleolos
acidoflicos tipo ojo de bho. Histolgicamente se estara confirmando una:
a) Clula de Call-Exner
b) Clula de Hurthle
c) Clula de Reed-Sternberg
d) Clula de Szary
e) Celula de Hunter
406.En la exploracin clnica de las varices, la prueba de Perthes se emplea para determinar:
a) La insuficiencia de las venas comunicantes.
b) La insuficiencia del cayado de la safena.
c) La localizacin de las venas comunicantes insuficientes.
d) La existencia de lceras varicosas.
e) La permeabilidad del sistema venoso profundo.
407.Entre las causas predisponentes a un carcinoma hepatocelular se consideran las siguientes, EXCEPTO:
a) Hepatopata alcohlica.
b) Hepatitis A.
c) Hepatitis B.
d) Hemocromatosis.
e) Dficit de la alfa-1 antitripsina.
408.A qu gnero corresponde la mayora de las bacterias patgenas originarias del colon y cultivadas de las infecciones intraabdominales?
a) Pseudomona.
b) Coliformes.
c) Streptococcus.
d) Bacteroides.
e) Escherichia.
409.Un alcohlico de 40 aos presenta congelamiento de ambas extremidades inferiores. Su temperatura corporal central es de 36 C. El
tratamiento inicial mas apropiado para este paciente con lesin trmica es:
a) Simpatectoma inmediata.
b) Desbridamiento de tejidos desvitalizados.
c) Recalentamiento lento a la temperatura corporal.
d) Recalentamiento lento con calor seco.
e) Recalentamiento rpido en agua tibia.
410.Un alcohlico de 56 aos de edad presenta astenia, adinamia, prdida del vello pubiano y axilar. Su abdomen comienza a aumentar de
volumen. Laboratorio: anemia normoctica, normocrmica y disminucin de la albmina en el suero. A la acumulacin exagerada de
lquido peritoneal, se le conoce con el nombre de:
a) Hidrotrax.
b) Hidrocele
c) Ascitis.
d) Neumoperitoneo.
e) Hemoperitoneo.
REPASO Nro. 1 Pag. 51 USAMEDIC 2017
411.La causa etiolgica de la presencia de lquido peritoneal en el abdomen de este enfermo es:
a) Tuberculosis.
b) Cncer.
c) Insuficiencia cardiaca.
d) Cirrosis.
e) Enfermedad de Budd-Chiari.
412.Varn de 65 aos fue admitido en el hospital por vmito biliar grave luego de una ciruga gstrica. En qu circunstancia ocurre esto?
a) Despus de la ingestin de lquidos gaseosos.
b) Espontneamente.
c) Despus de la ingestin de comidas grasosas.
d) Despus de la ingestin de carne magra.
e) Por la tarde.
413.Varn de 26 aos, antes sano, presenta una herida penetrante por instrumento punzocortante en el 5 espacio intercostal izquierdo en
la lnea medio-clavicular. FC 140 lpm, PA sistlica 80 mmHg y respiracin 20/min. La trquea esta en la lnea media y los ruidos cardiacos
son de intensidad disminuida con ruidos respiratorios simtricos. Con su diagnostico de sospecha, el tratamiento mas apropiado es.
a) Colocacin de sonda en el hemitrax izquierdo.
b) Toracotoma en la sala de urgencias.
c) Pericardiocentesis.
d) Administracin rpida de soluciones intravenosas.
e) Intubacin y ventilacin asistida.
414.Un hombre saludable de 75 aos sangra de una lcera duodenal. El tratamiento mdico y las medidas endoscpicas fallaron en
detener el sangrado. Cul es el siguiente paso en el tratamiento?
a) Transfusin continuada de 8 U de sangre.
b) Administracin de norepinefrina.
c) Sobrecosimiento del punto de sangrado.
d) Sobrecosimiento del punto de sangrado, vagotoma y piloroplasta.
e) Ligadura de la arteria heptica.
415.Un paciente se presenta en el departamento de urgencias con ictericia obstructiva. Se lleva a cabo un colangiograma transheptico
percutneo y un drenaje biliar. Poco tiempo despus, el paciente desarrolla un sangrado UGI. Cul es la causa ms probable?
a) El paciente ha desarrollado gastritis por estrs.
b) El paciente ingiri NSAID despus del procedimiento.
c) El paciente ha desarrollado hemobilia.
d) El paciente est sangrando de las vrices esofgicas.
e) El catter ha migrado del rbol biliar al estmago.
416.Varn de 70 aos, hace 5 das presenta un dolor abdominal intenso tipo clico, intermitente y estreimiento. Hace 72 horas se agrega
nuseas y vmitos fecaloideos. Examen: distensin abdominal, timpanismo y ausencia de ruidos hidroaereos. Rx: imagen radiolcida en
pico de pjaro. Cul es el probable diagnstico?
a) Diverticulitis.
b) Intususcepcin.
c) Vlvulo de sigmoides.
d) Cncer de colon izquierdo.
e) Divertculo de Meckel.
417.Varn de 76 aos de edad presenta un bulto doloroso en la regin inguinal derecha. El dolor tiene 12 horas de evolucin y se
acompaa de vmitos en las ltimas 4 horas. En la analtica se observa leucocitosis con desviacin izquierda. En la Rx de abdomen
aparecen niveles hidroareos. Cul es su sospecha diagnostica y tratamiento?.
a) Hernia estrangulada: reduccin manual.
b) Hernia incarcerada: reduccin manual.
c) Hernia estrangulada: tratamiento quirrgico inmediato.
d) Hernia inguinal: puncin de la masa.
e) Plastrn apendicular: tratamiento antibitico.
418.Un enfermo de 54 aos de edad presenta dolor en el piso abdominal superior tras la ingesta. Es diabtico y a veces presenta heces
voluminosas, de olor rancio, que flotan en el agua. Cul es su diagnstico de presuncin?
a) Insuficiencia vascular mesentrica.
b) Celiaqua.
c) Porfiria.
d) Litiasis biliar.
e) Pancreatitis crnica.
REPASO Nro. 1 Pag. 52 USAMEDIC 2017
419.Un joven de 18 aos recibe un golpe directo en la parte alta del abdomen. Hay dolor epigstrico moderado al momento de la lesin.
Luego de 48 horas presenta vmito bilioso progresivo. A su arrivo no hay distensin abdominal; dolor leve a la palpacin en el epigastrio y
el cuadrante superior derecho. Hematocrito 36%, recuento de leucocitos 11,000 y amilasa 235 UI/100 ml. Diagnostico mas probable:
a) Pancreatitis.
b) Gastritis.
c) Colecistitis aguda.
d) Hematoma duodenal.
e) Perforacin intestinal.
420.En relacin con la valoracin preoperatoria de un paciente para ciruga programada, la consulta pre-anestsica tiene por finalidad:
a) La modificacin de la tcnica anestsica prevista.
b) La variacin del procedimiento quirrgico previsto.
c) El rellenar un cuestionario sobre los antecedentes anestsicoquirrgicos.
d) La prescripcin de un frmaco ansioltico para reducir la ansiedad generada por la prxima ciruga.
e) Establecer la presencia de enfermedades o no, buscando el riesgo de complicaciones de la cirugia prevista.
421.Uno de los siguientes NO es caracterstico de la hipotermia asociada a la vasodilatacin perifrica del bloqueo subaracnoideo:
a) Un efecto mximo en los primeros 30-60 minutos.
b) Un descenso aproximado de la temperatura central de 1-2 C.
c) Su magnitud se relaciona con la extensin del bloqueo.
d) Su magnitud se relaciona con el tipo de anestsico local que se utilice.
e) Su magnitud se relaciona con la edad del paciente.
422.Nio de 15 meses inicia tras periodo asintomtico, luego de una infeccin por adenovirus, un dolor clico intenso y paroxstico frecuente.
Llora, esta cada vez ms dbil y vomita. Masa abdominal mal definida en el cuadrante superior derecho, que aumenta de tamao en las crisis de
dolor. A las 12 h del ingreso expulsa heces con moco y sangre roja. Tacto rectal: moco sanguinolento. Esta en shock. Diagnostico mas probable:
a) Gastroenteritis viral.
b) Enterocolitis necrotizante.
c) Divertculo de Meckel.
d) Prpura de Henoch Schonlein.
e) Invaginacin intestinal.
423.Una bailarina cae al suelo y se lesiona en el transcurso de un ensayo antes de la primera funcin de un espectculo. Se daa el nervio
toracodorsal. La potencia de cul de los movimientos siguientes estar probablemente afectada?
a) Aduccin de la escpula.
b) Elevacin de la escpula.
c) Abduccin del brazo.
d) Extensin del brazo.
e) Rotacin lateral del brazo.
424.Ante un paciente que ha sufrido un traumatismo cerrado a nivel del hombro y que presenta en el plano motor una prdida parcial de la
flexin del codo y la supinacin del antebrazo, lo mas probable es que nos encontremos ante una:
a) Neuroapraxia del nervio circunflejo
b) Seccin traumtica del nervio circunflejo.
c) Neuroapraxia del nervio musculocutneo.
d) Seccin traumtica del nervio mediano.
e) Neuroapraxia del nervio radial en su salida de fascculo posterior.
425.Una fractura de codo podra causar una deformidad de mano en garra por dao del nervio:
a) Radial
b) Mediano
c) Cubital
d) Braquial
e) Braquiorradial
426.Una de las siguientes NO es caracterstica del megacolon txico:
a) Prdida de clulas ganglionares de los plexos mientricos
b) Fiebre y leucocitosis
c) Disminucin o ausencia de ruido hidroareos
d) Dilatacin radiolgica del colon transverso mayor de 6 cm
e) Gran probabilidad de perforarse el colon
REPASO Nro. 1 Pag. 53 USAMEDIC 2017
427.Qu le siguiere la asociacin de poliposis intestinal, lipomas cutneos y osteoma mandibular?
a) Sndrome de Sturge-Weber.
b) Sndrome de Turcot.
c) Sndrome de Gardner.
d) Sndrome de Cronkhite-Canada.
e) Enfermedad de Cowden.
428.La incapacidad para controlar la evacuacin rectal se debe principalmente a:
a) Traumatismo del esfnter en un parto.
b) Traumatismo en una violacin.
c) Neuropata diabtica.
d) Esclerosis mltiple.
e) Relajacin deficiente del esfnter.
429.Seale de las siguientes, cul es el sntoma primario o fundamental para detectar la posible aparicin de una ulcera por presin?
a) Picor.
b) Calambres.
c) Dolor.
d) Hormigueo.
e) Calor.
430.Tras la extraccin endoscpica de un cuerpo extrao en el esfago, un paciente presenta dolor cervical, tumefaccin, crepitacin
cervical y fiebre. Cul es su sospecha diagnstica?
a) Perforacin del esfago cervical.
b) Perforacin del esfago torcico.
c) Fstula traqueoesofgica traumtica tipo III.
d) Esofagitis grado II.
e) Sndrome de Boerhaave.
431.La obstruccin crnica de las vas biliares se asocia al desarrollo de abscesos hepticos. Cul es el organismo causal mas
frecuente?
a) Bacteroides fragillis.
b) Enterobacter.
c) Streptococcus faecalis.
d) Escherichia coli.
e) Streptococcus bovis.
432.Cul es el examen ms importante para el diagnstico de apendicitis aguda en nios?
a) Exploracin fsica.
b) Hemograma completo.
c) Protena C reactiva.
d) Placa simple de abdomen.
e) Ecografa.
433.Complicacin mas frecuente del divertculo de Meckel:
a) La inflamacin sin peritonitis
b) La perforacin con peritonitis
c) La hemorragia
d) La obstruccin intestinal
e) La malignizacin
434.Causa mas frecuente de rectorragia en el nio:
a) Plipo intestinal
b) Fisura anal
c) Rectocolitis ulcerosa
d) Alergia
e) Absceso anal
435.Varn de 67 aos, cirrtico, padece fiebre de 15 das. Hepatomegalia dura de 4 traveses de dedo y una esplenomegalia de 2 traveses.
Analtica: Hto 51% y glucemia en ayunas de 54 mg/dl. Su diagnostico probable:
a) Hepatocarcinoma.
b) Pancreatitis crnica.
c) Descompensacin de la cirrosis.
d) Porfiria eritropoytica.
e) Sndrome de Budd-Chiari.
REPASO Nro. 1 Pag. 54 USAMEDIC 2017
436.Un hombre de 25 aos se cay de su bicicleta y choc contra una pared de concreto en su lado izquierdo. Un examen de ultrasonido
mostr lquido libre en el abdomen. Una TC confirm una herida en el bazo de grado III. La contraindicacin ms importante para un
manejo no quirrgico de la herida en el bazo es:
a) Inestabilidad hemodinmica.
b) Sangrado activo en CT.
c) Paciente adulto.
d) Falta de disponibilidad de sangre para transfusin.
e) Lesiones relacionadas extensas.
437.En la pancreatitis aguda, Cul de los siguientes exmenes es de valor diagnstico precoz?
a) Amilasa.
b) Transaminasa.
c) Lipasa.
d) Frmula y recuento leucocitario.
e) Protena e reactiva.
438.Un paciente presenta aumento de volumen escrotal, pesadez y cierto dolorimiento en el testculo izquierdo que aumenta con la
bipedestacin. Qu sospechara?
a) Hidrocele.
b) Varicocele.
c) Seminoma.
d) Epididimitis.
e) Torsin de hitdide.
439.Acude a urgencias un varn de 80 aos de edad, con un sndrome miccional y clnica de obstruccin uretral. Decides practicarle un
tacto rectal para descartar una prostatitis aguda. Qu textura tendra la prstata si se tratase de este tipo de infeccin?
a) Dureza ptrea.
b) Dura y elstica.
c) Tensa, empastada y dolorosa.
d) Dureza asimtrica.
e) Como cartlago nasal.
440.En una sinovitis purulenta, una puncin articular mostrar lquido con elevados niveles de:
a) Glucosa.
b) Complemento.
c) Polimorfonucleares.
d) Sangre.
e) Viscosidad.
441.Un paciente que ha sufrido una Fx del calcneo de un mes de evolucin comienza a notar dolor de carcter sordo en torno a la regin del
malolo tibial, con parestesias que se irradian claramente hacia la planta del pie. El diagnstico probable en este caso es, entre los siguientes:
a) Sindroma del seno del tarso.
b) Compresin de la vaina del musculo tibial posterior secundaria a la fractura.
c) Sndrome del tnel o canal tarsiano.
d) Subluxacin subastragalina a raz de la fractura.
e) Sndrome del canal de Guyn.
442.El diagnstico instrumental de un sndrome compartimental se realiza:
a) Estudio electrofisiolgico.
b) Midiendo la presin intracompartimental.
c) Gammagrafa.
d) Biopsia.
e) Resonancia nuclear magntica.
443.Qu compartimiento muscular se afecta fundamentalmente en la contractura isqumica de Volkmann en el antebrazo?
a) Posterior.
b) Lateral.
c) Anterior profundo.
d) Anterior superficial.
e) Lateral profundo.
REPASO Nro. 1 Pag. 55 USAMEDIC 2017
444.Un aneurisma sacular en la aorta abdominal es mas propenso a romperse que una en la aorta normal porque:
a) Se ejerce mas tensin en la pared del aneurisma
b) Hay menos presin en el aneurisma
c) Hay menos turbulencia en el aneurisma
d) Hay mas tensin ejercida y menor turbulencia en el aneurisma
e) Hay mas presin y menos turbulencia en el aneurisma
445.Un paciente de 40 aos acude al servicio de urgencias por cefalea y alteraciones visuales no bien definidas. En la TC craneal se
objetiva un gran tumor hipofisario Cual de los siguientes datos esperara encontrar en la exploracin?
a) Hemianopsia homnima izquierda.
b) Hemianopsia bitemporal.
c) Hemianopsia binasal.
d) Cuadrantanopsia nasal bilateral.
e) Nistagmo horizontal bidireccional.
446.El signo aislado ms importante para sospechar una contractura de Volkmann es:
a) Cianosis de los dedos.
b) Palidez de los dedos.
c) Obliteracin del pulso radial.
d) Dolor.
e) Parlisis de los msculos flexores del antebrazo.
447.La imagen en punta de lpiz en un examen con papilla baritada le har sospechar:
a) Espasmo esofgico primario.
b) Espasmo esofgico secundario.
c) Acalasia.
d) Tumor benigno.
e) Tumor maligno.
448.En la poliposis colnica familiar, el tratamiento de eleccin es:
a) Proctocolectoma total.
b) Hemicolectoma derecha y controles endoscpicos peridicos del colon izquierdo.
c) Radioterapia y quimioterapia combinadas.
d) Expectante, con control endoscpico cada seis meses.
e) Todas las anteriores pueden ser actitudes teraputicas correctas dependiendo de cada caso.
449.Un hombre de 40 aos de edad que pesa 65 kg est bajo observacin en la UCI. Veinticuatro horas despus de la operacin, desarrolla
convulsiones. Su sodio srico es de 118 meq/L. Cul de las siguientes opciones se incluye en un tratamiento apropiado?
a) Administracin de solucin salina normal (0.9%).
b) Administracin de solucin salina hipertnica (3%).
c) Hemodilisis de emergencia.
d) Administracin de vasopresina.
e) Administracin de furosemida, 40 mg, IV.
450.El sitio mas comn de tumores carcinoides gastrointestinales es:
a) Intestino delgado
b) Colon
c) Apndice
d) Esfago
e) Estomago

AREA DE GINECOLOGIA Y OBSTETRICIA

451.Una mujer de 20 aos de edad (Gesta 1) acaba de tener su parto. Despus de la salida de la placenta, se observa una superficie roja,
spera, en la entrada vaginal. En forma simultnea, la enfermera comenta que la paciente est plida y su PA es de 70/40. La hemorragia
externa es normal en cantidad. Cul de los siguientes es el diagnstico ms probable?
a) Quiste ovrico.
b) Rotura uterina.
c) Segundo gemelo.
d) Inversin uterina.
e) Rotura vaginal.
REPASO Nro. 1 Pag. 56 USAMEDIC 2017
452.Cul de los siguientes es el tratamiento para la paciente anterior?
a) Expulsin del producto.
b) Laparotoma exploradora.
c) Histerectoma inmediata.
d) Recolocacin inmediata del fondo.
e) Transfusin masiva de sangre.
453.Al realizar una laparotoma en una mujer de 24 aos en la que se sospecha un embarazo ectpico, y que desea embarazarse, el mdico
encuentra una rotura ectpica tubaria izquierda con cerca de 400 ml de sangre en la cavidad peritoneal. La otra trompa parece normal y no
hay participacin ovrica. Cul de los siguientes es el tratamiento aceptado?
a) Salpingectoma bilateral.
b) Salpingectoma o salpingostoma izquierda.
c) Salpingooforectoma bilateral (SOB).
d) Histerectoma y salpingectoma izquierda.
e) Salpingectoma derecha.
454.Una mujer casada de 35 aos de edad (Gesta 4, para 3, aborto 0), que ahora est aproximadamente en la semana 36 de gestacin,
presenta hemorragia vaginal copiosa, sin dolor, dos horas antes de la admisin al hospital. A la exploracin fsica, el tero es blando y no
doloroso. La FCF es de 140 y regular, el vrtice est flotando y no hay evidencia de hemorragia o signos de membranas rotas. Los signos
vitales maternos son estables. Cul de los siguientes constituye el diagnstico ms probable?
a) Carcinoma cervical.
b) Placenta previa.
c) Desprendimiento prematuro de placenta normoinserta.
d) Vasos previos.
e) Hematuria.
455.Cul sera el siguiente paso en el tratamiento de la paciente anterior?
a) Amniocentesis para estudios de madurez pulmonar.
b) Ecografia por localizacin placentaria.
c) Vigilancia contina en el hospital.
d) Exploracin vaginal muy superficial de los fondos de saco con cuidado para no atravesar el orificio cervical.
e) Exploracin con instrumento (con espejo) para visualizar el cuello uterino.
456.Una mujer de 35 aos se queja de dismenorrea que aumenta y de dolor plvico. No se ha embarazo a pesar de tres aos de relaciones
sexuales sin proteccin. Su exploracin plvica muestran sensibilidad y ndulos sobre sus ligamentos uterosacros y un quiste ovrico de
4 cm. El diagnstico ms obvio es:
a) Adenomiosis.
b) Sndrome de congestin plvica.
c) Embarazo ectpico crnico.
d) Endometriosis.
e) Enfermedad inflamatoria plvica crnica (PID).
457.Al observar una biopsia cervical, se nota una atipia de clulas escamosas. Se extiende desde la capa basal hasta poco ms de la mitad
del grosor del epitelio. Ms all de este nivel, la maduracin es evidente. No hay invasin del estroma. El diagnstico correcto basado en
estos datos de biopsia es:
a) Adenocarcinoma.
b) Hiperplasia microglandular.
c) Displasia moderada (CIN II).
d) CIS.
e) Carcinoma celular escamoso invasivo.
458.Una mujer de 20 aos de edad en la semana 36 de embarazo acude a su primera evaluacin. Se le diagnostica infeccin por Chlamydia
trachomatis de cuello uterino. Cul es la complicacin de mayor riesgo para el beb durante el parto?
a) Ictericia.
b) Hidrocefalia.
c) Triada de Hutchinson.
d) Conjuntivitis.
e) Sordera neurosensorial.
REPASO Nro. 1 Pag. 57 USAMEDIC 2017
459.Una de las siguientes NO parece aumentar el riesgo de padecer cancer de crvix:
a) Obesidad.
b) Promiscuidad.
c) Tabaquismo.
d) Promiscuidad del compaero sexual.
e) Inicio precoz de las relaciones sexuales.
460.Mujer de 38 aos, embarazada por 4 vez, presenta secrecin sanguinolenta por el pezn derecho. En la exploracin fsica no se
evidencia la existencia de ninguna masa en ninguno de los dos senos. Diagnostico mas probable:
a) Mastitis crnica.
b) Fibroadenoma.
c) Necrosis de la grasa subcutnea.
d) Papiloma intraductal.
e) Mastitis plasmocelular.
461.Una multigrvida de 38 aos se queja de prdida indolora de orina, que comienza inmediatamente cuando tose, se re, levanta algo o
se estira. El cese inmediato de la actividad detiene la prdida de la orina despus de slo unas gotas. Qu sugieren estos sntomas?
a) Fstula.
b) Incontinencia de esfuerzo.
c) Incontinencia de urgencia.
d) Divertculo uretral.
e) ITU.
462.Ante una esterilidad femenina por obstruccin tubrica bilateral, los mejores resultados se obtienen mediante:
a) Inseminacin artificial.
b) Microciruga tubrica.
c) Fertilizacin in vitro.
d) Insuflacin tubrica.
e) Tratamiento con gonadotropinas.
463.Cul es el mecanismo por el cual la T de cobre acta como anticonceptivo?
a) Inhibe la contraccin del msculo liso uterino.
b) Disminuye la formacin de prostaglandinas.
c) Bloquea la formacin de un folculo dominante.
d) Produccin de reaccin inflamatoria en la cavidad uterina.
e) Produce espesamiento del moco cervical.
464.Cul de las siguientes alternativas constituye un factor de riesgo para cncer de crvix?
a) Baja paridad.
b) Alto nivel socioeconmico.
c) Compaero sexual nico.
d) Inicio tardo de relaciones sexuales.
e) Tabaquismo.
465.Mujer de 56 aos, menopausia hace 8 aos, actualmente presenta sangrado de origen uterino. Procedimiento diagnstico a seguir:
a) Ecografia intravaginal.
b) Legrado uterino bipsico.
c) Laparoscopa.
d) Hstero sonografa.
e) Hstero salpingografa.
466.Cul es el germen productor con mayor frecuencia de una endometritis puerperal?
a) Staphylococcus aureus.
b) Chlamydias.
c) Streptococcus.
d) Escherichia coli.
e) Gonococo.
467.Una mujer de 25 aos de edad tiene condilomas vulvares. Cul de las siguientes es correcta?
a) La causa es el virus del herpes tipo 2.
b) Es una enfermedad de transmisin sexual.
c) La biopsia descartar carcinoma verrugoso.
d) Las lesiones por lo comn ceden y desaparecen en forma espontnea.
e) Con frecuencia ocurre hiperplasia endometrial.
REPASO Nro. 1 Pag. 58 USAMEDIC 2017
468.La bacteriuria asintomtica durante la gestacin es mas frecuentemente causada por uno de los siguientes grmenes:
a) Streptococcus agalactiae.
b) Escherichia coli.
c) Gardnerella vaginalis.
d) Enterococo.
e) Schistosoma haematobium.
469.Una secrecin vaginal de color gris o verde amarillento, en ocasiones espumosa, profusa, que puede ser de mal olor, causa prurito,
con enrojecimiento del vestbulo y los labios menores, con dolor durante la miccin, sugiere un cuadro de:
a) Vaginitis por candida.
b) Vaginitis atrfica.
c) Vaginitis por trichomonas.
d) Infeccin por chlamydia trachomatis.
e) Vaginosis bacteriana.
470.Una gestante de 12 semanas llega por metrorragia y dolor abdominal de varias horas de evolucin. Se nota un cuello uterino cerrado y
en la ecografia un embrin con latido cardiaco. Actitud teraputica mas correcta:
a) Reposo relativo.
b) Legrado uterino.
c) Oxitcicos.
d) Prostaglandinas.
e) Reposo y progesterona vaginal.
471.Seale el tumor benigno mas frecuente del tracto genital femenino:
a) Tumor de Brenner de ovario.
b) Cistoadenoma mucinoso de ovario.
c) Ginandroblastoma.
d) Cistoadenoma seroso de ovario.
e) Mioma uterino.
472.En el sindrome de ovarios poliquisticos se emplea uno de los siguientes tratamientos para intentar la gestacin:
a) Danazol.
b) Reseccin en cua de ambos ovarios.
c) Gonadotropinas.
d) Clomifeno.
e) Prednisona y megestrol.
473.En una evaluacin ecogrfica ginecolgica, de un tumor ovrico de 3 x 4 cm de borde regular, hipoecognicos, homogneos,
avascular y de aspecto vidrio esmerilado. Cul es la tumoracin ovrica ms probable?
a) Quiste mucinoso.
b) Cistoadenoma.
c) Absceso.
d) Endometrioma.
e) Mioma.
474.Por lo general, el cncer mamario se disemina a:
a) Cerebro
b) Pulmn
c) Ganglios linfticos
d) Huesos
e) Hgado
475.Una de las siguientes variedades de tumor presenta cuerpos de Call-Exner. De cul se trata?
a) Tumor de Brenner.
b) Cistoadenocarcinoma mucinoso.
c) Carcinoma endometroide.
d) Coriocarcinoma.
e) Tumor de los cordones sexuales.
476.Qu nos indicara la presencia del marcador CA 15-3 en un carcinoma de mama?
a) Metstasis.
b) Tamao mayor a 3 cm.
c) Carcinoma lobulillar.
d) Sarcoma.
e) Afectacin de ganglios axilares.
REPASO Nro. 1 Pag. 59 USAMEDIC 2017
477.Qu postura debe seguir en una mujer que quiere quedar embarazada y en que hemos descubierto serologa antitoxoplasma
positiva?
a) Desaconsejar el embarazo por peligro de toxoplasmosis congnita para el hijo.
b) No hacer nada, no existe peligro de que el hijo padezca toxoplasmosis congnita.
c) Dar tratamiento con espiramicina durante un mes.
d) Esperar hasta que los ttulos serolgicos disminuyan.
e) Practicar inmunofluorescencia indirecta IgM especfica.
478.Mujer de 26 aos, G2, P1, embarazo de 20 semanas, llega por atencin prenatal. Fondo uterino corresponde a 24 semanas y no se halla
la FCF con Doppler. Se hace ultrasonido para confirmar la falta de FCF y de movimientos o respiracin fetales. Su ltimo embarazo se
complic con preeclampsia grave a las 34 semanas, obligando al trmino del embarazo con producto prematuro. Destaca su historia: asma
leve. Cuadro de sangrado hace 4 semanas, sin clicos y no elimin cogulos o restos de tejido por la vagina. Diagnostico mas descriptivo:
a) Amenaza de aborto.
b) Muerte fetal.
c) Aborto incompleto.
d) Aborto espontneo.
e) Aborto retenido.
479.Una paciente ha estado en trabajo de parto durante cerca de 12 horas y tiene una temperatura de 38.4C. El pulso es de 110 lpm. La
frecuencia cardiaca fetal es de 180 con buena variabilidad. El trabajo de parto progresa de manera adecuada. Cul de las opciones es el
paso siguiente en el tratamiento?
a) Hidratacin intravenosa (IV).
b) Antibiticos.
c) Oxitocina.
d) Muestreo del cuero cabelludo fetal.
e) Cesrea.
480.Se revisa a una primigrvida de 22 aos de edad en trabajo de parto activo; ste tiene una duracin de 14 horas. Presenta una
dilatacin de 8 cm y est en el plano 0. Conforme la cabeza fetal desciende, la forma cambia. Cul de las siguientes es la causa ms
probable?
a) Cefalohematoma.
b) Amoldamiento.
c) Hematomas subdurales.
d) Hidrocefalia.
e) Caput succedaneum.
481.Una de las siguientes tiene una alta incidencia de prolapso de cordn:
a) OP (occipito pubiana)
b) Presentacin de nalgas incompleta.
c) Mentn posterior.
d) Presentacin de nalgas completas.
e) Mentn anterior.
482.Se llev a cabo un trabajo de parto rpido con una presentacin de vrtice y el lactante est coronando. Nos referimos a una de las
siguientes maniobras:
a) Maniobra de McRoberts.
b) Maniobra de Mauriceau.
c) Maniobra de Ritgen.
d) Maniobras de Leopold.
e) Maniobra de Ferguson.
483.En una de las siguientes etapas se comienza con la expulsin del producto:
a) Primer periodo del trabajo de parto.
b) Segundo periodo del trabajo de parto.
c) Tercer periodo del trabajo de parto.
d) Cuarto periodo del trabajo de parto.
e) Periodo posparto.
REPASO Nro. 1 Pag. 60 USAMEDIC 2017
484.Primigrvida de 21 aos en la semana 39 de gestacin presenta trabajo de parto y parto con molestias de contracciones uterinas
desde las 5 am de ese da. Fue revisada en una visita de rutina, en su clnica, a las 3 pm y el cuello uterino se encontr con 2 cm de
dilatacin, borramiento 50%, en la posicin media y moderado en consistencia, con el vrtice fetal en el plano 0. Al reexaminarla en la sala
de trabajo de parto y parto a las 7 pm, no muestra cambios cervicales significativos. FCF normal. Puntuacin de Bishop de la embarazada:
a) 8, cuello inmaduro.
b) 4, cuello inmaduro.
c) 2, cuello maduro.
d) 6, cuello maduro.
e) 9, cuello maduro.
485.A las 39 semanas de gestacin, se palpa a un feto que est en presentacin plvica segn la informacin obtenida mediante las
maniobras de Leopold. La presentacin est muy abajo en la pelvis, y el tero est irritable. La pelvimetra se encuentra dentro de los
lmites normales y el peso estimado fue de 3.4 kg. Cul de los siguientes debe realizarse?
a) Cesrea.
b) Versin ceflica externa.
c) Versin podlica interna.
d) Induccin con oxitocina.
e) Tratamiento expectante.
486.Una paciente presenta parto prematuro espontneo en la semana 28 de gestacin. Durante el parto ceflico cul de los siguientes es
indispensable llevar a cabo?
a) Se recomienda anestesia epidural para controlar el parto.
b) Realizar una episiotoma.
c) Utilizar frceps profilctico.
d) Utilizar extraccin al vaco.
e) Permitir el parto vaginal espontneo.
487.Una mujer de 19 aos de edad en la semana 36 de gestacin con una PA de 150/100, edema 2+ y proteinuria 2+, sin otros sntomas.
Diagnostico mas probable:
a) Preeclampsia.
b) Preeclampsia grave.
c) Enfermedad hipertensiva crnica.
d) Eclampsia.
e) Enfermedad renal crnica.
488.Con los siguientes datos ecogrficos, tras valoracin de RCI, se halla DBP muy disminuido, relacin AC/AA = 1. Ud. lo clasifica como:
a) Intrnseco-armnico.
b) Distrfico.
c) Extrnseco-disarmnico.
d) Extrnseco-semiarmnico.
e) Malnutricin.
489.Primigesta de 32 aos que acude a urgencias refiriendo sangrado genital y prueba de embarazo en farmacia positiva. Se realiza
exploracin ginecolgica y los hallazgos ecogrficos y analticos sugieren el diagnstico de mola hidatidiforme. Tratamiento de eleccin:
a) Quimioterapia con Metotrexate.
b) Legrado uterino con legra de Recamier y pinza Winter.
c) Legrado por aspiracin.
d) Histerectoma simple (conservando ovarios).
e) Histerectoma radical.
490.Cul de las siguientes es la causa ms frecuente de coagulacin intravascular diseminada durante el embarazo?
a) Diabetes gestacional.
b) Placenta previa.
c) Hipertensin arterial crnica asociada al embarazo.
d) Desprendimiento prematuro de placenta.
e) Aborto incompleto.
491.El tipo de tcnica analgsica a realizar en el parto vaginal, depende de las siguientes situaciones, EXCEPTO:
a) Deseo de la mujer.
b) Estado medico de la mujer.
c) Edad de la mujer.
d) Progreso del parto.
e) Recursos disponibles.
REPASO Nro. 1 Pag. 61 USAMEDIC 2017
492.Paciente femenino de 25 aos de edad con embarazo a trmino, en fase latente de trabajo de parto. Sin datos de sufrimiento fetal.
Segn la norma oficial, que acciones NO se deben realizar rutinariamente?
a) Rasurado del vello pbico.
b) Induccin y conduccin.
c) Amniorrexis.
d) Episiotoma.
e) Todas las anteriores.
493.Las 2 patologas mas frecuentes que causan hemorragia en la ltima etapa del embarazo son desprendimiento prematuro de placenta
(DPP) y placenta previa (PP), esto nos obliga a un diagnostico diferencial. Seale la FALSA:
a) La cesrea siempre es la solucin para atacar ambos cuadros.
b) En DPP suelen existir contracciones uterinas frecuentes.
c) Es difcil la palpacin de partes fetales en DPP.
d) La PP raramente se asocia a preeclampsia.
e) En la PP la hemorragia es indolora y roja.
494.Cul de las siguientes etiologas de amenorrea cursa con gonadotropinas elevadas?
a) Sndrome de Turner.
b) Craneofaringioma.
c) Meningitis.
d) Sindrome de Sheehan.
e) Sindrome de Kallman.
495.Ante una posible infeccin transplacentaria NO se planteara uno de los microorganismos:
a) Toxoplasma gondii.
b) Virus citomeglico.
c) Treponema pallidum.
d) Chlamydia trachomatis.
e) Virus de la rubola.
496.El seudohermafroditismo femenino (hiperplasia suprarrenal congenita) se refiere a individuos que tienen ciertas caractersticas:
a) Ovarios, cariotipo XX y grados variables de masculinizacin.
b) Testculos, cariotipo XY y grados variables de falta de masculinizacin.
c) Ovarios, cariotipo XY y grados variables de falta de masculinizacin.
d) Testculos, cariotipo XX y masculinizacin grave.
e) Ovarios y tejido testicular.
497.Lo mas importante en el comportamiento del herpes genital es que:
a) Se trasmite por el HV-1 y el HV-2, preferentemente
b) Se acompaa de ppulas, vesculas y pstulas
c) Provoca ardor y prurito
d) Tiene tendencia a la recidiva
e) Se trata con la administracin de aciclovir
498.Mujer de 60 aos. En la revisin rutinaria se detecta una masa mvil en pelvis. El estudio ecogrfico pone de manifiesto, como nico
hallazgo, una lesin qustica de 70 mm, con proliferaciones papilares internas y dependientes de ovario derecho. El marcador tumoral Ca
125 es de 70 U/ml (normal <35 U/ml). Cul es la conducta diagnstico/teraputica adecuada?
a) Control evolutivo mediante seriacin ecogrfica y del nivel del marcador.
b) Completar el estudio mediante marcadores de estirpe germinal (alfafetoprotena/gonadotropina corinica).
c) Estudio citolgico de la lesin mediante puncin aspiracin con aguja fina.
d) Tratamiento quirrgico: anexectoma y estudio intraoperatorio de la lesin.
e) Tratamiento quirrgico: quistectoma y estudio intraoperatorio de la lesin.
499.Una paciente de 19 aos, nuligesta, acude a la consulta por irritabilidad, inestabilidad emocional, cefaleas, sensibilidad y distensin
mamaria. La sintomatologa se inicia una semana antes de la menstruacin y desaparece tras presentarse la regla. Una vez comprobada la
relacin con el ciclo menstrual mediante un diario de sntomas, decidimos pautar tratamiento pues la paciente refiere que la
sintomatologa interfiere con su trabajo y sus relaciones sociales. Qu aconsejamos como primera eleccin teraputica?
a) Aumento de ingesta de cafena y azcares.
b) Progesterona 200-300 mg/da.
c) Evitar ejercicio fsico y tomar suplemento vitamina B12
d) Alprazolam 0.25 mg/8 horas
e) ACO ms un diurtico suave.
REPASO Nro. 1 Pag. 62 USAMEDIC 2017
500.Una mujer de 75 aos presenta lesiones blancas lisas en la vulva y ano con prurito. De entre las siguientes, Cul es su actitud?
a) Biopsia con la prueba de Richard Collins.
b) Corticoides tpicos.
c) Vulvoscopia (con tincin de ac. actico).
d) Amputacin abdominoperineal.
e) Clindamicina tpica.
501.Una mujer llega por amenorrea primaria, agenesia de vagina y tero. Su cariotipo es 46,XX. Cul ser el diagnstico mas probable?
a) Sindrome de Rokitansky.
b) Sindrome de Kallman.
c) Sindrome de Turner.
d) Sindrome de Down.
e) Sindrome de Albright.
502.Una mujer presenta una secrecin a travs del pezn izquierdo, de caractersticas serohemorrgicas. No hay tumor a la palpacin.
Sabiendo que se trata del segundo o tercer tumor benigno de mama en frecuencia, se sospecha de:
a) Mastopata fibroqustica.
b) Carcinoma in situ.
c) Tumor Phyllodes.
d) Papiloma intraductal.
e) Fibroadenoma.
503.Respecto al abruptio placentae (DPP) es FALSA:
a) Se produce a partir de la 20 semana.
b) Los estados hipertensivos del embarazo podran ser factores patognicos desencadenantes.
c) Se produce un hematoma retroplacentario.
d) El tero se encuentra relajado y atnico.
e) El shock hipovolmico y la CID son dos posibles complicaciones
504.En las mujeres existe un control muy importante en el ciclo genital. Hay interaccin entre hormonas y rganos como hipotlamo,
hipfisis, ovario, endometrio entre otros. Cul de los siguientes NO es cierto sobre el ciclo genital femenino?
a) La FSH se produce principalmente durante la fase profilerativa.
b) Si no hay elevacin de LH no se produce la ovulacin.
c) El cuerpo lteo produce progesterona.
d) Los pulsos rpidos de la GnRH estimulan la FSH.
e) El mittelschmerz generalmente se produce a la mitad del ciclo menstrual.
505.Una hernia verdadera en el espacio potencial de la vagina, se denomina:
a) Cistocele.
b) Enterocele.
c) Hernia femoral.
d) Hernia incisional.
e) Rectocele.
506.A qu nivel se sita el tercer plano de Hodge?
a) Coxis.
b) Borde superior de la snfisis pbica.
c) Estrecho superior de la pelvis.
d) Estrecho inferior de la pelvis.
e) Espina citica.
507.Mujer de 45 aos presenta un bulto indoloro en la mama izquierda. Exploracin: retraccin del pezn y adenopata axilar. Palpacin:
ndulo de consistencia leosa y reducida movilidad. De su historia clnica, son factores de riesgo para padecer cncer de mama, EXCEPTO:
a) Menarquia precoz.
b) Menopausia tarda.
c) Nuliparidad.
d) Elevada ingesta de alcohol.
e) Edad temprana del primer embarazo.
508.Dnde metastatiza con mayor frecuencia el cncer de mama?
a) Hgado.
b) Pulmn.
c) Hueso.
d) Cerebro.
e) Ojo.
REPASO Nro. 1 Pag. 63 USAMEDIC 2017
509.Algunos de los siguientes NO constituyen criterios mayores para el diagnstico de la enfermedad inflamatoria plvica:
a) Leucocitosis mayor a 10500 / mm3.
b) Fiebre mayor de 38 C.
c) Cervix doloroso a la movilizacion.
d) Dolor anexial en la exploracin abdominal.
e) Las respuestas ciertas son a y b.
510.Qu diagnostico de sospecha establecera ante una mujer que presenta agenesia total de vagina, tero rudimentario, malformaciones
renales, fenotipo femenino, 46 XX?
a) Sindrome de Swyer.
b) Sindrome de Rosle.
c) Sindrome de Rokitansky.
d) Sindrome de Turner.
e) Sindrome de Kallman.
511.Sobre los quistes de Gartner uno de los siguientes es verdadero:
a) Son remanentes del conducto de Wolff
b) Estn localizados en la vulva
c) Estn ubicados en el tercio inferior de la vagina
d) Estn ubicados en el tercio medio de los ovarios
e) Estn localizados en el tero
512.Uno de los siguientes frmacos, administrado durante el embarazo NO se asocia a efectos indeseables en el neonato:
a) Clorpropamida
b) Salicilatos
c) Indometacina
d) Nitrazepam
e) Eritromicina
513.Sobre las indicaciones de cesrea, una de las siguientes es FALSA:
a) Est indicada en pelvis estrechas de 4 grado y un conjugado verdadero de 6,5 cm o menor.
b) El abruptio de placenta con feto vivo constituye indicacin urgente de cesrea.
c) Entre las indicaciones urgentes se encuentra: placenta previa que sangra abundantemente.
d) La situacin transversa constituye indicacin de cesrea.
e) Las presentaciones de nalgas en primparas no son indicacin de cesrea.
514.Uno de los siguientes es un hallazgo colposcpico benigno:
a) Zona de transformacin.
b) Leucoplasia.
c) Base.
d) Zona de trasformacin atpica.
e) Mosaico.
515.Una mujer de 58 aos de edad acude a consulta por un prurito vulvar. En una exploracin plvica se observa piel atrfica y delgada
con coloracin blancuzca sobre toda la vulva. De los siguientes diagnsticos, el ms seguro es:
a) Carcinoma vulvar.
b) Neoplasia intraepitelial vulvar.
c) Hiperqueratosis.
d) Vulvitis atrfica.
e) Liquen escleroso.
516.Mujer de 19 aos padece amenorrea primaria. Caracteres sexuales y genitales externos femeninos. Vagina corta, sin tero ni trompas.
Ecografa: 2 tumoraciones que se valoran como testculos; el estudio cromosmico confirma sexo masculino. Diagnostico mas probable:
a) Sindrome de disgenesia gonadal pura.
b) Sindrome de Turner.
c) Sindrome de Morris.
d) Sindrome de pseudohermafroditismo por defecto de la gnesis de andrgenos.
e) Disgenesia gonadal con feminizacin.
REPASO Nro. 1 Pag. 64 USAMEDIC 2017
517.Los resultados de un defecto de las uniones musculofaciales del elevador posterior del ano da como resultado un prolapso genital en
las mujeres:
a) Cistocele.
b) Enterocele.
c) Hernia inguinal indirecta.
d) Rectocele.
e) Uretrocele.
518.Multpara con 33 semanas de gestacin, llega por metrorragia de sangre roja, aparicin brusca, indolora, progresiva, que cede con el
reposo. Abdomen blando y depresible, taquicardia, hipotensin y palidez. Hemoglobina y hematocrito un poco por debajo de la
normalidad. Tras su ingreso, cul sera de las siguientes una conducta incorrecta?
a) Eventual reposicin de las prdidas sanguneas si es necesario.
b) Practicar tacto vaginal para constatar la procedencia de la hemorragia.
c) Constatacin de la vitalidad y el estado fetal.
d) Si el feto esta gravemente afectado, cesrea urgente.
e) Si la placenta es lateral o marginal puede permitirse la va vaginal si no existen complicaciones.
519.Una primpara a trmino tiene la bolsa rota y el feto esta vivo con presentacin de hombro. Qu conducta obsttrica seguira?
a) Cesrea.
b) Monitorizar.
c) Expectante.
d) Versin interna.
e) Versin externa.
520.Conociendo la fisiologa materna normal, cul de los siguientes podra utilizarse si una paciente de 38 semanas se desmaya mientras
est en posicin supina en la mesa de exploracin?
a) Sales de amoniaco.
b) Poner a la paciente sobre su costado.
c) Oxgeno mediante una mascarilla.
d) Frmacos intravenosos (IV) para aumentar la presin arterial.
e) Solucin salina IV.
521.Cul es el mtodo mas fiable para evaluar la edad gestacional?
a) La fecha de la ltima regla.
b) La comprobacin del crecimiento uterino.
c) La fecha de aparicin de los primeros movimientos fetales.
d) La ecografa del primer trimestre.
e) El conocimiento de la fecha de fecundacin.
522.Las siguientes patologas pueden ser causa de una telorrea unilateral y uniorificial, EXCEPTO:
a) Ectasia ductal.
b) Carcinoma ductal.
c) Papiloma intraductal.
d) Enfermedad fibroqustica.
e) Prolactinoma.
523.Cul de los siguientes est determinado por la relacin de las partes fetales entre s?
a) Presentacin.
b) Situacin.
c) Actitud.
d) Posicin.
e) Intencin.
524.Tratamiento de eleccin en una mujer que ha sufrido una rotura uterina durante un parto vaginal:
a) Transfusin de varios concentrados de hemates en funcin de la cuanta del sangrado vaginal.
b) Laparotoma.
c) Uso de frceps.
d) Uso de ventosa.
e) Antibiticos va i.v. y expansores del plasma.
REPASO Nro. 1 Pag. 65 USAMEDIC 2017
525.Mujer de 27 aos llega en la semana 20 de gestacin por una sensacin de peso en la vagina y flujo acuoso, sin metrorragias ni dolor.
Historia de prdida fetal en la gestacin anterior durante la semana 19. Exploracin fsica: cuello dilatado a 7 cm, siendo prominente la
bolsa amnitica. El feto fue expulsado espontneamente a los 30 min. del ingreso, falleciendo tras el nacimiento. Diagnostico probable:
a) Presencia de anticuerpos antifosfolpidos.
b) Incompetencia cervical.
c) Infeccin intratero.
d) Presencia de tabique intrauterino.
e) Cromosomopata fetal.
526.Cul de los siguientes debe prescribirse para la mujer embarazada promedio?
a) Un incremento no mayor de 6.8 a 9 kg durante el embarazo.
b) Complemento de hierro, adems de las vitaminas prenatales.
c) Continuar ejercicio moderado.
d) Baos calientes en tina con jacuzzi para relajarse.
e) Duchas vaginales de agua con vinagre en el tercer trimestre.
527.Cul de los siguientes corresponde a una unidad Montevideo?
a) El nmero de contracciones en 10 minutos.
b) El nmero de contracciones por minuto que dure su intensidad.
c) La intensidad de cualquiera de las contracciones que mide el tiempo que toma para que ocurran.
d) El nmero de contracciones de 50 mmHg en 10 minutos.
e) El nmero de contracciones en 10 minutos que dure el promedio de su intensidad.
528.Indique el tratamiento de eleccin de una presin arterial de 176/110 en una embarazada:
a) Nitroprusiato.
b) Captopril.
c) Diazxido.
d) Hidralazina.
e) Furosemida.
529.Como causas de incontinencia urinaria, la uretritis y la vaginitis atrfica se pueden reconocer por:
a) Telangiectasia
b) Petequias
c) Inflamacin uretral
d) Eritema
e) Fragilidad de la mucosa vaginal
530.Una mujer de 18 aos de edad se queja de dolor y aumento del volumen de la mama derecha. Est esperando el periodo menstrual.
Este cuadro puede ser sugestivo de:
a) Mastitis crnica
b) Adenocarcinoma
c) Enfermedad fibroqustica
d) Papiloma intraductal
e) Enfermedad de Paget
531.Una mujer de 25 aos de edad presenta metrorragias y contracciones importantes en su 20 semana de gestacion. Exploracin:
dilatacin del cuello uterino sin expulsin del huevo. Qu sospecha?
a) Aborto diferido.
b) Aborto consumado.
c) Aborto habitual.
d) Amenaza de aborto.
e) Aborto en curso.
532.Una mujer presenta oligomenorrea, galactorrea e hirsutismo, usted descartar:
a) Dficit completo de la 21-OH-asa.
b) Dficit parcial de la 21-OH-asa.
c) Prolactinoma.
d) Ovario polimicroqustico.
e) Tumor secretor de andrgenos.
REPASO Nro. 1 Pag. 66 USAMEDIC 2017
533.Cul de los siguientes cambios endometriales se observara en caso de que se realice una biopsia de endometrio a una paciente al
final de la fase proliferativa del ciclo menstrual?
a) Secreciones glandulares de localizacin apical.
b) Hipertrofia.
c) Crecimiento de las arterias espirales.
d) Acumulacin de glucgeno.
e) Degeneracin de estructuras glandulares.
534.Las bacterias pueden cultivarse a partir de una gran parte de las cavidades endometriales dos a tres das posparto en pacientes que
son asintomticas. Cul de los siguientes es el microorganismo anaerbico que se encuentra con mayor frecuencia?
a) Streptococcus grupo B.
b) Pseudomona.
c) Escherichia coli.
d) Klebsiella.
e) Peptostreptococcus.
535.Cul de las siguientes patologas cursa con notables incrementos de los niveles sricos de -HCG?
a) Coriocarcinoma.
b) Tumor del seno endodrmico.
c) Carcinoma embrionario.
d) Disgerminoma.
e) Todos los anteriores.
536.Una joven de 17 aos desea mtodos anticonceptivos. Se observa una lesin ulcerativa en el fornix vaginal, borde irregular, enrollado
con una base granular de apariencia rojiza. La lesin es algo sensible a la palpacin. Diagnostico mas probable:
a) Neoplasia intraepitelial vaginal.
b) Carcinoma vulvar.
c) Sfilis.
d) Ulcera por uso de tampones.
e) Herpes genital.
537.Una mujer de 32 aos de edad presenta irregularidades menstruales tipo amenorreico. Tras darle tratamiento con ACO se regulariza el
calendario menstrual, aparece secundariamente al tratamiento emisin lctea por el pezn. Por eso, ella deja el tratamiento, persistiendo la
galactorrea y asociando amenorrea secundaria. En el tratamiento se indica:
a) Estrgenos naturales.
b) Estrgenos sintticos.
c) Estrgenos y gestgenos.
d) Antiandrgenos.
e) Bromocriptina.
538.Seale lo incorrecto sobre las enfermedades de transmisin sexual:
a) El tratamiento de eleccin de la trichonomiasis o giardiasis vaginal es el metronidazol.
b) El chancro blando es producido por Haemophilus ducreyi.
c) El chancro blando se trata con tetraciclinas.
d) El linfogranuloma venreo es causado por Chlamydia trachomatis (L1, L2, L3).
e) Los cuerpos de Donovan aparecen en el linfogranuloma venreo tropical.
539.Mujer de 33 aos presenta masa en mama izquierda de 2 meses. No duele, no secrecin por el pezn ni retraccin cutnea. Su ultima
regla fue hace 2 semanas. Toma ACO (la abuela materna tuvo cncer de mama). Masa de 2 cm, firme, bien definida, oval, en el cuadrante
supero-externo de la mama izquierda, sin adenopata regional. Tejido mamario denso y masa slida bien delimitada. Diagnostico mas
probable:
a) Carcinoma ductal invasor.
b) Carcinoma ductal in situ.
c) Cambios fibroqusticos.
d) Fibroadenoma.
e) Papiloma intraductal.
540.Cul es una medida recomendada como parte del tratamiento de tal paciente?
a) Interrumpir la administracin de ACO.
b) Nueva exploracin al final del siguiente ciclo menstrual.
c) Biopsia con aguja.
d) Biopsia excisional.
e) Repetir la ecografa en seis meses.
REPASO Nro. 1 Pag. 67 USAMEDIC 2017
541.Una mujer de 29 aos (G2, P1) se halla en trabajo de parto rpido. En los minutos de su ingreso est totalmente dilatada con el vrtex
en estacin 0 y empieza a pujar. Contracciones regulares, cada 2-3 minutos y fuertes al ser palpadas. Latidos cardiacos fetales aprox. de
70 lpm. Vrtex en occipitoposterior derecha en estacin 0 sin caput apreciado. Se nota meconio grueso. La actitud al inicio:
a) Instruye a la paciente como ambulatoria.
b) Voltea a la paciente sobre su costado y administra oxgeno por mascarilla.
c) Inicia amnioinfusin y aumenta lquidos IV.
d) Espera alumbramiento vaginal.
e) Realiza cesrea inmediatamente.
542.Sus esfuerzos relativos al caso anterior no rinden cambio en los parmetros, entonces:
a) Toma medidas para realizar cesrea de emergencia.
b) Contina su curso inicial de accin.
c) Pide frceps para alumbramiento vaginal asistido prontamente.
d) Pide un anestesilogo que administre halotano para relajacin uterina.
e) Administra oxitocina y prostaglandina.
543.Mujer de 23 aos con embarazo de 16 semanas presenta sangrado transvaginal leve varias horas luego del coito. No hay dolor abdominal.
Su atencin prenatal curs sin complicaciones y se detect la FCF desde las 12 semanas. Su tero est a mitad de distancia entre la snfisis
del pubis y la cicatriz umbilical, blando y no doloroso. FCF 148 lpm y no hay dilatacin del cuello uterino. Diagnostico mas probable:
a) Amenaza de aborto.
b) Enfermedad trofoblstica gestacional.
c) Cervicitis.
d) Placenta previa.
e) Desprendimiento prematuro de placenta.
544.Una mujer con historia de aborto tardo anterior, ingresa a la 18 semana de gestacin por dolor hipogstrico leve no acompaado de
hemorragia. En la exploracin se aprecia una crvix dilatada y bolsa integra prominente. Dos horas ms tarde se produce un aborto
consumado incompleto. La etiologa mas probable del cuadro es:
a) Sndrome antifosfolpido primario.
b) Insuficiencia cervico-stmica.
c) Mioma uterino de localizacin submucosa.
d) Sndrome de Asherman.
e) Infeccin por micoplasma.
545.Una paciente con antecedentes de isoinmunizacin Rh en el embarazo anterior queda embarazada. Para determinar el grado de
afectacin fetal en este nuevo embarazo se realizar:
a) Test de Coombs indirecto.
b) Genotipo del marido.
c) Amnioscopia y amniografa.
d) Determinacin de bilirrubina en el lquido amnitico.
e) Test de Coombs directo.
546.Mujer de 34 aos, G3, P1, con un aborto inducido y con embarazo de 42 1/7 semanas segn ultrasonido. En su prueba sin estrs se
halla al producto reactivo y el volumen de lquido amnitico 8.5. Dilatacin de 0.5 cm, borramiento de 20%, posicin media, de consistencia
firme con vrtice en la estacin -4. Cul es el siguiente paso en el tratamiento?
a) Oxitocina.
b) Anlogos de prostaglandina.
c) Prueba sin estrs dos veces por semana.
d) Repetir el perfil biofsico (prueba sin estrs y medicin del volumen de lquido amnitico).
e) Rotura artificial de membranas.
547.Uno de estos productos es de gran utilidad para el diagnostico y seguimiento de la enfermedad molar:
a) FSH.
b) Curva de estriol en orina.
c) Niveles de pregnandiol urinario.
d) b-HCG.
e) LPH (Lactgeno placentario).
548.En cul de estas situaciones de gestaciones gemelares se puede practicar un parto vaginal?
a) Gestacin gemelar monoamnitica con el 1 gemelo en ceflica, el 2 en transversa y antecedente de cesrea anterior.
b) Gestacin gemelar bicorial biamnitica, con el 1 gemelo en ceflica y 2 en podlica, en la semana 33 del embarazo.
c) Gestacin gemelar bicorial biamnitica, con el 1 gemelo en ceflica y 2 en transversa, en la semana 36 del embarazo.
d) Gestacin gemelar bicorial biamnitica, con el 1 gemelo en ceflica, 2 en podlica y antecedente de cesrea anterior.
e) Gestacin gemelar bicorial biamnitica, con el 1 gemelo en ceflica, 2 en transversa y antecedente de cesrea anterior.
REPASO Nro. 1 Pag. 68 USAMEDIC 2017
549.Primigravida de 19 aos, a trmino, se halla en trabajo de parto activo 4 horas. Las membranas se acaban de romper; la estacin es 3,
los tonos cardacos fetales (FHT) son 140 y regulares, y la crvix est dilatada 4 cm. Las contracciones son cada 5 minutos y duran
aproximadamente 40 segundos. El mejor de los siguientes planes de manejo es:
a) Paciente ambulatoria.
b) Aumento de oxitocina.
c) Seccin cesrea.
d) Pelvimetra clnica y estimacin del tamao fetal.
e) Voltear a la paciente sobre su costado.
550.En relacin al caso, se halla que 3 horas despus, la crvix est 5 cm dilatada y el patrn de contraccin es irregular, a pesar de la
infusin significativa de oxitocina. La estacin es -2 y la cabeza amoldada. FHT normales. De los siguientes, la mejor eleccin es:
a) Incisiones de Duhrssen.
b) Alumbramiento con frceps.
c) Aumento de oxitocina.
d) Sedacin fuerte.
e) Seccin cesrea.
551.Primigesta en trabajo de parto presenta desaceleraciones tipo II en la monitorizacin. Se hace microtoma fetal obteniendo un pH de
7,10. Dilatacin completa, bolsa rota y III y IV plano. Cul es su actitud?
a) Cesrea.
b) Forceps.
c) Esptulas.
d) Ventosa.
e) Parto espontneo.
552.El efecto secundario ms comn de anticonceptivos orales de baja dosis es:
a) Interrupcin de hemorragia.
b) Cloasma.
c) Dismenorrea.
d) Mastalgia.
e) Nuseas.
553.Una primpara que ha iniciado el perodo de dilatacin hace 5 horas ha alcanzado una dilatacin de 4 cm. Como prevemos un
expulsivo largo, cogemos una muestra fetal para monitorizar el estado del feto. En una primera muestra el pH fue de 7,13. media hora ms
tarde fue de 7,15. Nuestra actitud debe ser:
a) Acelerar y terminar el parto en 1 hora como lmite.
b) Cesrea de inmediato.
c) Administrar uterorelajantes y quitar los oxitcicos.
d) Administrar oxgeno y sueros glucosados a la madre.
e) Seguir el curso del parto con control del pH y si baja de 7,10 cesrea.
554.En cul de estos cuadros obsttrico-ginecolgicos existe un mayor nivel de estriol plasmtico?
a) Embarazo normal.
b) Mola hidatdica.
c) Tecoma ovrico.
d) Corioepitelioma primitivo de ovario.
e) Tumor del seno endodrmico.
555.En el caso de una paciente en la que no hayan descendido sus valores de B-HCG despus de la expulsin de una mola o bien estos
asciendan, el tratamiento de eleccin sera:
a) Azatioprina.
b) Metotrexato.
c) Radioterapia intracavitaria.
d) Histerectoma sin anexectoma.
e) Cisplatino.
556.Una mujer padece sofocos y alteraciones menstruales. El ltimo ao solo sangr de vez en cuando, requiriendo en un episodio
legrado fraccionado, que revela un patrn histolgico dbilmente proliferativo. El tratamiento mas correcto es:
a) Estrgeno continuo con progestgeno cclico
b) Parche de estradiol
c) Estrgeno y testosterona continas
d) Estrgeno y progestgenos continuos
e) Tratamiento no hormonal.
REPASO Nro. 1 Pag. 69 USAMEDIC 2017
557. Una paciente de 35 aos de edad con estenosis mitral y fibrilacin auricular presenta un ACV secundario a embolismo cardiaco. En la
20 semana de gestacin y ante la necesidad de anticoagulacin, usted indicara la utilizacin de:
a) Heparina sdica EV.
b) Heparina clcica IV.
c) Heparina clcica SC.
d) Heparina de bajo peso molecular SC.
e) Dicumarnicos.
558.Cul de los siguientes mtodos permite hacer el diagnstico del embarazo mas precozmente?
a) El tacto vaginal.
b) La determinacin de gonadotropina corinica en sangre.
c) La ecografa transvaginal.
d) La radiografa.
e) El electrocardiograma fetal por va transvaginal.
559.Multpara de 39 aos presenta retencin de placenta 60 minutos luego del parto vaginal de nio sano de 3.6 Kg. No hubo episiotoma
ni laceraciones del perineo, vagina o cuello uterino. Tiene sangrado vaginal intenso y PA 80/50, FC 120 lpm. Fondo uterino firme. La causa
mas probable de la hemorragia puerperal es:
a) Laceracin cervicouterina.
b) Atona uterina.
c) tero de Couvelaire.
d) Trombocitopenia aguda.
e) Placenta accreta.
560.Cul es el tratamiento mas apropiado para el control de la hemorragia?
a) Transfusin de sangre entera.
b) Empacamiento uterino.
c) Histerectoma supracervicouterina (subtotal).
d) Ligadura de arterias hipogstricas.
e) Administracin de metilergonovina intravenosa.
561.Mujer de 27 aos ha utilizado ACO sin problemas por 5 aos. No obstante, lee un artculo con respecto a las complicaciones de los
ACO y pregunta sobre los riesgos y peligros de los ACO. Qu informacin podra proporcionarle en forma correcta?
a) Se incrementa el riesgo de cncer ovrico.
b) Se incrementa el riesgo de desarrollar EIP.
c) Disminuye el riesgo de cncer endometrial.
d) Aumenta el riesgo de tener hijos con anomalas congnitas mayores si se toma ACO durante el embarazo.
e) Se incrementa el riesgo de embarazo ectpico.
562.Gestante de 27 aos; en la 39 semana sufre un dolor violento como pualada en el rea suprapubiana con emisin de sangre negruzca
por la vulva y cese de contracciones. Auscultacin fetal negativa. Diagnostico mas probable:
a) Abruptio placentae.
b) Rotura uterina.
c) Rotura de las varices vaginales.
d) Embarazo ectpico.
e) Placenta previa.
563.La tcnica de eleccin para el diagnostico prenatal de un mielomeningocele en una pareja en situacin de riesgo es:
a) Ecografa fetal en la 6 semana.
b) Ecografa fetal en la 12 semana.
c) Citologa del lquido de amniocentesis.
d) Biopsia corial.
e) Cuantificacin de alfa-protena del suero materno.
564.Una mujer de 35 aos gestante de 30 semanas presenta una metrorragia escasa, de color oscuro, dolor abdominal. Se halla
obnubilada; T 36,5C; 87 lpm; PA 75/30. Auscultacin cardiopulmonar: taquicardia y taquipnea. Palpacin abdominal: tero de
consistencia muy dura que hace imposible palpar las partes fetales. Tacto vaginal: dilatacin cervical. Diagnostico mas probable:
a) Placenta previa.
b) Amenaza de parto pretrmino.
c) Rotura de vasos previos.
d) Desprendimiento prematuro de placenta.
e) Rotura uterina.
REPASO Nro. 1 Pag. 70 USAMEDIC 2017
565.La localizacin mas frecuente de un embarazo ectpico es:
a) Infundibular.
b) Ampular.
c) Istmico.
d) Fmbrico.
e) Ovrico.
566.Si una mujer presenta una leucorrea muy abundante, de color amarillento y espumosa con intenso prurito y un punteado hemorrgico
en la vagina, hay que pensar en una infeccin por:
a) Cndida albicans
b) Trichomona vaginalis
c) Virus herpes tipo II
d) Gardnerella vaginalis
e) Bacilo de Doderlein
567.En cul de los siguientes casos NO estar indicada la colocacin de un DIU?
a) Mujer nulpara de 33 aos de edad.
b) Mujer con antecedentes de aborto.
c) Mujer de 35 aos con estenosis cervical.
d) Mujer con valvulopata.
e) Mujer de 32 aos con antecedentes de EIP.
568.Cul de los siguientes factores de riesgo de parto pretrmino es el mas frecuente en nuestro medio?
a) Condiciones socioeconmicas bajas (subnutricin, exceso de trabajo).
b) Enfermedades y afecciones maternas.
c) Embarazo no deseado.
d) Antecedentes de abortos.
e) Rotura prematura de membranas.
569.La mejor actitud ante el diagnstico ecogrfico de un endometrioma ovrico es:
a) Tratamiento con anlogos de GnRH por 6 meses.
b) Laparotoma exploradora.
c) Laparoscopia quirrgica.
d) Observacin y repetir en 3 meses.
e) Valorar la actitud segn el CA 125.
570.Una mujer de 53 aos de edad inicia un tratamiento hormonal sustitutivo con estrgenos conjugados de forma continua y acetato de
medroxiprogesterona durante los primeros 12 das de cada mes. Presenta una ligera hemorragia poco despus de completar cada ciclo de
progesterona. Cul sera la actitud a tomar?
a) Realizar una biopsia endometrial.
b) Continuar la terapia actual.
c) Aumentar la duracin de la terapia con progesterona.
d) Aumentar la dosis de progesterona.
e) Aumentar la dosis de estrgenos.
571.Cul de estos preparados NO se emplea en el tratamiento mdico de la endometriosis?
a) Progestgenos.
b) Danazol.
c) Gestrinona.
d) Clomifeno.
e) GnRh.
572.La endometriosis sintomtica con dolor intenso, dismenorrea e infertilidad, debe ser tratada por:
a) Histerectoma total con salpingooferectoma bilateral
b) Tratamiento mdico
c) Tratamiento quirrgico conservador
d) Drenaje del endometrioma
e) Vigilancia
REPASO Nro. 1 Pag. 71 USAMEDIC 2017
573.Mujer de 25 aos es operada pues presenta datos clnicos y de laboratorio coincidentes con un cuadro de apendicitis aguda; 23 das
antes haba tenido su ltimo ciclo menstrual con caractersticas normales. Exploracin quirrgica: masa de 4 cm en el ovario derecho, que
abarcaba la mayor parte de la glndula. Aspecto transparente nacarado y en su interior contiene lquido claro. Diagnostico mas probable:
a) Cistoadenoma seroso
b) Quiste dermoide
c) Quiste folicular
d) Quiste del cuerpo lteo
e) Tumor de Brenner
574.El tratamiento del caso anterior es:
a) Utilizacin de progestgenos
b) Ooforectoma y estudio histolgico
c) Cistectoma simple
d) Drenaje externo
e) Aspiracin del contenido del quiste
575.Todos los siguientes son factores de riesgo para la enfermedad plvica inflamatoria, EXCEPTO:
a) Edad menor a 30 aos.
b) Un solo compaero sexual.
c) Uso de DIU.
d) Ectopia cervical.
e) Promiscuidad sexual.
576.Los subtipos 16 y 18 del papilomavirus humano (HPV) se relacionan con:
a) Trastornos linfoproliferativos crnicos de clulas T
b) Linfoma de clulas T
c) Cncer cervical de clulas escamosas
d) Cncer hepatocelular
e) Sarcoma de Kaposi
577.La mejor alternativa para el tratamiento primario del cncer mamario invasivo es:
a) Mastectoma parcial
b) Mastectoma radical modificada
c) Mastectoma preventiva
d) Radioterapia mamaria sin ciruga
e) Quimioterapia y teraputica hormonal
578.Ante una mujer de 32 aos, nuligesta, con un mioma subseroso en el fondo uterino de 12 cm de dimetro, sin dolor ni trastornos
menstruales, la conducta correcta a seguir es:
a) Darle anlogos de la LHRH durante tres meses.
b) Miomectoma.
c) Histerectoma sin anexectoma.
d) Histerectoma con doble anexectoma.
e) Esperar mientras no tenga sintomatologa.

AREA DE SALUD PBLICA Y GESTION

579.Son principios bsicos de la biotica todos los siguientes, EXCEPTO:


a) Beneficiencia.
b) No maleficencia
c) Eficiencia.
d) Justicia.
e) Autonoma
580.Entre las funciones del centro de salud podemos citar:
a) Albergar la estructura fsica de las consultas.
b) Albergar los recursos materiales precisos para la realizacin de las pruebas complementarias.
c) Facilitar el trabajo en equipo de los profesionales sanitarios de la zona.
d) Realizar actividades de promocin y fomento de la salud.
e) Todas son correctas.
REPASO Nro. 1 Pag. 72 USAMEDIC 2017
581.Cmo se denomina el tiempo que se tarda en llegar al centro sanitario ms cercano desde el domicilio del usuario como una variante
de la atencin primaria de la salud definida por la OMS?
a) No debe sobrepasar los 60 min.
b) Se denomina iscrona.
c) Se denomina tiempo de respuesta.
d) Se llama cronograma.
e) Se llama cronologa.
582.El grado en que la asistencia recibida por el usuario concuerda con sus necesidades define a la:
a) Oportunidad.
b) Conveniencia.
c) Accesibilidad.
d) Eficiencia.
e) Suficiencia.
583.Un sistema nacional de salud tiene las siguientes caractersticas, EXCEPTO:
a) Libre acceso de todos los ciudadanos.
b) Financiado por contribuciones de empresarios y empleados.
c) Gestin y control por el gobierno.
d) Propiedad estatal de los medios de produccin.
e) Los mdicos son asalariados.
584.Si hablamos de factibilidad de una intervencin, nos estamos refiriendo:
a) Al costo de la implantacin de dicha intervencin.
b) A la capacidad que tiene la intervencin de solucionar el problema.
c) Al numero de personas que afecta el problema (magnitud del problema).
d) A la aceptabilidad de esa intervencin en el contexto en que se aplique.
e) A la severidad o gravedad del problema.
585.Si un centro de salud est midiendo la calidad de los sistemas de registro, en qu elemento se est centrando?
a) Proceso.
b) Resultado.
c) Acceso.
d) Estructura.
e) Efectividad.
586.Si definimos la evaluacin, diramos que se considera como un medio sistemtico de:
a) Aprender de las acciones realizadas.
b) Mejorar la actividad.
c) Ayudar a la toma de decisiones.
d) Mejorar los resultados.
e) Todas son correctas.
587.Seale los profesionales que forman parte del grupo de apoyo especializado en Atencin Primaria:
a) Enfermera.
b) Medico.
c) Pediatra.
d) Conserje.
e) Farmacutico.
588.Un adulto de 32 aos pesa 75 Kg, este valor corresponde al valor 70 de percentil de peso para adultos de esa edad. Qu significa?
a) El adulto pesa un 70% ms de lo que corresponde a su edad.
b) El 30% de los adultos de esa edad pesan 75 kg o ms.
c) El 70% de los adultos de esa edad pesa menos de 75 kg.
d) El 70% de los adultos de esa edad pesan 75 Kg de media.
e) El 70% de los adultos de esa edad pesan 75 kg o menos.
589.Si usted desea comparar dos mtodos de determinacin de la colesterolemia que utilizan unidades de medidas diferentes, cul de las
siguientes medidas de dispersin le permitir comparar mas correctamente su variabilidad?
a) Desviacin tpica.
b) Coeficiente de variacin.
c) Desviacin media.
d) Varianza.
e) Rango intercuartilico.
REPASO Nro. 1 Pag. 73 USAMEDIC 2017
590.En el departamento de traumatologa del Hospital Naval se hace un estudio a 105 pacientes con artrosis avanzada, midindoles el
dolor mediante una escala de 0 a 10, quedando el 0 definido como ausencia de dolor y el 10 como el dolor de mxima intensidad. La
mediana es 6 en dicho estudio, qu significa este valor de la mediana?
a) La mitad de los sujetos de la muestra tienen valores de intensidad de dolor iguales o inferiores a 6.
b) El 6 indica la intensidad de dolor que puede ser considerada como normal, en la escala utilizada.
c) El 6 significa la diferencia entre la puntuacin mxima y la mnima obtenida por los sujetos de la muestra.
d) El 6 es la media aritmtica de las puntuaciones obtenidas por los sujetos de la muestra.
e) El valor 6 es el menos frecuentemente obtenido por los pacientes de la muestra.
591.Cul de los enunciados se corresponde con el error tipo B en un estudio de contraste de hiptesis?
a) Aceptar la hiptesis alternativa cuando la hiptesis nula es verdadera.
b) Rechazar la hiptesis nula cuando la hiptesis alternativa es verdadera.
c) Rechazar la hiptesis alternativa cuando la hiptesis nula es verdadera.
d) Siendo la hiptesis nula falsa, aceptar la hiptesis alternativa.
e) Aceptar la hiptesis nula cuando la hiptesis alternativa es verdadera.
592.A la probabilidad a posteriori de, habiendo dado negativo a una prueba diagnostica, estar libre de la enfermedad se le denomina:
a) Valor predictivo positivo.
b) Valor predictivo negativo.
c) Sensibilidad.
d) Especificidad.
e) Valor global.
593.La caracterstica fundamental de un test de screening o despistaje de una enfermedad en la poblacin es:
a) Bajo valor predictivo positivo.
b) Alta especificidad.
c) Pocos falsos positivos.
d) Baja especificidad.
e) Alta sensibilidad.
594.Hay una nueva prueba para la deteccin del cncer de prstata. Se valora si la prueba ser til en la prctica al revisar las
caractersticas de sta. Cmo se conoce a la capacidad de la prueba para clasificar en forma correcta a una persona enferma?
a) Especificidad.
b) Sensibilidad.
c) Valor predictivo positivo.
d) Valor predictivo negativo.
e) Reproductibilidad.
595.La medida epidemiolgica que indica la probabilidad de que una enfermedad se desarrolle en un grupo de individuos expuestos a un
factor de riesgo comparada con la de un grupo no expuesto, se denomina:
a) Incidencia acumulada.
b) Densidad de incidencia.
c) Fraccin atribuible.
d) Riesgo relativo.
e) Odds ratio.
596.En un estudio epidemiolgico contamos con una poblacin abierta. La forma mas apropiada para cuantificar la presentacin de una
enfermedad ser:
a) Razn de prevalencia.
b) Incidencia acumulada.
c) Densidad de incidencia.
d) Razn de riesgos.
e) Odds Ratio.
597.Un estudio utilizado para estudiar la relacin entre una exposicin infrecuente en la poblacin general y una enfermedad determinada,
siempre y cuando esa exposicin se asocie fuertemente a dicha enfermedad, se denomina:
a) Estudio de casos y controles.
b) T de Student.
c) Ensayo clnico cruzado.
d) Anlisis de varianza.
e) Estudio de cohortes.
REPASO Nro. 1 Pag. 74 USAMEDIC 2017
598.Cmo se denomina la prevencin que se realiza al reducir la exposicin a las radiaciones de una poblacin determinada?
a) Prevencin primaria.
b) Prevencin secundaria.
c) Prevencin terciaria.
d) Prevencin relativa.
e) Prevencin global.
599.La garanta de calidad es un medio para conseguir todo lo siguiente, EXCEPTO:
a) Garantizar que las personas tengan no slo derecho a la salud, sino tambin a servicios sanitarios de alta calidad.
b) Asegurar el uso de procedimientos de riesgo.
c) Reducir el riesgo producido por la complejidad de la atencin.
d) Reducir las consecuencias indeseables de la competencia.
e) Reducir los trastornos ocasionados por las listas de espera.
600.Las reclamaciones, quejas y sugerencias que el usuario puede ejercer es:
a) Un derecho.
b) Un deber.
c) Una necesidad.
d) Una decisin.
e) Una voluntariedad.
601.Cul es el segundo nivel de la pirmide de Maslow?
a) Necesidad de ego.
b) Necesidad social.
c) Necesidad de autorrealizacin.
d) Necesidades fisiolgicas bsicas.
e) Necesidad de proteccin y seguridad.
602.La medicin de la frecuencia de una enfermedad se puede realizar de cuatro de las siguientes formas, siendo slo una FALSA:
a) Nmero de personas afectas.
b) Proporcin.
c) Rango.
d) Tasa.
e) Razn.
603.Una de las siguientes NO se considera un tipo de evaluacin:
a) De actitudes.
b) De estructura.
c) De proceso.
d) De resultados.
e) De consecuencias.

AREA DE CIENCIAS BASICAS

604.Dnde produce sus principales efectos secundarios el aciclovir?


a) Pulmn.
b) Corazn.
c) Cerebro.
d) Rin.
e) Odo.
605.La hematopoyesis en el embrin, se realiza por primera vez en:
a) Saco vitelino.
b) Bazo.
c) Timo.
d) Hgado.
e) Mdula sea.
606.Las glndulas de Brunner se hallan con mas frecuencia en:
a) Estmago.
b) Duodeno.
c) Intestino delgado.
d) Intestino grueso.
e) Recto.
REPASO Nro. 1 Pag. 75 USAMEDIC 2017
607.El vulo se fecunda en:
a) La superficie del ovario
b) El istmo de la trompa de Falopio
c) El mpula de la trompa de Falopio
d) El extremo de la fimbria o en la primera porcin de la trompa
e) El tero
608.Cul de las siguientes aseveraciones acerca de las clulas del msculo cardiaco es cierta?
a) Tienen forma de huso
b) Requieren un estimulo externo para experimentar contraccin
c) Son clulas multinucleares
d) Estn unidas juntas de extremo a extremo por discos intercalados
e) Poseen numerosas caveolas
609.Qu clulas del epitelio respiratorio son indiferenciadas y actan como clulas madre para reemplazar a otras clulas muertas?
a) Basales.
b) Cilndricas ciliadas.
c) Serosas.
d) En cepillo.
e) Caliciformes.
610.La estructura anatmica mas importante del conducto inguinal es:
a) Aponeurosis del msculo oblicuo mayor del abdomen.
b) Fascia transversalis.
c) Ligamento inguinal.
d) Cordn espermtico.
e) Fibras musculares del oblicuo menor y transverso del abdomen.
611.El nervio craneal que tiene como funcin los movimientos de la laringe, la cabeza, el cuello y los hombros es el:
a) Espinal motor (XI).
b) Hipogloso motor (XII).
c) Trigmino, rama mandibular motor.
d) Vago motor.
e) Glosofarngeo motor.
612.De los siguientes aneurismas, el ms frecuente es de:
a) Arteria iliaca
b) Arteria esplnica
c) Aorta abdominal por encima de los vasos renales
d) Aorta por debajo de los vasos renales
e) Inflamatorios atpicos de la aorta abdominal
613.En el hemograma, en cuanto a la formula leucocitaria, se considera desviacin derecha al aumento del porcentaje de:
a) Eosinfilos y basfilos.
b) Linfocitos y monocitos.
c) Linfocitos y eosinfilos.
d) Neutrfilos abastonados.
e) Neutrfilos segmentados.
614.Los espermicidas destruyen a los espermatozoides..
a) Activando las enzimas acrosmicas.
b) Desbaratando las membranas celulares.
c) Inhibiendo el transporte de glucosa.
d) Alterando las enzimas vaginales.
e) Incrementando el pH vaginal.
615.La va principal para la excrecin de la bilirrubina en el feto en tero es:
a) A travs del rion.
b) Pasaje transplacentario.
c) Degradacin a biliverdina.
d) Reincorporacion a la hemoglobina.
e) Secrecin hepatica y almacenamiento en la luz intestinal.
REPASO Nro. 1 Pag. 76 USAMEDIC 2017
616.Seale la secuela nerviosa mas probable en una fractura de hmero:
a) Mano pndula.
b) Mano de simio.
c) Mano en parrilla.
d) Mano en predicador.
e) Nunca hay lesin nerviosa.
617.Seale un frmaco antiarrtmico del grupo Ia:
a) Procainamida.
b) Bretilio.
c) Propranolol.
d) Amiodarona.
e) Verapamilo.
618.El intercambio de lquidos entre los espacios intersticial e intravascular se regula mediante:
a) Presin osmtica.
b) Flujo linftico.
c) Presin hidrosttica.
d) Presin onctica.
e) Permeabilidad vascular.
619.La funcin del osteoblasto es la de:
a) Mantener un periostio intacto.
b) Depositar en la matriz sales de calcio previamente secretadas.
c) Disolver sales de calcio del tejido seo necrtico.
d) Proporcionar una superficie articular al hueso largo.
e) Secretar una matriz de colgena y mucopolisacridos para la neoformacin de hueso.
620.La arteria gastroepiploica izquierda es una rama de la arteria:
a) Esplnica.
b) Coronaria estomquica.
c) Heptica comn.
d) Gastroduodenal.
e) Gstrica izquierda.
621.Un hombre de 72 aos de edad llega a la consulta de su mdico para su revisin anual. Cul de las estructuras siguientes se palpa
ms fcilmente en el transcurso de la exploracin rectal?
a) Prstata.
b) Epiddimo.
c) Conducto eyaculador.
d) Urter.
e) Testculo.
622.Una de las principales funciones del surfactante es:
a) Aumentar la compliancia pulmonar
b) Elevar la tensin superficial alveolar
c) Incrementar el trabajo de la respiracin
d) Aumentar la tendencia pulmonar al colapso
e) Lubricar los alvolos
623.Cul de los siguientes es verdadero respecto al xido ntrico endgeno en nuestra fisiologa?
a) Acta a travs de estimulacin de la adenilatociclasa.
b) Produce contraccin muscular por fosforilacin de la miosina.
c) Vasodilatador que acta a travs de las endotelinas.
d) Su formacin es dependiente de la presencia de L-glutamina.
e) Se produce exclusivamente en las clulas endoteliales.
624.La complicacin grave mediata de la succinilcolina se observa de preferencia en pacientes con:
a) Insuficiencia renal
b) Quemaduras
c) Aplastamiento neuromuscular
d) Enfermedades de la neurona superior
e) Arritmias ventriculares
REPASO Nro. 1 Pag. 77 USAMEDIC 2017
625.Varn de 50 aos con fibrilacin auricular toma warfarina. Con cul de las siguientes opciones disminuye el efecto de la warfarina?
a) La presencia de una deficiencia de la vitamina K.
b) Fenilbutazona.
c) Quinidina.
d) Barbitricos.
e) Tirotoxicosis.
626.La mayora de agentes para profilaxis de derrame cerebral muestran un efecto antiplaquetario. Cul afecta la cascada de
coagulacin?
a) Warfarina.
b) Aspirina.
c) Ticloplidina.
d) Clopidogrel.
e) Dipiridamola.
627.La mayora de frmacos gana entrada a las clulas por:
a) Difusin pasiva con cintica de orden cero
b) Difusin pasiva con cintica de primer orden
c) Transporte activo con cintica de orden cero
d) Transporte activo con cintica de primer orden
e) Difusin pasiva a travs de poros de membrana
628.Cul de los siguientes describe mejor los tres estrgenos principales en las mujeres en orden decreciente de potencia?
a) Estradiol, estriol, estrona.
b) Estradiol, estrona, estriol.
c) Estriol, estradiol, estrona.
d) Estriol, estrona, estradiol.
e) Estrona, estriol, estradiol.
629.Las partculas muy pequeas son removidas del sistema respiratorio por:
a) Flujo de residuos.
b) Difusin.
c) Expectoracin.
d) Fagocitosis.
e) Transporte ciliar.
630.El endomisio es un revestimiento de tejido conectivo que envuelve:
a) Cada clula muscular.
b) Los fascculos musculares.
c) Las miofibrillas individuales.
d) Un msculo entero.
e) Pequeos haces de clulas musculares.
631. Cul de los siguientes nervios se encarga del control motor del msculo masetero?
a) Facial.
b) Crvico-facial.
c) Maxilar superior.
d) Maxilar inferior.
e) Oftlmico.
632. Cul de las siguientes arterias, transcurre por la curvatura menor del estmago?
a) Gastroomental izquierda.
b) Gastroduodenal.
c) Coronaria estomquica.
d) Grastroomental derecha.
e) Vasos cortos.
633.En la mujer, los vestigios de la glndula prosttica corresponden a:
a) Cltoris
b) Glndulas del cltoris
c) Glndulas vestibulares
d) Glndulas parauretrales
e) Bulbo del vestbulo
REPASO Nro. 1 Pag. 78 USAMEDIC 2017
634.La mayor parte del calcio del cuerpo humano se concentra en:
a) Liquido extracelular y tejidos blandos.
b) Suero.
c) Esqueleto.
d) Estado libre.
e) Paratiroides.
635.Cul es el nervio lesionado en un paciente con imposibilidad de extender la mueca y los dedos?
a) Cubital.
b) Mediano.
c) Radial.
d) Msculocutneo.
e) Axilar.
636.Cul de las siguientes clulas se hallan en la sustancia blanca de la mdula espinal?
a) Clulas de Schwann
b) Clulas ependimarias
c) Oligodendrocitos
d) Clulas piramidales
e) Motoneuronas alfa
637.Qu arteria de las siguientes puede causar gangrena si se liga?
a) Femoral superficial
b) Branquial
c) Radial
d) Popltea
e) Cubital
638.El conducto lacrimonasal drena en el cornete nasal
a) Superior.
b) Supremo.
c) Medio.
d) Inferior.
e) Lateral.
639.El efecto de Wolff-Chaikoff se presenta al administrar:
a) Propiltiouracilo.
b) Yodo radiactivo.
c) Goitirina.
d) Yoduros.
e) Indometacina.
640.Qu grupo ganglionar recibe la mayor cuanta del flujo linftico de la mama?
a) Ganglios paraesternales.
b) Ganglios intercostales.
c) Ganglios axilares.
d) Ganglios diafragmticos.
e) Ganglios abdominales.
641.La siguiente gasometra arterial : pH 7.40, PaO2 98, PaCO2 38, HCO3 25, refleja:
a) Acidosis metablica.
b) Alcalosis metablica.
c) Valores normales.
d) Alcalosis respiratoria.
e) Acidosis respiratoria.
642.La hipertricosis es un efecto adverso de uno de los siguientes:
a) Reserpina
b) Hidralazina
c) Amiodarona
d) Hidralazina
e) Minoxidil.
REPASO Nro. 1 Pag. 79 USAMEDIC 2017
643.Una de las siguientes arterias NO irriga el recto y el canal anal:
a) Arteria sacra.
b) Arteria marginal.
c) Arteria hemorroidal superior.
d) Arterias hemorroidales medias.
e) Arterias hemorroidales inferiores.
644.A qu clase de antiarrtmicos pertenece la lidocana?
a) Beta bloqueantes.
b) Bloqueantes de los canales de sodio.
c) Bloqueantes de los canales de calcio.
d) Bloqueantes de los canales de potasio.
e) Bloqueantes de los canales de magnesio.
645.Seale la afirmacin incorrecta sobre las hormonas:
a) La TRH tiene una funcin liberadora de PRL.
b) Existe un pico de FSH en la mitad del ciclo.
c) Existe un pico de LH en la mitad del ciclo.
d) Las gonadotropinas se producen en el lbulo posterior de la hipfisis.
e) La GnRH se libera de forma pulstil.
646.En el estado de choque, la liberacin de angiotensina II produce:
a) Aumento de la permeabilidad capilar
b) Aumento de la aldosterona
c) Vasoconstriccin
d) Depresin del sistema reticuloendotelial
e) Depresin en las funciones del corazn
647.Hormonas reguladoras del metabolismo de calcio y fsforo:
a) PTH, vitamina D, calcitonina.
b) PTH, bifosfonatos, estrgenos.
c) Vitamina D, magnesio, calcitonina.
d) Albumina, PTH, calcitonina.
e) Solo PTH.
648.Las vacunas como mecanismo de inmunidad en las infecciones constituyen:
a) Resistencia inespecfica.
b) Inmunidad adquirida artificial pasiva.
c) Inmunidad adquirida natural activa.
d) Inmunidad adquirida artificial activa.
e) Inmunidad adquirida natural pasiva.
649.Mujer de 40 aos con diagnstico de embarazo de 14 semanas de gestacin. Antecedentes de importancia: 2 embarazos, en los
cuales no hubo complicaciones, los dos fueron partos. Los productos sin alteraciones. Acude preocupada, pues sabe que los embarazos a
edades mayores se asocian a alteraciones en los nios. En qu momento de la meiosis se realiza las dos detenciones en la ovognesis?
a) Paquitena y anafase II.
b) Diplotena y metafase II.
c) Metafase I y metafase II.
d) Diacinesis y telofase II.
e) Leptotena y anafase II.
650.Los 3 sistemas excretores se forman sucesivamente, con transitoria sobreposicin en el perodo embrionario. En secuencia de desarrollo
son:
a) Pronefros, mesonefros, metanefros
b) Pronefros, metanefros, mesonefros
c) Metanefros, pronefros, mesonefros
d) Cordn nefrognico, pronefros, mesonefros
e) Cordn nefrognico, pronefros, tbulos pronefricos
651.En los ltimos estados del embarazo, la sangre materna es separada de la sangre fetal por el:
a) Sincitiotrofoblasto solamente.
b) Citotrofoblasto solamente.
c) Sincitiotrofoblasto y citotrofoblasto.
d) Sincitiotrofoblasto y endotelio fetal.
e) Citotrofoblasto y endotelio fetal.
REPASO Nro. 1 Pag. 80 USAMEDIC 2017
652.Al tbulo contorneado distal del rin se le diferencia histolgicamente del tbulo proximal por su:
a) Curso ms contorneado.
b) Epitelio ms alto.
c) Falta de borde en cepillo.
d) Seccin transversal que muestra menos clulas por dimetro.
e) Sus ncleos situados ms lejos de la luz.
653.Cul de las siguientes clulas interviene en la produccin de anticuerpos?
a) Glbulo rojo.
b) Linfocito B.
c) Plaqueta.
d) Neutrfilo.
e) Basfilo.
654.En cul tejido abundan mas los macrfagos?
a) Tejido conectivo laxo (areolar).
b) Tejido conectivo denso regular.
c) Tejido adiposo pardo.
d) Tejido conectivo denso irregular.
e) Tejido embrionario.
655.Por el tnel carpiano NO pasa el:
a) Nervio mediano
b) Nervio cubital
c) Flexor comn superficial
d) Flexor comn profundo
e) Extensor largo del pulgar
656.Si hay dao a este nervio el paciente presenta ligera debilidad en el arco lateral del pie sobre un lado y NO podr dorsiflexionar el pie:
a) Peroneo superficial
b) Plantar lateral
c) Peroneo profundo
d) Sural
e) Tibial posterior
657.Si el paciente tiene este nervio afectado, consecuentemente hay dificultad para oponer el dedo pulgar y atrofia de la eminencia tenar:
a) Mediano
b) Cubital
c) Radial
d) Circunflejo
e) Braquial cutneo
658.El nodo sinoauricular del corazn recibe la mayor parte de su irrigacin de la arteria:
a) Coronaria derecha.
b) Coronaria izquierda circunfleja.
c) Coronaria izquierda interventricular anterior.
d) Coronaria izquierda descendente anterior.
e) Coronaria izquierda ascendente posterior.
659.De los siguientes, hay un frmaco que administrado en monodosis (1 gramo) es til para el tratamiento de las uretritis no
gonoccicas:
a) Lansoprazol.
b) Misoprostol.
c) Ticlopidina.
d) Azitromicina.
e) Claritromicina.
660.Cul es el efecto secundario mas frecuente del fenobarbital?
a) Anemia aplsica.
b) Dermatitis exfoliativa.
c) Hepatitis fulminante.
d) Sedacin.
e) Epilepsia.
REPASO Nro. 1 Pag. 81 USAMEDIC 2017
661.El agente farmacolgico de eleccin para la esquistosomiasis es:
a) Praziquantel.
b) Ivermectina.
c) Mebendazol.
d) Metronidazol.
e) Clindamicina.
662.En un electrocardiograma estndar la derivacin DII se registra cuando se conecta el polo positivo en
a) La pierna izquierda y polo negativo en el brazo derecho.
b) El brazo izquierdo y polo negativo en el brazo derecho.
c) La pierna izquierda y polo negativo en el brazo izquierdo.
d) El brazo izquierdo y polo negativo en los dos restantes.
e) El brazo derecho y polo negativo en los dos restantes.
663.Dnde ocurre la secrecin del hierro?
a) Fondo del estmago.
b) Antro del estmago.
c) Duodeno del intestino.
d) Ileon del intestino.
e) Colon.
664.El surfactante pulmonar es un compuesto complejo primariamente consistente en:
a) Proteina A del surfactante.
b) Proteina B del surfactante.
c) Lipidos neutrales.
d) Agua.
e) Fosfolipidos.
665. Cul es la va ascendente de la mdula espinal que conduce los impulsos sensitivos de la piel, los msculos, los tendones y el tacto
fino?
a) Espinotalmica anterior.
b) Espinotalmica lateral.
c) Fascculo grcil y fascculo cuneiforme.
d) Espinocerebelo anterior.
e) Espinocerebelo posterior.
666.La onda QRS en un ECG se produce por:
a) Repolarizacin de las aurculas.
b) Despolarizacin de las aurculas.
c) Repolarizacin de los ventrculos.
d) Despolarizacin de los ventrculos.
e) Segmento ST.
667.Cul de los siguientes defectos es ocasionado por ausencia de la formacin del septo aorticopulmonar en forma de espiral en 180?
a) Dextrocardia.
b) Comunicacin interventricular de tipo membranosa.
c) Comunicacin interauricular de tipo ostium secundum.
d) Tetraloga de Fallot.
e) Transposicin de las grandes arterias.
668.Los podocitos de la cpsula de Bowman proceden del:
a) Ectodermo.
b) Endodermo.
c) Mesodermo.
d) Endodermo y mesodermo.
e) Clulas de la cresta neural.
669.Un paciente presenta infeccin parasitaria. Para elegir el tratamiento adecuado se analiza la sangre perifrica. En cul de las clulas
esperara encontrar incremento en el frotis de sangre perifrica?
a) Linfocitos.
b) Basfilos.
c) Neutrfilos.
d) Monocitos.
e) Eosinfilos.
REPASO Nro. 1 Pag. 82 USAMEDIC 2017
670.La mielinacin de los axones de gran calibre en los nervios perifricos es una funcin de:
a) Astrocitos
b) Fibroblastos
c) Oligodendrocitos
d) Clulas perineurales
e) Clulas de Schwann
671.Los principales blancos naturales de las clulas citotxicas T son:
a) Transplantes tisulares alogenicos
b) Bacterias
c) Toxinas bacterianas
d) Protozoarios
e) Clulas infectadas por virus
672.Un paciente con dificultad para oponer el dedo pulgar y con atrofia de la eminencia tenar, tendr afectado uno de los siguientes nervios:
a) Cubital.
b) Radial.
c) Circunflejo.
d) Mediano.
e) Braquial cutneo.
673.En el movimiento de retropulsin mandibular propiamente dicha, participa uno de los siguientes msculos:
a) Fascculo superficial del masetero.
b) Temporal.
c) Pterigoideo.
d) Geniogloso.
e) Bucinador.
674.Uno de los siguientes es un msculo NO incluido en el triangulo femoral:
a) Aductor mediano
b) Sartorio
c) Psoas iliaco
d) Recto interno
e) Pectineo
675.Cul msculo es un rotador externo del muslo?
a) Recto interno.
b) Bceps femoral.
c) Semitendinoso.
d) Sartorio.
e) Semimembranoso.
676.El uso mas comn de los interferones humanos es en:
a) Condiloma acuminado.
b) Enfermedad granulomatosa crnica.
c) Leucemia de clulas peludas.
d) Hepatitis crnica causada por hepatitis B, C y D.
e) Sarcoma de Kaposi.
677.De las siguientes, cul o cules son pioneras en el grupo de las penicilinas de amplio espectro?
a) Ticarcilina.
b) Piperacilina.
c) Ampicilina y amoxicilina.
d) Ureidopenicilinas.
e) Mezlocilina.
678.La eritromicina pertenece al grupo de:
a) Los aminoglucsidos.
b) Los macrlidos.
c) Los polipptidos.
d) Los polienos.
e) Las tetraciclinas.
REPASO Nro. 1 Pag. 83 USAMEDIC 2017
679.La fructosa es una fuente de energa para el espermatozoide, y se halla principalmente en las secreciones de:
a) Vesculas seminales.
b) Glndulas bulbouretrales.
c) Testculos.
d) Glndula prosttica.
e) Epiddimo.
680.Cul es el tratamiento de eleccin de la disnea en la fase terminal de la enfermedad?
a) Broncodilatadores.
b) Morfina.
c) Oxigenoterapia al 35% por mascarilla.
d) Digital y diurticos.
e) Antibiticos y fisioterapia respiratoria.
681.Una de las siguientes pueden ser entidades distintas o fusionadas:
a) Placenta bipartita.
b) Placenta circunvalada.
c) Placenta de embarazo mltiple.
d) Placenta accreta.
e) Placenta previa.
682.Cul de las siguientes caractersticas histolgicas permitira fechar el endometrio de una mujer en edad menstrual, en una biopsia
endometrial?
a) Nmero de arterias por unidad de rea de superficie en el endometrio.
b) Grosor del endometrio, aspecto de las glndulas y edema del estroma.
c) Cantidad de moco dentro de las glndulas.
d) Cantidad de hemorragia dentro de las glndulas.
e) Ancho y cantidad de cilios en las glndulas endometriales y coloracin eosinoflica de la basal.
683.De cul de los siguientes se deriva el seno urogenital?
a) Invaginacin de las crestas gonadales.
b) Proliferacin del intestino posterior.
c) Particin de la cloaca endodrmica.
d) Un problema en desarrollo en el mesodermo genital.
e) Hiperplasia del metanefros.
684.Al realizar una ciruga, la posicin de estructuras importantes debe conocerse bien para evitar lesiones. Cul es la relacin del urter
con las arterias en su trayecto a travs de la pelvis?
a) Anterior a las arterias iliaca interna y uterina.
b) Posterior a la arteria iliaca y anterior a la arteria uterina.
c) Anterior a la arteria uterina y posterior a la arteria iliaca.
d) Posterior a la arteria uterina y medial a la arteria iliaca.
e) Posterior a la arteria uterina y posterior a la arteria hipogstrica.
685.Uno de los siguientes hiatos NO se encuentra en el diafragma:
a) Hiato de Bochdaleck.
b) Hiato esofgico.
c) Hiato de Winslow.
d) Hiato artico.
e) Hiato de Morgagni.

S-ar putea să vă placă și